Download as pdf or txt
Download as pdf or txt
You are on page 1of 177

CH-9(RAY OPTICS AND OPTICAL INSTRUMENTS)

Session 9.1(Reflection and Spherical Mirrors)


1. Optics:
It is a branch of physics which deals with the study of nature, production and propagation of light.
It is divided into two sub branches: ray optics (or geometric optics) and wave optics (or physical optics).
Ray optics is based on the rectilinear propagation of light and deals with reflection of light and the formation
of images by mirrors and lenses.
2. Light:
Electromagnetic radiations belonging to the visible region of spectrum (wavelength(λ)-400nm to 750nm) is
called light.
Light travels with a speed of 3×108m/s in vacuum.
The path along which light travels between two points in a straight line is called ray of light.
Bundle of light rays is known as beam of light.
3. Reflection of Light:
The phenomenon of bouncing back of light in the same medium when the light falls on a surface is known as
reflection of light.
4. Laws of reflection:
(a) First law:
The incident ray, the reflected ray and the normal
to the reflecting surface at the point of incidence
all lie in the same plane.
(b) Second law:
The angle of incidence is equal to the angle of
reflection, i.e, ∠i = ∠r.
(NOTE: Angles are always measured with respect to normal axis in optics.)

A ray of light falling normally on a


mirror retraced its path.

For normal incidence ∠i = 0


therefore ∠r=0

1
5. Images:
• Virtual Images:
Virtual images are images that cannot be formed on screen.
Virtual images are formed when light rays appear to meet but do not
actually meet.
Virtual images are formed behind the mirror.
Virtual images are erect.
• Real Images:
A real image is an image that can be obtained on screen.
Real images are formed when light rays actually meet at a point.
Real images are formed in front of the mirror.
Real images are inverted.
Image seen on a screen at a movie theater is a real image whereas the
image one sees in a flat mirror is not a real image, but rather a virtual image.

6. Properties of image by Plane mirror:


(i) Image formed is virtual and upright or erect.
(ii) The object distance is equal to the image distance, i.e., the image is at the same distance behind the
mirror as the object is in front of the mirror.
(iii) Size of the image is same as the size of the object.

(iv) The image is laterally inverted that means left of the object appears right and right appears left.

7. Spherical mirrors:
They are the reflecting surface
which forms part of a hollow
sphere.
Two types of spherical mirrors are:
(i) Concave mirror
(converging mirror):
It is a spherical mirror in
which the outer bulged
surface is silvered polished
and the reflection of light
takes place from the inner
hollow surface.
These are also known as converging mirrors as light rays after striking spherical surface will converge
(fig 3(i)).
(ii) Convex mirror (diverging mirror):
It is a spherical mirror in which the inner hollow surface is silvered polished and the reflection of
light takes place from outer bulged surface.
These are also known as diverging mirrors as light rays after striking spherical surface will diverge (fig
3(ii)).

2
8. Sign Convention used:

9. Important terms in the case of spherical mirrors:


(a) Pole: It is the middle point of the spherical mirror.
(b) Center of curvature: It is the centre of the sphere of which the mirror forms a part of.
(c) Radius of curvature: It is radius of the sphere of which the mirror is a part of.
(d) Principal Axis: The line passing through the pole and the centre of curvature of the mirror is called the
principal axis.
(e) Linear aperture: It is the diameter of the circular boundary of the spherical mirror.
(f) Principal focus: Narrow beams of light incident parallel to the principal axis either actually converges to
or appears diverge from a point on the principal axis after reflection from the spherical mirror.
(g) Focal length: It is the distance between the focus and the pole of the mirror.
(h) Paraxial Rays: Rays which are close to the optical axis of the system, and makes small angle with the
optical axis of the system.
10. Spherical Aberration:
The inability of a spherical mirror of large aperture to bring all the rays
of wide beam of light falling on it to focus at a single point is called
spherical aberration. It causes formation of blurred image.
Spherical aberration can be reduced by following ways:
i) By using spherical mirrors of small apertures.
ii) By using parabolic mirrors.
Fig 5
11. Rules for drawing the ray diagram to show image formation:
(i) A ray parallel to the principal axis, after reflection, will pass
through the principal focus in case of a concave mirror or
appear to diverge from the principal focus in case of a convex
mirror.

Fig 6

(ii) A ray passing through the principal focus of a concave mirror or a ray
which is directed towards the principal focus of a convex mirror, after
reflection, will emerge parallel to the principal axis.

Fig 7
(iii) A ray passing through the centre of curvature of a concave mirror or
directed in the direction of the centre of curvature of a convex mirror,
after reflection, is reflected back along the same path.

Fig 8

3
(iv) A ray incident obliquely to the principal axis, towards a point P (pole
of the mirror), on the concave mirror or a convex mirror, is reflected
obliquely. The incident and reflected rays follow the laws of
reflection at the point of incidence, making equal angles with the
principal axis.

Image formation by concave mirrors:


(a)When object is at infinity: (b) When object is beyond C: Fig 9

Fig 10(a)
Fig 10(b)

(c) When object is at centre of curvature: (d) When object is between F and C:

Fig 10(c) Fig 10(d)

(e)When object is at focus: (f) When object is between pole and focus:

Fig 10(e) Fig 10(f)


12. Image formation by convex mirrors:
(a)When object is at infinity: (b) When object is between infinity and the pole:

Fig 11(b)
Fig 11(a)

4
13. Relationship between the focal length & radius of curvature of spherical mirror:

Consider a ray parallel to the principal axis striking the (a) concave,
(b) convex mirror at point M. The aperture of the mirror is small.
Let C be center of curvature of mirror as shown in figure 12(a).
Let θ be angle of incidence and MD normal from M on the principal
axis.
Then, ∠MCP = θ (Alternate interior angles)
By law of reflection, ∠i = ∠r.
So, ∠CMF = θ
So, from fig 12(a) & 12(b), ∠MFP = 2θ (Exterior angle theorem)
In ΔCMD, Fig 12(a)
𝑀𝐷
tan θ = 𝐶𝐷
In ΔFMD
𝑀𝐷
tan 2θ = 𝐹𝐷
For paraxial rays, θ is very small ⟹tanθ ≈ θ & tan2θ ≈ 2θ
𝑀𝐷 𝑀𝐷
∴ θ = 𝐶𝐷 & 2θ = 𝐹𝐷
𝑀𝐷 𝑀𝐷
⟹ 2( 𝐶𝐷 ) = 𝐹𝐷 Fig 12(b)
𝐶𝐷
⟹ FD = 2
Since the aperture of the mirror is small, point D is very close to pole P.
∴ FD = focal length ‘f’ and CD = Radius of curvature = ‘R’
𝑹
⟹f = or Focal length = ½ ×Radius of curvature
𝟐

14. The Mirror Equation(Must Do - Examiner Favourite):


It is a mathematical relationship between the object distance ‘u’, image distance ‘v’ & focal length ‘f’ of
spherical mirror.
• Mirror equation for concave mirror:
Consider an object AB placed on principal axis beyond center of curvature ‘C’ of concave mirror of small
aperture, as shown in fig 13.
For paraxial rays and mirrors of small aperture, MP can be considered to be a straight line normal to CP.
After reflection from the mirror, a real and inverted image A’B’ is formed.
From figure 10, two right angled Δ A’B’C & ΔABC are similar (By AA similarity).
Since ΔA’B’C ~ ΔABC,
𝐴′𝐵′ 𝐶𝐵′
𝐴𝐵
= 𝐶𝐵
→①
Also, ΔABP~ ΔA’B’P, (By AA similarity)
𝐴′𝐵′ 𝑃𝐵′
∴ 𝐴𝐵
= 𝑃𝐵
→②
n
Comparing eq s ①&②,
𝐶𝐵′ 𝑃𝐵′
𝐶𝐵
= 𝑃𝐵
𝑃𝐶−𝑃𝐵′ 𝑃𝐵′
⟹ 𝑃𝐵−𝑃𝐶
= 𝑃𝐵 →③
Applying sign convention for the concave mirror,
PB’ = -v, PC =−𝑅 , PB = -u.
Using these in eqn ③, Fig 13
−𝑅 –(−𝑣) −𝑣
⟹ −𝑢−(−𝑅) = −𝑢
−𝑅+ 𝑣 𝑣
⟹ −𝑢+𝑅
=𝑢
⟹ −uR + uv = −uv + vR uR + vR = 2uv
Dividing throughout by 𝑢𝑣𝑅,
1 1 2
• Linear Magnification ⟹ 𝑣 + 𝑢 = 𝑅
𝟏 𝟏 𝟏
⟹ 𝐯+𝐮=𝐟
This is known as the mirror equation.
5
Linear magnification (m):
It is defined as the ratio of the size of the image (h’) to the size of the object (h).
h′
m= h
Figure shows the formation of the image A’B’ of the object AB by the concave mirror.
ΔABP ~ ΔA’B’P (By AA similarity)
A′B′ PB′
AB
= PB
→①
Applying sign conventions:
AB = h; A’B’ = −h’; PB = −u; PB ′ = −v
• Mirror equation
h′ −v
for convex mirror: (Do it yourself)
Eqn. ① becomes , − h = −u
h′ −v
⟹m = h
= u
Linear Magnification in terms of u and f:
v f
⟹m = - =
u f−u
Linear Magnification in terms of v and f:
v f−v
⟹m = - u = f
Linear magnification in different cases:
i) If |m|>1, the image is magnified.
ii) If |m|<1, the image is diminished.
iii) If |m|=1, the image is of same size as the object.
iv) If m is positive (or v is positive), the image is virtual and erect.
v) If m is negative (or v is negative), the image is real and diminished
Mirror equation for convex mirror:
vi) In case of concave mirror m may be positive or negative , in case of convex mirror m is always
positive
vii)
Mirror equation for concave mirror

Consider an object AB placed on principal axis in front of convex mirror of small aperture, as shown in fig 14.
For paraxial rays and mirrors of small aperture, MP can be considered to be a straight line normal to CP.
After reflection from the mirror, a virtual and erect image A’B’ is formed.
From figure 14, two right angled Δ A’B’C & ΔABC are similar (By AA similarity).
Since ΔA’B’C ~ ΔABC,
𝐴′𝐵′ 𝐶𝐵′
𝐴𝐵
= 𝐶𝐵
→①
Also, ΔABP~ ΔA’B’P, (By AA similarity)
𝐴′𝐵′ 𝑃𝐵′
∴ 𝐴𝐵
= 𝑃𝐵
→②
Comparing equations ①&②,
𝐶𝐵′ 𝑃𝐵′
𝐶𝐵
= 𝑃𝐵
𝑃𝐶−𝑃𝐵′ 𝑃𝐵′
⟹ 𝑃𝐵+𝑃𝐶
= 𝑃𝐵 →③
Applying sign convention for the convex mirror,
PB’ = v, PC = 𝑅 , PB = -u.
Using these in eqn ③,
𝑅 –𝑣 𝑣
⟹ −𝑢+𝑅 = −𝑢
𝑅− 𝑣 𝑣
Fig 14
⟹ =
𝑢−𝑅 𝑢
⟹ uR − uv = uv − vR uR + vR = 2uv
1 1 2
Dividing throughout by 𝑢𝑣𝑅, 𝑣 + 𝑢 = 𝑅
𝟏 𝟏 𝟏
⟹ 𝐯+𝐮=𝐟
This is known as the mirror equation.
Solved Examples

6
Eg 1: How is the focal length of a spherical mirror affected when the wavelength of the light used is
increased? [Foreign 2010] [1 mark]
Ans: Focal length of spherical mirror remains unaffected as it depends only on radius of curvature of sphere.

Eg 2: An object is placed at a distance of 40cm from a concave mirror of focal length 15cm. If the object is
displaced through a distance of 20 cm towards the mirror, by how much distance is the image displaced?
[2 marks]
Sol: Initially, u = −40cm, f = −15cm
1 1 1
Using mirror equation, 𝑣 + 𝑢 = 𝑓, we get,
1 1 1
⟹𝑣=𝑓−𝑢
1 −1 (−1)
⟹ 𝑣 = 15 – 40
1 −1 1
⟹ = +
𝑣 15 40
1 −8+3
⟹ 𝑣
= 120
120
⟹ v=
−5
⟹ v = −24cm
When object is displaced towards the mirror, then let u’ and v’ be the new object and image distance.
Hence, u’ = -40+20= -20cm
1 1 1
Using mirror equation, + = , we get,
𝑣 𝑢 𝑓
1 1 1
⟹ 𝑣′ = 𝑓 − 𝑢′
1 −1
(−1)
⟹ 𝑣′ = 15 – 20
1 −1 1
⟹ = +
𝑣′ 15 20
1 −4+3
⟹ 𝑣′
= 60
60
⟹ v’ =
−1
⟹ v’ = −60cm
Thus, displacement in image = 60 – 24 = 36cm
Ans. Displacement of image will be by 36cm.

Eg 3: Use the mirror equation to show that (a) an object placed between f and 2f of a concave mirror
produces a real image beyond 2f, (b) a convex mirror always produces a virtual image independent of the
location of the object, (c) an object placed between the pole and focus of a concave mirror produces a
virtual and enlarged image. [Compartment 2011, Outside Delhi 2011] [3 marks]
Sol: (a)For a concave mirror, the focal length (f) is negative, i.e., f<0.
When the object is placed on the left side of the mirror, the object distance (u) is negative, i.e., u<0.
From Mirror Equation,
1 1 1
𝑣
+𝑢=𝑓
1 1 1
⟹𝑣=𝑓−𝑢 ⟶①
The object lies between f and 2f.
∴2f < u < f (∵u and f are –ve)
1 1 1
⟹ 2𝑓>𝑢>𝑓
1 1 1
⟹ − 2𝑓<− 𝑢<− 𝑓
1 1 1 1 1 1 1
⟹ − < − < − (Adding on both sides)
𝑓 2𝑓 𝑓 𝑢 𝑓 𝑓 𝑓
From ①,

7
1 1
⟹ < < 0 ⟶②
2𝑓 𝑣
1
∴ is negative, i.e., v is negative.
𝑣
1 1
From ②, <
2𝑓 𝑣
⟹ 2f > v
⟹ -2f< -v
Therefore, image lies beyond 2f.

(b) For a convex mirror, the focal length (f) is positive, i.e., f>0
When the object is placed on the left side of the mirror, the object distance (u) is negative, i.e., u<0.
From Mirror Equation,
1 1 1
𝑣
+𝑢=𝑓
1 1 1
⟹ = − ⟶①
𝑣 𝑓 𝑢
1 1
Now, f>0 and u<0, thus, > 0 and < 0.
𝑓 𝑢
1 1
⟹ − >0
𝑓 𝑢
From ②,
1
⟹ 𝑣> 0
⟹v>0
Thus, the image is formed behind the mirror.
Hence, a convex mirror always produces a virtual image, regardless of the object distance
(c) For a concave mirror, the focal length (f) is negative, i.e., f<0.
When the object is placed on the left side of the mirror, the object distance (u) is negative, i.e., u<0.
From Mirror Equation,
1 1 1
𝑣
+𝑢=𝑓
1 1 1
⟹𝑣=𝑓−𝑢 ⟶①
Since object is placed between focus and pole,∴f <u < 0
1 1
⟹ >
𝑓 𝑢
1 1
⟹ − >−
𝑓 𝑢
1 1 1 1
⟹ 𝑓
− 𝑓<𝑓 − 𝑢From ①
1
⟹ 0<𝑣
⟹v>0
The image is formed behind the mirror. Hence, it is a virtual image.
For u< 0 and v> 0, we can write:
1 1
⟹ 𝑢 <𝑣
⟹v<u
𝑣
⟹ 𝑢< 1
−𝑣
⟹ 𝑢
> 1
⟹m>1
Hence, the image formed is enlarged.

8
Eg 4: A mobile phone lies along the principal axis of a concave mirror, as shown in Fig 15. Show by suitable
diagram, the formation of its image. Explain why the magnification is not
uniform. Will the distortion of image depend on the location of the
phone with respect to the mirror? [Outside Delhi 2013]
[3 marks]
Sol: Image of the mobile phone is shown by A’B’C in fig 14.
Magnification is not uniform, because the object is not a point sized
object. Hence, u (object distance) is not same for every point on object.
1 1 1
Thus, by relation, + = , v is different for each and every point. Hence,
𝑣 𝑢 𝑓 Fig 15
−𝑣
by m = 𝑢
,magnification is different for different points on object. Hence,
magnification is not uniform.
Yes, the distortion of image depends on the location of phone with respect to the mirror.

9
Classroom Questions:
Level – 1
1. An object is placed at (i) 10 cm, (ii) 5 cm in front of a concave mirror of radius of curvature 15 cm. Find the
position, nature, and magnification of the image in each case. Ans. −30cm, −3, 15cm, 3[2 marks]

Level – 2
2. Use the mirror equation to show that (a) an object placed between f and 2f of a concave mirror produces a
real image beyond 2f, (b) a convex mirror always produces a virtual image independent of the location of the
object, (c) an object placed between the pole and focus of a concave mirror produces a virtual and enlarged
image.[Compartment 2011, Outside Delhi 2011] [3 marks]
3. An object is placed in front of a concave mirror of focal length 15cm. The image formed is three times the
size of the object. Calculate the two possible distances of the object from the mirror. Ans. -20cm, -10cm

[DPS] [2 marks]
4. An object is placed at a distance of 36cm from a convex mirror. A plane mirror is placed in between so that
the two virtual images so formed coincide. If the plane distance is at a distance of 24cm from the object, find
the radius of curvature of the convex mirror. Ans. 36cm [2 marks]

10
Home work Sheet 9.1
Theory
1. Define linear magnification for a mirror. Write the expression for magnification (m) of spherical mirrors in terms
u, v and f. [Board 2018] [2 marks]
Derivations
2. Derive the expression for mirror equation. [Board 2018] [DPS] [2 marks]
3. Derive the relation between radius of curvature and focal length. [2 marks]
Diagram
4. Draw a ray diagram to show the image formation by a concave mirror when the object is kept between its focus
and the pole. [Compartment 2011] [1 mark]
5. Draw a ray diagram to show image formation when the concave mirror produces a real, inverted and magnified
image of the object. [Board 2018] [1 mark]
Application Based
6. An object AB is kept in front of a concave mirror as shown in the figure 16.
(i) Complete the ray diagram showing the image formation of the object.
(ii) How will the position and intensity of the image be affected if the lower half
of the mirror’s reflecting surface is painted black?
Fig 16
Ans. (ii) If the lower half of the mirror is painted black, then the intensity of the
image will be reduced (halved) but the position and size of image will remain same.
[Outside Delhi 2012] [2 marks]
7. How is the focal length of a spherical mirror affected when the wavelength of the light used is increased?
[Foreign 2010] [1 mark]
8. Why are convex mirrors used as side view mirrors in cars?
Ans. Convex mirrors form diminished image of object. Hence, a larger area can be viewed using convex
mirrors as side view mirrors. [Delhi 2009, KV] [1 mark]
9. A virtual image, we always say cannot be caught on a screen. Yet, when we see a virtual image we are obviously,
bringing it on the screen of our eye. Is there a contradiction? [1 mark]

Numerical
10. A small candle, 2.5 cm in size is placed at 27 cm in front of a concave mirror of radius of curvature 36 cm. At
what distance from the mirror should a screen be placed in order to obtain a sharp image? Describe the nature
and size of the image. If the candle is moved closer to the mirror, how would the screen have to be moved?
Ans. 54cm, 5cm [2 marks]
11. A 4.5 cm needle is placed 12 cm away from a convex mirror of focal length 15 cm. Give the location of the image
and the magnification. Describe what happens as the needle is moved farther from the mirror.
Ans. 6.67cm, 5/9 [2 marks]

11
(Space for classroom notes)

12
Session 9.2 (Refraction and TIR)
1. Refraction:
It is the phenomenon of bending of light from its straight path when it passes obliquely from one
transparent medium to another.
2. Cause of refraction:
The bending of light or refraction occurs due to the change in the speed of light as it passes from one
medium to another. Larger the change in speed of light as it passes from one medium to another, the more
is the bending due to refraction.
3. Laws of refraction:
i. The incident ray, the refracted ray and the normal to the
interface at the point of incidence, all lie in the same plane.
ii. The ratio of the sine of the angle of incidence to the sine of
angle of refraction is constant for a given pair of media and given
colour of light0.
µ12= sin i
sin r
where µ is called refractive index of the medium 2 w.r.t medium
1
4. Refractive index:
Fig 17
(a) Refractive index in terms of light:
The refractive index of a medium for light of given wavelength may be defined as the ratio of speed of
𝑐
light in vacuum to its speed of light in that medium, i.e. 𝜇 = 𝑣.
If the first medium is vacuum or air, then the refractive index is known as absolute refractive index of the
medium 2.
(b) Refractive index in terms of wavelength:
The refractive index of a medium can be defined as the ratio of wavelength of light in vacuum as
𝜆𝑣𝑎𝑐𝑢𝑢𝑚
compared to the wavelength of light in that medium, i.e. 𝜇 = 𝜆𝑚𝑒𝑑𝑖𝑢𝑚
.
• Factors on which the refractive index of a medium depends :
➢ Nature of the medium.
➢ Wavelength of the light used
➢ Temperature
➢ Nature of the surrounding medium
• Relative Refractive index:
It is given by
𝐬𝐢𝐧 𝒊 𝒗
= 𝝁𝟐𝟏 = 𝒗𝟏,
𝐬𝐢𝐧 𝒓 𝟐
where 𝜇21 is relative refractive index of medium 2 w.r.t. medium 1.
Relative refractive index is a dimensionless quantity.
• A medium having higher value of refractive index is called optically denser medium, while a medium
having lower value of refractive index us called optically rarer medium.
• When light travels from rarer to denser medium, it bends towards normal.
In this case μ21> 1, ∠r <∠i
• When light travels from denser to rarer medium, it bends away from the normal.
In this case μ21<1, ∠r >∠i
• If μ21 is the refractive index of medium 2 w.r.t. medium 1, then μ12 is the refractive index of medium 1
w.r.t. medium 2. Mathematically, μ21 = 1/μ12. (Principle of reversibility of light)
❖ Note: Optical density of a medium is not always proportional to mass density of medium. An optically
rarer medium may have high mass density and an optically denser medium may have less mass density.
In the case of turpentine and water, mass density of turpentine is less than that of water but its optical
density is higher.

µ21 * µ32 = µ31

13
5. Lateral Shift:
It is the perpendicular distance between the incident and emergent
rays when light is incident obliquely in a refracting slab with parallel
faces (fig 18).

6. Normal Shift:
When we view an object immersed in a tank filled with water or any Fig 18
liquid, the object appears to be raised.
The height through which an object appears to be raised in a denser medium is
called normal shift.
As the refractive index of any medium (other than vacuum) is greater than
unity, so the apparent depth is less than the real depth(t).
Real Depth
Refractive Index = Apparent Depth,
t
⟹μ =
(Apparent depth )
t
Hence, Apparent depth = μ
Fig 19
Normal shift =Real depth – Apparent depth.
1
Mathematically, d = t (1 − µ)
Where d is the normal shift
7. Apparent shift in position of sun at sunshine and sunset:
The refraction of light through atmosphere leads apparent shift of the
position of sun at sunrise and sunset. With altitude, the density and
refractive index of the air decreases.
The light rays from the sun thus travel from rarer to denser layers of
atmosphere and hence bends more towards the normal. Fig 20
Thus, actually when the sun is below the horizon, it appears to be above
the horizon and the same happens during sunset which leads to
increase in length of day by 4 minutes (2 minutes before actual sunrise
and 2 min after actual sunset).
❖ Refraction of light through atmosphere is also responsible for
apparent flattening of the sun at sunrise and sunset.
8. Critical Angle:
The angle of incidence in the denser medium for which the angle of
refraction in the rarer medium is 900is called critical angle for pair of
media and is denoted by ic .
9. Total Internal Reflection(Must Do - Examiner Favourite):
The phenomenon in which a ray of light travelling at an angle of
incidence greater than the critical angle from denser to rarer Fig 21
medium is totally reflected back into the denser medium is called total internal reflection (TIR).
sin 𝑖 1
From Snell’s law, sin 𝑟 = 𝜇21 = 𝜇
12
sin 𝑖𝑐 1
When i =ic& r=900, sin 90
=𝜇
12
𝟏
µ12= , (μ12 is the refractive index of denser medium w.r.t. rarer medium).
𝒔𝒊𝒏 𝒊𝒄
Necessary conditions for total internal reflection:
i) Light must travel from an optically denser to optically rarer medium.
ii) The angle of incidence in the denser medium must be greater than the critical angle for the two
media.

14
10. Applications of Total Internal Reflection(Must Do - Examiner Favourite):
i. Mirage:
It is an illusion observed in hot-deserts or over hot extended surfaces like coal-tarred road, due to
which a traveler sees a shimmering pond of water some
distance ahead of him and in which the surrounding
objects like trees, etc. appear inverted.
On a hot summer day, the air near the earth’s surface is
hot and thus has less density and refractive index. Thus,
light travels from denser levels to rarer levels and thus
bend away from normal.
A stage is reached when angle of incidence becomes
Fig 22
greater than critical angle, and thus rays are totally
reflected. These rays move now from low to high density
layers and thus go refraction in opposite direction as that before. These rays when reach the
observer’s eyes form an inverted image, as if formed in a pond.
ii. Sparkling of a diamond:
The refractive index of diamond is very large, hence its
critical angle is very small=24.40. The faces of diamond are
so cut that the light entering the diamond suffers total
internal reflections repeatedly and hence collected inside.
These leads to sparkling of diamond.
iii. Prisms: Fig 23(a)
Prisms designed to bend light by 900 or 1800 make use of
Fig 23(b)
total internal reflection. Such prisms are also used to invert
images without changing their size.
Figures 23(a), 23(b) and 24 represent prisms which deviate rays
by 900(critical angle<450), 1800(critical angle<450) and invert
image without deviation of rays.
iv. Optical Fibres:
An optical fiber is a hair thin long strand of quality glass or
quartz surrounded by a glass coating of slightly lower reflective Fig 24
index.
The thin fiber of optical fiber is called core and the
coating or the surrounding layer of optical fiber is
called cladding.
The refractive index of core is generally 1.458 and that
of cladding is 1.440. Fig 25
It uses the phenomenon of total internal reflection. It
is used as guided medium for transmitting an optical signal (for communication) from one place to
another.
When light is incident on one end of the fiber at small angle, it goes inside and suffers repeated
internal reflections because the angle of incidence is greater than critical angle of fiber material with
respect to its outer coating. This happens even if the fiber is bent. As there is no loss of intensity,
thus outcoming beam is of same intensity as ingoing beam.

15
Solved Examples
Eg 5: A tank is filled with water to a height of 12.5 cm. The apparent depth of a needle lying at the bottom
of the tank is measured by a microscope to be 9.4 cm. What is the refractive index of water? If water is
replaced by a liquid of refractive index 1.63 up to the same height, by what distance would the microscope
have to be moved to focus on the needle again? [2 marks]
Sol: Here, real depth = 12.5cm and apparent depth = 9.4cm.
𝑅𝑒𝑎𝑙 𝐷𝑒𝑝𝑡ℎ
Refractive Index of water = 𝐴𝑝𝑝𝑎𝑟𝑒𝑛𝑡 𝐷𝑒𝑝𝑡ℎ
12.5
⇒ μw = 9.4
⇒ μw = 1.33
Now, in next case, μ = 1.63 and real depth = 12.5cm
𝑅𝑒𝑎𝑙 𝐷𝑒𝑝𝑡ℎ
Hence, Apparent depth = 𝑅𝑒𝑓𝑟𝑎𝑐𝑡𝑖𝑣𝑒 𝐼𝑛𝑑𝑒𝑥 𝑜𝑓 𝑙𝑖𝑞𝑢𝑖𝑑
12.5
⇒ Apparent Depth = 1.63
⇒ Apparent Depth = 7.7cm
Thus, microscope will have to be shifted by distance = 9.4 – 7.7 = 1.7cm
Ans. Refractive index of water is 1.33 and microscope will have to be shifted by 1.7cm

Eg 6: A small bulb (assumed to be a point source) is placed at the bottom of a tank containing water to a
depth of 80cm. Find out the area of the surface of water through which light from the bulb can emerge. Take
the value of refractive index of water to be 4/3. [Compartment 2013, Udgam] [3 marks]
Sol: The light rays from the small bulbs S which are incident at i >𝑖𝑐 are totally internally reflected and cannot
emerge out of water surface.
The light from the bulb S comes out through a circular patch of
radius r given by
𝑂𝐴 𝑟
tan𝑖𝑐 =𝑂𝑆 =ℎ
⟹r=htan𝑖𝑐
Therefore,
𝑠𝑖𝑛2 𝑖 𝜋ℎ 2 𝑠𝑖𝑛𝑖
Area of patch =π𝑟 2 = πℎ2 𝑡𝑎𝑛2 𝑖𝑐 =πℎ2 𝑐𝑜𝑠2 𝑖𝑐 = 1−𝑠𝑖𝑛2 𝑖𝑐
𝑐 𝑐
1 1 3
Now h=80cm= 0.8m and 𝑠𝑖𝑛𝑖𝑐 =µ = 1.33 =4
3.14×(0.80)2 ×(3/4)2 Fig 26
Therefore area of patch= 32
=2.58𝑚2
1−( )
4
Ans. Area of surface of water covered is 2.58m

16
Class Room Questions: -
Level – 1
1. A tank is filled with water to a height of 12.5 cm. The apparent depth of a needle lying at the bottom of
the tank is measured by a microscope to be 9.4 cm. What is the refractive index of water? If water is
replaced by a liquid of refractive index 1.63 up to the same height, by what distance would the
microscope have to be moved to focus on the needle again? [2 marks]
2. Three rays of light- red (R), green (G) and blue (B) - are incident on the
face AB of a right angled prism ABC as shown in figure 27. The refractive
indices of the material of the prism for red, green and blue wavelengths
are 1.39, 1.44 and 1.47 respectively. Trace the path of the rays through
the prism. How will the situation change if these rays were incident
normally on one of the faces of an equilateral prism.
[Delhi 2013, DPS, KV, Udgam] [3 marks]
Fig 27

Level – 2
1. A small bulb (assumed to be a point source) is placed at the bottom of a tank containing water to a
depth of 80cm. Find out the area of the surface of water through which light from the bulb can emerge.
Take the value of refractive index of water to be 4/3. [Compartment 2013, Udgam] [3 marks]

17
Homework Sheet 9.2
Theory
1. What do you mean by refraction of light? State the laws of refraction of light. [2 marks]
2. Define refractive index of a medium in terms of (i) speed, (ii) wavelength of light. What is relative
refractive index? [KV] [2 marks]

3. State the conditions for the phenomenon of total internal reflection to occur.[Delhi 2010] [1 mark]
4. Draw a schematic diagram of a single optical fiber structure. Describe in brief, the mechanism of
propagation of light signal through an optical fiber. [Delhi 2013] [2 marks]
Diagram
5. A right-angled crown glass with critical angle 410is placed
before an object, PQ, in two positions as shown in the figures
28(i) and 28(ii). Trace the paths of the rays from P and Q
passing through the prisms in the two cases.
Fig 28
[Outside Delhi 2013, Udgam] [2 marks]
Application Based
6. For the same value of angle of incidence, the angles of refraction in three media A, B and C are 150, 250
and 350 respectively. In which medium would the velocity of light be minimum?
1 1
Ans For same value of i, μ∝ sin 𝑟 andμ∝ 𝑣. Thus, v ∝ sin r. Hence, for 150, velocity of light is minimum.
[Outside Delhi 2012, DPS, KV] [1 mark]
7. When light travels from a rarer to a denser medium, the speed decreases. Does this decrease in speed
imply a decrease in the energy carried by the light wave? Justify your answer.
Ans. No, energy does not decrease as it depends on frequency and frequency does not change in going
from one medium to another medium. [Outside Delhi 2010] [1 mark]
8. Only the stars near the horizon twinkle while those overhead do not twinkle. Why?
Ans. For stars near the horizon, light has to travel through various layers of air. Hence, it undergoes
multiple refractions, which leads to twinkling. Whereas for stars overhead, light comes straight without
any refraction. Hence, they do not twinkle. [1 mark]
9. An empty test tube dipped into water in a beaker appears silvery, when viewed from a suitable
direction. Why?
Ans. An empty test tube when viewed at an angle greater than critical angle of water, light undergoes
multiple internal reflections and gather at a certain point and hence water appears silvery. [1 mark]
Numerical
10. Calculate the speed of light in a medium whose critical angle is 300. Ans. 1.5×108 m/s
[Delhi 2010] [1 mark]

11. The figure 29 shows a ray of light falling normally on the face AB of an
3
equilateral prism having refractive index 2, placed in water of refractive
4
index 3. Will this ray suffer total internal reflection on striking the surface
AC? Justify your answer. Ans. No [Board 2018] [2 marks]

Fig 29

18
12. Trace the path of a ray of light passing through a glass prism (ABC) as shown in the figure 30. If the
refractive index of glass is √3, find out the value of the angle of emergence from the prism.[Outside
Delhi 2012] Ans. 600 [2 marks

Fig 30

19
Session 9.3 (Refraction Through Spherical Lenses)
1. Spherical lenses:
A lens is a piece of a refracting medium bounded by two surfaces, at least one of which is a curved surface.
Types of Spherical lenses:
(a) Convex lens/Converging lens:
It is thicker at the centre than at the edges. It converges a parallel beam of light on refraction through it.
It has real focus.
Types of Convex lens
Double Convex lens
Plano Convex lens
Concavo Convex lens

Fig 31(a)

(b) Concave lens:


It is thinner at the centre than at the edges. It diverges parallel beam of light on refraction through it. It
has a virtual focus.
Types of Concave lens
Double Concave lens
Plano Concave lens
Convexo concave lens

Fig 31(b)

2. Important terms in the case of spherical lenses:


i. Center of curvature (C): The centre of curvature of the surface of a lens is the centre of the sphere of
which it forms a part. Because a lens has two surfaces, so it has two centers of curvature.
ii. Radius of curvature(R): the radius of curvature of the surface of a lens is the radius of the sphere of
which it forms a part.
iii. Principle axis: it is the line passing through the two centers of curvatures of the lens.
iv. Optical centre: If a ray of light is incident on a lens such that after refraction through the lens the
emergent ray is parallel to the incident ray, then the point at which the refracted ray intersects the
principal axis is called the optical centre of the lens.

20
3. Refraction at a spherical surface(Must Do - Examiner Favourite):
As shown in fig 32, we consider a convex refracting surface which
separates a rarer medium of refractive index n1 from denser
medium of refractive index n2.The object is kept at O and image
is formed at I. For small aperture, NP=NM.
Suppose all the rays are paraxial,
𝑀𝑁
then ∠NOM ≈ tan ∠NOM = 𝑂𝑀 →①
𝑀𝑁
∠NCM ≈ tan ∠NCM = 𝐶𝑀 →②
𝑀𝑁
∠NIM ≈ tan ∠NIM = 𝐼𝑀
→③
Now, for ΔNOC, ‘i’ is the exterior angle.
So, by exterior angle property, i = ∠NOM + ∠NCM
𝑀𝑁 𝑀𝑁
⟹ i = 𝑂𝑀 + 𝐶𝑀 →④ [From ①&②]
Fig 32
Similarly, for ΔNCI, r = ∠NCM - ∠NIM (By exterion angle property)
𝑀𝑁 𝑀𝑁
⟹r= - →⑤ [From ②&③]
𝐶𝑀 𝐼𝑀
sin 𝑖 𝑛
Now, by Snell’s law, sin 𝑟 = 𝑛2
1
⟹ n1 sini = n2sinr
For small angle, n1i = n2r
𝑀𝑁 𝑀𝑁 𝑀𝑁 𝑀𝑁
⟹ n1[𝑂𝑀 + 𝐶𝑀
]= n2[ 𝐶𝑀 − 𝐼𝑀 ] [From ④&⑤]
1 1 1 1
⟹ n1MN[𝑂𝑀 + 𝐶𝑀]= n2 MN[𝐶𝑀 − 𝐼𝑀]
𝑛1 𝑛 𝑛 −𝑛
⟹ + 2 = 2 1 →⑥
𝑂𝑀 𝐼𝑀 𝐶𝑀
Using Cartesian sign conventions, OM = -u, IM = +v, CM = +R
Substituting these in eqn ⑥,
𝒏𝟐 𝒏𝟏 𝒏𝟐 −𝒏𝟏
- = →⑦
𝒗 𝒖 𝑹

21
4. Lens Maker Formula(Must Do - Examiner Favourite):

Assumptions:
(i) The lens is so thin that distances measured from the
its surfaces can be taken as equal to the distances from the
optical centre of the lens.
(ii) The aperture of the lens is small.
(iii) The object is point sized lying on the principle axes of
the lens.
(iv) The incident and refracted ray make small angles with the
Fig 33(a)
principle axis of the lens.
Fig 30(a) shows the geometry of image formation by a double convex
lens. Image formation can also be seen in two steps:
(i) The first refracting surface forms image I1 of object O as
shown in fig 33(b).
𝑛 𝑛 𝑛 −𝑛
Applying eqn 𝑣2 - 𝑢1 = 2 𝑅 1 to first interface ABC,
we get,
𝑛2 𝑛1 𝑛2 −𝑛1
- = →⑧
𝑣1 𝑢 𝑅1
(ii) For Fig 33(c), the image I1 acts as a virtual object for
the second surface ADC that forms image at I. Fig 33(b)
Here, I1 which is the virtual object is
placed in medium of refractive index n2 and I is the
final image formed in medium of refractive index n1.
Here, DI1 is –ve because it is measured against the
direction of light.
𝑛 𝑛 𝑛 −𝑛
Applying eqn 𝑣2 - 𝑢1 = 2 𝑅 1 to interface ADC, we get,
𝑛1 𝑛2
- = 𝑛1𝑅−𝑛2 →⑨
𝑣 𝑣1 2

Adding eqns ⑧&⑨, we get Fig 33(c)


𝑛1 𝑛1 1 1
- = (n2 – n1) [ − ] →⑩
𝑣 𝑢 𝑅1 𝑅2
Suppose the object is at infinity, i.e., u → ∞ , then image is formed at focus (f), i.e., v = f,
𝑛1 1 1
𝑓
= (𝑛2 − 𝑛1 ) [𝑅 − 𝑅 ] →⑪
1 2
1 𝑛2 −𝑛1 1 1
⟹ 𝑓
= 𝑛 [𝑅 − 𝑅 ]
1 1 2
𝟏 𝟏 𝟏
⟹ 𝐟
= (n21 -1)[𝐑 − 𝐑 ]
𝟏 𝟐
This equation is known as lens maker formula.
From eqn ⑩&⑪, we get
𝟏 𝟏 𝟏
- =
𝐯 𝐮 𝐟
This is known as thin lens formula.

5. Sign Convention:
For a lens, radius of curvature is taken to be positive, if it is taken in direction of incident light and it is taken
negative, if it is in opposite direction of incident light.
1 1
(i) For a convex lens, − 𝑅 is positive as R1 is positive and R2 is negative.
𝑅1 2
1 1
(ii) For a concave lens, − 𝑅 is negative as R1 is negative and R2 is positive.
𝑅1 2

22
Fig 34 Fig 35

6. Magnification:
Magnification produced by a lens is defined as ratio of size of image to that of the object (same as that of
𝒉′ 𝒗
mirror). ∴m = 𝒉 = 𝒖
When we apply sign convention, for erect (&virtual) image, formed by a convex or concave lens, m is
positive, while for an inverted (&real) image, m is negative.
7. Power of a lens:
It is defined as the extent of convergence or divergence a lens can produce.
Power of lens is also defined as reciprocal of its focal length.
𝟏
P=𝒇
𝟏𝟎𝟎
SI unit is dioptre (D). 1D = 1m-1. If f is in cm , P = 𝒇
8. Combination of thin lenses in contact:
As shown in figure 28 let L1& L2 be two thin lenses of focal length f1 & f2 respectively placed coaxially in
contact with one another. Let O be a point object on principal axis of lens system.
Let OC1 = u. In absence of 2nd lens L2, first lens L1 will form a real
image I1 of O at distance v1.
∴ Using the lens formula,
1 1 1
𝑓1
=𝑣 -𝑢 →①
1
Image I1 now acts as a virtual object (u=v1) for L2 which finally forms its real image I at distance v. Thus,
1 1 1
𝑓2
=𝑣-𝑣 →②
1
Adding eqns ①&②, we get,
1 1 1 1 1
+ = - =
𝑓1 𝑓2 𝑣 𝑢 𝑓
𝟏 𝟏 𝟏
⟹ = +
𝒇 𝒇𝟏 𝒇𝟐
∴ Equivalent power, P = P1 + P2
Equivalent magnification, m = m1.m2 Fig 36

If one lens is convex and


other is concave then
𝟏 𝟏 𝟏
= -
𝒇 𝒇𝟏 𝒇𝟐

23
Solved Examples
Eg 7: You are given three lenses L1, L2 and L3 each of focal
length 20cm. An object is kept at 40cm in front of L1. The final
real image is formed at the focus ‘I’ of L3. Find the separations
between L1, L2 and L3. [Outside Delhi 2012] [3 marks]
Sol: For Lens 𝐿1 , Fig 37
1 1 1
𝑓
= -
𝑣 𝑢
u = - 40 cm, v = ?, f = + 20 cm
1 1 1
= +
20 𝑣 40
Distance of image from lens 𝐿1
=v = 40 cm
For Lens 𝐿3
1 1 1
= -
𝑓 𝑣 𝑢
Distance of image from lens 𝐿3
v = 20 cm
1 1 1
= +
20 20 𝑢
= u = 
= The refracted rays from lens 𝐿1 becomes parallel to principal axis. It is possible only when image formed
by 𝐿1 lies at first focus of 𝐿2 i.e., at a distance of 20 cm from 𝐿2 .
Ans. Separation between 𝐿1 and 𝐿2 = 40 + 20 = 60 cm.
The distance between 𝐿2 and 𝐿3 may take any value.

Eg 8: A beam of light converges to a point P. A lens is placed in the path of the convergent beam 12 cm
from P. At what point does the beam converge if the lens is (i) a convex lens of 20 cm focal length and (ii) a
concave lens of 16 cm focal length? [Foreign 2013] [3 marks]
Sol: (a) For convex lens, u=+12cm, f=+20cm
1 1 1
Now 𝑢 -𝑣=𝑓
1 1 1
Therefore, 𝑣-12=20
1 1 1 3+5 8
⟹ = + = =
𝑣 20 12 60 60
15
⟹v= = 7.5cm
2
(b) For concave lens, u=+20cm, f=-16cm
1 1 1 1 1 −3+4 1
Now = - = + = =
𝑣 𝑓 𝑢 −16 12 48 48
⟹v=48cm
Ans. (a) Thus the beam converges at a point 7.5 cm to the right of the lens.
(b) Thus the beam converges at the point 48cm to the right of the lens.

Eg 9: A biconvex lens has a focal length 2/3 times the radius of curvature of either surface. Calculate the
refractive index of lens material. [Delhi 2010, DPS] [2 marks]
Sol: Let f be the focal length and R be the radius of curvature.
2
Hence, f = 3R
1 1 1
Using lens maker formula, 𝑓 = (n21 -1)[𝑅 − 𝑅 ], where R1 = R & R2 = -R.
1 2
3 𝑅 3 7
Thus, (n21- 1) = (2𝑅)( 2 ) ⟹ n21 = + 1⟹ n21 = 4
4
7
Ans. Refractive index of material is 4

24
Class Room Questions: -
Level – 1
1. A converging lens has a focal length of 20 cm in air. It is made of material of refractive index 1.6. If it is
immersed in a liquid of refractive index 1.3, what will be its new focal length? How does the nature of
the lens change if this lens is immersed in a liquid of refractive index 1.8?
Ans. 52cm, it will become diverging lens [Delhi 2013, Outside Delhi 2011] [2 marks]
2. A double convex lens made of glass of refractive index 1.6 has its both surfaces of equal radii of
curvature of 30 cm each. An object of 5 cm height is placed at a distance of 12.5 cm from the lens. Find
the position, nature and size of the image. Ans. 25cm , height is 10cm
[Outside Delhi 2013][3 marks]
3. Find the radius of curvature of the convex surface of a plano-convex lens, whose focal length is 0.3m and
the refractive index of the material of the lens is 1.5.
Ans. 15cm [Delhi 2010, KV] [2 marks]

Level – 2
1. You are given three lenses L1, L2 and L3 each of focal length 20cm. An object is kept at 40cm in front of L1.
The final real image is formed at the focus ‘I’ of L3. Find the separations between L1, L2 and L3.
[Outside Delhi 2012] [3 marks]

2 Find the position of the image formed of the object ‘O’ by the lens
combination given in the figure 38. Ans. 30cm [Foreign 2011]
[3 marks]

Fig 38
3 A symmetric biconvex lens of radius of curvature R and made of glass of refractive index 1.5, is placed on
a layer of liquid placed on top of a plane mirror as shown in the figure 39. An optical needle with its tip
on the principle axis of the lens is moved along the axis until its real, inverted image coincides with the
needle itself. The distance of the needle from the lens is measured to be x. On removing the liquid layer
and repeating the experiment, the distance is found to be y. Obtain the expression for the refractive
index of the liquid in terms of x and y. [Board 2018]
[3 marks]
Fig 39

25
Homework Sheet 9.3
Derivations
4 Draw ray diagram showing the geometry of formation of the image of a point object situated on the
principal axis and on the convex side of a spherical surface of radius of curvature R . Taking the rays as
𝑛2 𝑛1
incident from a medium of refractive index n1 to another of refractive index n2, show that − =
𝑣 𝑢
𝑛2 −𝑛1
𝑅
, where the symbols have their usual meaning. [Delhi 2008, Delhi 2011] [3 marks]
𝑛2 𝑛1 𝑛2 −𝑛1
5 Obtain lens makers formula using the expression 𝑣
− 𝑢
= 𝑅
. Here the ray of light propagating from
a rarer medium of refractive index (n1) to a denser medium of refractive index (n2) is incident on the
convex side of spherical refracting surface of radius of curvature R.
[Compartment 2011, Delhi 2008, Foreign 2009] [3 marks]
1 1 1
6 Define power of a lens. Write its units. Deduce the relation 𝑓 = 𝑓 + 𝑓 for two thin lenses kept in contact
1 2
coaxially. [Outside Delhi 2012, KV] [3 marks]
Application Based
7 Under what condition does a biconvex lens of glass having a certain refractive index acts as a plane glass
sheet when immersed in a liquid?
Ans. When refractive index of biconvex lens is equal to the refractive index of liquid.
[Compartment 2012, Delhi 2010] [1 mark]
8 How does the focal length of a lens change when red light incident on it is replaced by violet light? Give
reason for your answer.
1 1
Ans. By lens maker formula, f∝ 𝜇& μ∝ 𝜆. Thus, f ∝ λ. Since, λred> λviolet, hence focal length decreases on
replacing red light by violet light. [Outside Delhi 2012] [1 mark]
9 A converging lens is kept co-axially in contact with a diverging lens – both the lenses being of equal focal
lengths. What is the focal length of the combination? [Outside Delhi 2010] [1 mark]
10 A convex lens made up of glass of refractive index 1.5 is dipped, in turn, in (i) a medium of refractive
index 1.65, (ii) a medium of refractive index 1.33. Will it behave as a converging or a diverging lens in the
two cases? [Outside Delhi 2014, Compartment 2011, Outside Delhi 2011, KV, DPS, Udgam] [2 marks]
Numerical

11 A convex lens has 20cm focal length in air. What is its focal length in water? (Refractive index of
water=4/3, refractive index of glass=1.5). Ans. 80cm [Foreign 2010] [2 marks]

12 An illuminated object and a screen are placed 90cm apart. Determine the focal length and nature of the
lens required to produce a clear image on the screen, twice the size of the object. Ans. 20cm
[Outside Delhi 2010, Udgam] [3 marks]
13 A screen is placed at a distance of 100 cm from an object. The image of the objective is formed on the
screen by a convex lens for two different location of the lens separated by 20 cm. calculate the focal
length of the lens used. Ans. 24cm [Board 2016] [3 marks]

26
(Space for classroom notes)

27
Session 9.4 (Deviation through Prism)
1. Prism:
A prism is a wedge shaped portion of a transparent refracting medium bound by two plane faces inclined to
each other at a certain angle.
The angle A included between the two refracting faces of prism is called angle of prism.
2. Refraction through a prism:
Figure 40 shows the passage of light through a triangular prism ABC. Ray PQ is incident on face AB.
As it enters the denser medium (glass), it bends towards the normal along path QR.
Ray QR suffers another refraction at face AC, bending away from normal, it emerges along path RS.
The angle of deviation (δ) is the angle between the incident ray and the emergent ray.
Let ∠i = angle of incidence, ∠e = angle of emergence, ∠A = angle of prism.
In quadrilateral AQNR, two of the ∠s (∠AQN & ∠ARN) are right angles.
∠A + ∠QNR + ∠AQN + ∠ARN = 3600 (Sum of angles of a quadrilateral)
∴ ∠A + ∠QNR = 180° →①
In ΔQNR, sum of all ∠s of a Δ is 180°
r1+ r2 + ∠QNR = 1800→②
Comparing eqns ①&②, r1 + r2 = A →③
Now, from ΔMQR, the deviation produced by prism is,
δ = ∠MQR + ∠MRQ = (i - r1) + (e-r2) (Exterior angle property)
⟹δ = i + e – (r1+r2)
⟹δ = i + e – A [From ③]
δ+A = i +e →④ Fig 40

Angle of incidence + Angle of emergence = Angle of Prism + Angle of deviation

Thus, the angle of deviation depends on the angle of incidence.


A graph between the angle of deviation and angle of incidence is shown in Fig 41.
Thus, in general, for any given value of δ, except for i = e,
there are two corresponding values of i and hence of e.
This is expected from the symmetry of i and e in Eq. ④,
i.e., δ remains the same if i and e are interchanged.
At the minimum deviation δm, the refracted ray inside prism becomes
parallel to its base. Then, we have,
δ=δm, i = e, ⟹ r1= r2= r.
∴ From eqn ③, r + r = A ⟹2r = A ⟹r = A/2
𝛿𝑚 +𝐴
Similarly from eqn ④, δm = 2i – A or i = 2
From Snell’s law, refractive index of material of prism is, Fig 41
𝛿 +𝐴
sin 𝑖 sin( 𝑚 )
2
μ21 = sin 𝑟 = 𝐴
sin( )
2
𝛿 +𝐴 𝛿𝑚 +𝐴
sin( 𝑚 ) 𝛿𝑚 +𝐴
2 2
For a small angle prism, i.e., a thin prism, δm is also very small & we get, μ21= 𝐴 ≈ 𝐴 = 𝐴
sin( )
2 2
⟹ δm = (μ21 - 1)A
❖ Note: The deviation produced by prism is maximum when the angle of incidence is 900. \
Thus, δmax=90 + e - A.

For minimum deviation

δ=δm , i = e, r1 = r2

28
3. Dispersion by a prism:
The phenomenon of splitting of white light into its component colours on passing through a refracting medium
is called dispersion of light. The pattern of the coloured bands
obtained on the screen is called spectrum.
When a narrow beam of sunlight is incident on a glass prism,
the emergent light when projected onto a screen shows a
coloured band. The component colours in the sequence are:
Violet, Indigo, Blue, Green, Yellow, Orange and Red (acronym
VIBGYOR).
Newton’s explanation for dispersion:
Fig 42
It was observed by Newton that when a prism and another
inverted one are placed side by side and white light is incident on one, then we obtain the same white light on
the screen instead of the coloured spectrum.
This happens because the dispersed light when passes through the inverted prism, the component colours are
recombined to give white light again. This proves that white light consists of different colours which are
dispersed by prism.
4. Cause of dispersion:
We know that each component colour of light has definite wavelength. Red light has the longest wavelength
(nearly 700nm) and violet the shortest (nearly 400nm). Dispersion takes place because the refractive index of
refracting medium is different for different wavelengths.
Since, λred>λviolet, thus µred<µviolet.
For a small angled prism, angle of deviation is given by 𝛿 = (𝜇 − 1)𝐴. Thus, δred<δviolet.
A medium in which light undergoes dispersion is called dispersive media, eg. glass, water whereas in which
dispersion does not occur is known as non-dispersive media, eg. vacuum.
5. Dispersive Power:
The dispersive power of the material of a prism for any two colours is defined as the ratio of the angular
dispersion for those two colours to the mean deviation produced by the prism.
6. Scattering of light:
The phenomenon in which light is deflected from its path due to its interaction with the particles of the medium
through which it passes is known as scattering of light.
Rayleigh’s Criteria:
According to Rayleigh’s law of scattering , the intensity o flight of wavelength λ present in the scattering of light
is inversely proportional to the fourth power of λ, provided the size of scattering particles are much smaller
than λ. Mathematically, I α 1/λ4 , a << λ
7. Phenomenon based on scattering:
i. Blue colour of the sky:
As sunlight passes through atmosphere, nitrogen and
oxygen molecules absorb some amount of sunlight and re-
emit it. The free gas molecules scatter the light in all
directions. According to Rayleigh’s law of scattering, the
light at shorter wavelength (blue) end is scattered 10 times
more than that of red light. Thus, sky appears blue.
ii. Reddishness at sunrise and sunset:
When the sun is near horizon, the light rays have to Fig 43
traverse a large thickness of atmosphere, thus the blue light is scattered away totally and the least
scattered red light reaches our eyes.
iii. Also, clouds appear white and danger signals are of red colour are due to scattering of light.

29
Classroom Questions:
Level-1
1. A ray of light, incident on an equilateral glass prism (µg=√3) moves parallel to the base line of the prism
inside it. Find the angle of incidence for this ray. Ans. 600
[Compartment 2012, Delhi 2012,Outside Delhi 2012, Udgam] [2 marks]

Level-2
1. A ray PQ incident on the refracting face BA is refracted in the prism BAC as shown in the figure 44 and
emerges from the other refracting face AC as RS such that AQ=AR. If the angle of prism A=60 and
refractive index of material of prism is 3, calculate angle θ. [Board 2016] [3 marks]

Fig 44

30
Homework Sheet 9.4
Theory
2. Define angle of prism.
Ans. It is the angle between two refracting surfaces of a prism. [1 mark]
3. What are the conditions for minimum deviation through prism?
Ans. Refracted ray is parallel to base of prism. Also, angle of emergence = Angle of incidence[1 mark]
4. Plot graph for angle of deviation (δ) versus angle of incidence (i ) for a triangular prism. [1 mark]
5. What do you mean by dispersion? Explain dispersion through triangular prism. [2 marks]
6. What do mean by scattering of light. Explain one phenomenon based on scattering of light. [2 marks]
Derivations
7. Draw a ray diagram to show refraction of a ray of monochromatic light passing through a glass prism.
Deduce the expression for the refractive index of glass in terms of angle of prism and angle of minimum
deviation. [Compartment 2011, Foreign 2011] [3 marks]
Application Based
8. Why can’t we see clearly through fog? Name the phenomenon responsible for it. [Board 2016]
[1 mark]
9. (a) The bluish colour predominates in clear sky. (b) Violet colour is seen at the bottom of the spectrum
when white light is dispersed by a prism. State reason to explain these observations.
[Delhi 2010] [2 marks]
10. (i) Why does the Sun appear reddish at sunset or sunrise? (ii) For which colour the refractive index of
prism material is maximum and minimum? [Delhi 2010, DPS] [2 marks]
11. (i) Out of blue and red light which is deviated more by a prism? Give reason. (ii) Give the formula that
can be used to determine refractive index of material of a prism in minimum deviation condition.
[Delhi 2010, KV] [2 marks]
12. Why red colour is used in danger sign board?
1
Ans. Red colour has largest wavelength. By Rayleigh’s criteria, deviation ∝ 𝜆4 . Hence, deviation is least
for red colour. Thus, it is used in danger sign board. [1 mark]
Numerical
13. A ray of light strikes one face of the prisms at an angle of incidence 600 and angle of refraction is 300. If
the angle of prism is 600, find the angle of emergence. Ans. 600 [2 marks]

31
(Space for classroom notes)

32
Session 9.5 (Microscope and Telescope)
1. Simple microscope:
𝐃
Magnifying Power / Angular Magnification(|𝐮|):
It is defined as the ratio of the angle formed by the image at the eye to the angle formed by the object at the
eye, when situated at the least distance of distinct vision.
(i) When the image is formed at least distance of distinct vision:
Magnifying power:
Let f be the focal length of the lens and object be placed at a distance
‘u’ from the lens. Also, v = -D.
1 1 1
Using thin lens formula, v − u = f , we get,
1 1 1
⟹ −D − −u = f
1 1 1 D
⟹u=D+ f ⟹ u
=
D Fig 45
1+ f
𝐃
⟹ 𝐦 = 𝟏 + 𝐟.
Thus, shorter is the focal length; greater is its magnifying power.
(ii) When the final image is formed at infinity:
Angular Magnification:
Suppose the height of the object is h. The maximum angle it can
subtend, and be clearly visible (without a lens), is when it is at the
near point, i.e., at distance D. The angle subtended is then given by, Fig 46

tan θ0= 𝐷 ≈θ0 (Fig 46)
We now find the angle subtended at the eye by the image when the
object is at u.
ℎ′ 𝑣
From m = ℎ = 𝑢, we have the angle subtended by the image,
ℎ′ ℎ 𝑣 𝑣
tan θi = −𝑣 = −𝑣 . 𝑢 [h’ = h.𝑢]

= −𝑢 ≈θ

The angle subtended by object when it is at u = -f, θi = 𝑓 [Fig 47]
Fig 47
The angular magnification is, therefore,
𝜽𝒊 𝑫
⟹m = =𝒇
𝜽𝟎
This magnification is one less than the magnification when the image is at near point, but the viewing is more
comfortable and the difference in magnification is usually small.
A simple microscope has a limited maximum magnification (≤ 9) for realistic focal lengths. For larger
magnification, two lenses are used, one compounding the effect of other. This is known as compound
microscope.

33
2. Compound microscope:
Construction:
It consists of two convex lenses of short focal length, arranged co-axially at the ends of two sliding metal tubes.
(i) Objective is a convex lens of very short focal length.
(ii) Eyepiece is a convex lens of comparatively larger focal length.
(a) When the final image is formed at the least distance of
distinct vision:-
The object is placed at a distance slightly larger than
the focal length from the objective . The objective
forms a real, inverted and magnified image of the
object on the other side of the objective lens.
The separation between eyepiece and objective is so
adjusted that the image lies within the focal length of
the eyepiece. Fig 48
The image so formed, now acts as an object for the
eyepiece.
This inverted image is near the focal plane of the
eyepiece, at a distance appropriate for final image formation at near point.
The eyepiece forms a virtual and magnified image of the object which is inverted w.r.t. the original object.
Magnifying power:
For a compound microscope,
m = mome
where mo is magnification due to objective
and me is magnification due to eyepiece.
ℎ′ 𝑣𝑜 𝐿 ℎ ℎ′
Linear magnification due to the objective, m0 = = = , [tan β = = ]
ℎ 𝑢𝑜 𝑓0 𝑓0 𝐿
Here h’ is the size of the first image, the object size is h and f0 is the focal length of the objective.
The distance L is called the tube length of the compound microscope. It is the distance between the two
lenses.
𝐷
As the eyepiece acts as a simple microscope, so me = 1 + 𝑓
𝑒
𝒗𝒐 𝑫 𝑳 𝑫
∴m = m0me = 𝒖𝒐
(𝟏 + 𝒇𝒆
) = 𝒇 (𝟏 + 𝒇𝒆
)
𝟎
In this case Length of compound microscope = |vo| + |ue|
(b) When final image is at infinity:
When the image formed by objective is formed at focus of
eyepiece, then the final image is formed at infinity.
𝐷
For this, me= 𝑓 ∴
𝑒
𝐋 𝐃
𝐦=𝐟 ×𝐟
𝟎 𝐞
In this case L = fo + fe
Fig 49

34
3. Telescope:
It is an optical device which enables us to see distant objects clearly. It provides angular magnification of the
distant objects.
Different types of telescopes are as follows:
(i) Refracting telescopes: They make use of lenses to view distant objects.
(ii) Reflecting telescopes: They make use of converging mirrors to view distant objects

4. Refracting telescopes:
Construction: In this the objective is of large focal
length, whereas the eyepiece is of small focal
length. Both are made up of convex lens.
Magnifying power (m):
Magnifying power is the ratio of the angle β
subtended at the eye by the final image to the
angle α which the object subtends at the lens or Fig 50
the eye.
a) When the final image is formed at infinity:
𝛃 𝐡 𝐟𝟎 𝐟
m≈𝛂≈𝐟 . 𝐡
= 𝐟𝟎
𝐞 𝐞
In this case, the length of the telescope tube is, L = f0 + fe
When the image is formed at infinity the telescope is said to be in normal adjustment.
b) When the final image is formed at near point:
𝒇𝒐 𝒇𝒆
𝒎= 𝒇𝒆
(𝟏 + 𝑫
)
The resolving power, or the ability to observe two objects distinctly which are in very nearly the same
direction, also depends on the diameter of the objective. So, the desirable aim in optical telescopes is to
make them with objective of large diameter.
Limitations of refracting type telescopes:
• The refracting type telescope suffers from chromatic and spherical aberrations.
• Objective lenses of large aperture are very difficult to manufacture and handle.
5. Reflecting telescopes:
Telescopes with mirror objectives, i.e. , working on principle of reflection are called reflecting telescopes.
Advantages of a reflecting type telescope:
a) A concave mirror of large aperture has high gathering power and absorbs very less amount of light than
the lenses of large aperture. The final image formed in reflecting type telescope is very bright. So very
distant objects or faint stars can be seen clearly.
b) Due to large aperture of mirror used the resolving power of reflecting telescope is very high.
c) As the objective is a mirror and not a lens, it is free from chromatic aberration.
d) A mirror requires grinding and polishing of one surface only. So, cost is less for construction.
e) A lens of large aperture tends to be very heavy and thus difficult to support and make.
Cassegrain Telescope:
One such arrangement using a convex secondary mirror to
focus the incident light, which now passes through a hole in
the objective primary mirror, is shown in fig 51. This is known
as a Cassegrain telescope. It has the advantages of a large
focal length in a short telescope.

Fig 51

35
Solved Examples
Eg 10: A compound microscope uses an objective lens of focal length 4 cm and eyepiece lens of focal length
10 cm. An object is placed at 6 cm from the objective lens. Calculate the magnifying power of the compound
microscope. Also calculate the length of the microscope.
Sol: For compound microscope, 𝑓𝑜 = 4cm, 𝑓𝑒 =10cm 𝑢𝑜 = -6cm, 𝑣𝑒 = −𝐷 = −25𝑐𝑚
For objective lens,
1 1 1
𝑓
=𝑣 -𝑢
𝑜 𝑜 𝑜
1 1 1
=
4 𝑣𝑜
+ (6)
1 1 1 1
= - =
𝑣𝑜 4 6 12
vo = 12 cm
therefore magnifying power(m)
𝑣 𝐷
= -(𝑢𝑜 )(1+𝑓 )
𝑜 𝑒
12 25
=-( 6 )(1+10)
7
= -2(2) = -7
Magnifying power(m) = -7
Length of microscope=I Vo I + I Ue I
Where vo = 12cm
For eye lens,
ve = -25cm, fe= 10cm, ue= ?
1 1 1
= -
𝑓𝑒 𝑣𝑒 𝑢𝑒
1 1 1
𝑢𝑒
=𝑣 -𝑓
𝑒 𝑒
1 1
= −25 - 10
1 −2−5 −7
𝑢𝑒
= 50 = 50
50
⟹ue = - 7 cm
⟹ue = -7.14cm
Therefore length of microscope= IvoI + IuoI = 12 + 7.14=19.14cm
Ans. Magnifying power is -7 and length of microscope is 19.14cm.

Eg 11: A giant reflecting telescope at an observatory has an objective lens of focal length 15m. If an
eyepiece lens of focal length 1.0cm is used, find the angular magnification of the telescope.
If this telescope is used to view the moon, what is the diameter, of the image of the moon formed by the
objective lens? The diameter of the moon is 3.42 × 106 m and radius of the lunar orbit is 3.8 × 108 m.
Sol:For astronomical telescope,
fo = 15m =1500cm
fe = 1cm
𝑓𝑜
Angular magnification(m) =
𝑓𝑒
15 ×100 𝑐𝑚
=-
1𝑐𝑚
m= - 1500.
Therefore angle subtended by moon at objective,
𝐷
α = 𝑅𝑎𝑑𝑖𝑢𝑠 𝑜𝑓 𝑙𝑢𝑛𝑎𝑟 𝑜𝑟𝑏𝑖𝑡
3.42 ×106 𝑚
⟹α = 3.8 × 108 𝑚
…..(a)
Also, then the angle subtended by image formed by objective on itself.
𝑑
α=𝑓 …..(b)
𝑜
where, d=diameter of image.

From eqs (a) and (b), we get

36
3.42 × 106 𝐷
= (fo = 15m = 1500cm)
3.8 × 108 1500
1500 ×3.42 106
⟹D = =13.5cm.
3.8 × 106
Ans. Diameter of the image of the moon formed is 13.5cm.

Eg 12: Two convex lenses of focal length 20cm and 1cm constitute a telescope. The telescope is focused on
a point which is 1m away from the objective. Calculate the magnification produced and the length of the
tube, if the final image is formed at a distance 25 cm from the eyepiece.
Sol: Given, fo = 20cm, fe = 1cm, ve = -25cm
For objective
uo = -100cm, f0 = 20cm
1 1 1
𝑓𝑜
=𝑣 − 𝑢𝑜

𝑜
1 1 1
= − ⟹
20 𝑣𝑜 (−100)
1 1 1
= −
𝑣𝑜 20 100
5−1 4
= 100 = 100
⟹vo = 25 cm
For eye lens
fe = 1cm
ue = ?
ve = -25
1 1 1
𝑓𝑒
=𝑣𝑒
− 𝑢 ⟹
𝑒
1 1 1
= −
1 −25 𝑢𝑒
1 1
⟹1+ 25 = - 𝑢
𝑒
26 1
⟹ =-
25 𝑢𝑒
25
⟹ue = - 26
⟹IueI = 0.96 cm
Magnification(m)
𝑣𝑜 25
⟹m= - (1+ )
𝑢𝑜 1
25 25
= - ( )(1 + )
100 1
1
⟹m = -4 × 26
⟹m= - 6.5
Length of telescope L = (vo) + (ue)
= 25 + 0.96
L = 25.96 cm
Ans. Magnification produced is -6.5 and length of telescope is 25.96cm.
Eg 13: The magnifying power of a telescope in its normal adjustment is 20. If the length of the telescope is
105 cm in this adjustment, find the focal lengths of the two lenses.
Sol:In normal adjustment,
𝑓
M = | 𝑓𝑜 | = 20
𝑒
fo = 20 fe
Also length of telescope
fo + fe = 105
20fe + fe =105
21fe =105
fe = 5cm
fo =20fe = 20 × 5 = 100cm.
Ans. Focal lengths are 5cm and 100cm.

37
Eg 14: A small telescope has an objective lens of focal length 150 cm and an eyepiece of focal length 5 cm.
If this telescope is used to view a 100m high tower 3 km away, find the height of the final image when it is
formed 25 cm away from the eyepiece.
Sol: Focal length of objective lens fo = 150cm = 1.5m
Focal length of eye lens fe = 5cm
For objective lens, u0 = -3km = -3000m and h = 100m.
1 1 1
Using Lens formula, - = , we get,
𝑣 𝑢 𝑓
1 1 1
⟹ = 1.5 - 3000
𝑣
12000−1 1999
⟹ = 3000 = 3000
𝑣
3000 3
⟹v= ≈ m
1999 2
ℎ′ 𝑣
We know that, m = ℎ =𝑢
ℎ′ 3/2
⟹ =
100 3000
1
⟹ h’ = 20m
Now, for eyepiece lens, ve = -25cm.
1 1 1
Using Lens formula, 𝑣 - 𝑢 = 𝑓, we get,
1 −1 1
⟹ 𝑢 = 25 - 5
1 −1−5 −6
⟹𝑢= 25
= 25
−25
⟹u= 6
cm
ℎ” 𝑣
We know that, m = = . Here, image by objective lens acts as virtual object for eyepiece lens.
ℎ′ 𝑢
ℎ" −25
Hence, 1 =
( )𝑚 −25/6
20
6
⟹ h” = m
20
⟹ h” = 30cm
Ans. Final Height of image is 30cm.

Eg 15: The focal lengths of the objective and eyepiece of a microscope are 1.25cm and 5cm respectively.
Find the position of the object relative to the objective in order to obtain an angular magnification of 30 in
normal adjustment.
Sol: For microscope:
fo = 1.25cm, fe = 5cm
when final image forms at infinity then magnification produced by eye lens is given by
𝐿 𝐷
m=- .
𝑓𝑜 𝑓𝑒
𝐿 25
-30 = - 1.25 . 5
30 ×1.25
L=
5
L =7.50cm
For objective lens
vo = L = 7.5cm
fo = 1.25cm
uo = ?
Applying lens formula
1 1 1 1 1 1
𝑓
= 𝑣 − 𝑢 1.25
= 7.5
− 𝑢
𝑜 𝑜 𝑜 𝑜
1 1 1
𝑢𝑜
= 7.5 − 1.25
1.25−7.5
=
7.5 ×1.25
6.25
= - 7.5 × 1.25
7.5 ×1.25
uo = - 6.25 = 1.5cm
Ans. The object must be at a distance of 1.5cm from objective lens.

38
Eg 16:A compound microscope consists of an objective lens of focal length 2.0cm and a eyepiece of focal
length 6.25cm separated by a distance of 15cm. How far from the objective should an object be placed in
order to obtain the final image at (i) the least distance of distinct vision (25cm), (ii) infinity?
What is the magnifying power of the microscope in each case?
Sol: Here fo = 2.0cm, fe = 6.25cm, uo =?
(i)When the final image is obtained at the least distance of distinct vision:
1 1 1
As − =
𝑣𝑒 𝑢𝑒 𝑓𝑒
1 1 1 1 1
𝑢𝑒
= 𝑣 − 𝑓𝑒
= −25 − 6.25
𝑒
−1−4 −5 1
= = = −
25 25 5
ue = -5cm
Now distance between objective and eyepiece =15cm
∴ Distance of the image from objective is
vo = 15-5 = 10cm
1 1 1 1 1 1−5 2
𝑢
= 𝑣 − 𝑓 = 10 − 2 = 10 = − 5
𝑜 𝑜 𝑜
5
uo = -2 = -2.5cm
∴ Distance of object from objective = 2.5cm
Magnifying power,
𝑣 𝐷
m = mo× 𝑚𝑒 = 𝑢𝑜 (1 + 𝑓 )
𝑜 𝑒
10 25
= 2.5 (1 + 6.25) = 20

(ii)When the final image is formed at infinity:


Here ve = ∞ fe =6.25cm
1 1 1 1 1 1
As − = ∴ − =
𝑣𝑒 𝑢𝑒 𝑓𝑒 ∞ 𝑢𝑒 𝑓𝑒
Or Distance between objective and eyepiece = 15cm
Therefore distance of the objective from the image formed by itself,
vo = 15-6.25 = 8.75cm
Also fo = +2.0cm
1 1 1 1 1 2−8.75 −6.75
Therefore 𝑢 = 𝑣 − 𝑓 = 8.75 − 2 = 17.5 = 17.5
𝑜 𝑜 𝑜
17.5
uo = - 6.75 =
-2.59cm
∴ The distance of the object from objective = 2.59cm.
Magnifying power,
𝑣 25
m=mo × me = 𝑢𝑜 × 6.25
𝑜
27
= 8
×4 = 13.46 = 13.5

Eg 17: A person with a normal near point (25cm) using a compound microscope with an objective of focal
length 8.0mm and eyepiece of focal length 2.5cm can bring an object placed 9.0 mm from the objective in
sharp focus. What is the separation between the two lenses? How much is the magnifying power of the
microscope?
Sol: Here fo= 0.8cm, uo = -9.0cm, vo = ?
1 1 1 1 1
As 𝑣 = 𝑓 + 𝑢 = 0.8 − 0.9
𝑜 𝑜 𝑜
0.9−0.8 0.1
= 0.9 ×0.8 = 0.8 ×0.9
0.8 × 0.9
vo = 0.1 = 7.2cm
Now for the eyepiece, we have
fe = 2.5cm, ve = -D 25cm, ue =? ∴
1 1 1 1 1 −1−10 −11
𝑢
= 𝑣 − 𝑓 = − 25 − 2.5 = 25 = 25
𝑒 𝑒 𝑒
25
Or ue = - = 2.27cm
11
Hence the separation between the two lenses

39
= |vo| + | ue | = 7.2 +2.27 = 9.47cm
Magnifying power,
𝑣 𝐷
m = mo× me = |𝑢𝑜 | (1 + )
𝑜 𝑓𝑒
7.2 25
= 0.9
(1 + 2.5) = 88

Eg 18: A small telescope has an objective lens of focal length 140cm and an eyepiece of focal length 5.0cm.
What is he magnifying power of the telescope for viewing distant objects when
(a)the telescope is in normal adjustment(i.e., when the final image is at infinity),
(b)the final image is formed at the least distance of distinct vision (25cm)?
Sol: Here fo = 140cm, fe = 5.0cm
(a)In normal adjustment:
Magnifying power,
𝑓 140
m = 𝑓𝑜 = 5 = 28
𝑒
(b)When the final image is formed at the least distance of distinct vision (25cm):
𝑓𝑜 𝑓
m= 𝑓𝑒
(1 + 𝐷𝑒 )
140 5
= 5 (1 + 25)= 28 × 1.2 = 33.6.

40
Classroom Questions:
Level-1
1. The magnifying power of a telescope in its normal adjustment is 20. If the length of the telescope is 105
cm in this adjustment, find the focal lengths of the two lenses.
2. A small telescope has an objective lens of focal length 150cm and an eye piece of focal length 5cm. If this
telescope is used to view a 100m high tower 3km away, find the height of the final image when it is
formed 25cm away from the eye piece. Ans. 30cm [Compartment 2012, Outside Delhi 2012]
[3 marks]
2
3. A figure divided into squares, each of size 1 mm , is being viewed at a distance of 9 cm through a
magnifying lens of focal length 10 cm, held close to the eye.(a) Draw a ray diagram showing the
formation of the image.(b)What is the magnification produced by the lens? How much is the area of
each square in the virtual image? (c) What is the angular magnification of the lens?
Ans. 10, 100mm2, 2.8 [Delhi 2013] [3 marks]
4. In a compound microscope, the focal lengths of the objective and eye-piece are 10 cm and 20 cm,
respectively. The tube length of the microscope is 20 cm. Calculate its magnification. Why must both the
objective and the eye-piece of a compound microscope have short focal lengths?
Ans.2.5 [Foreign 2013, DPS] [3 marks]

Level-2
1. (a) A giant refracting telescope at an observatory has an objective lens of focal length 15 m. If an
eyepiece lens of focal length 1.0 cm is used, find the angular magnification of the telescope. (b)If this
telescope is used to view the moon, what is the diameter of the image of the moon formed by the
objective lens? The diameter of the moon is 3.42 x 106 m and the radius of the lunar orbit is 3.8 x 108 m.
Ans. 1500, 13.5cm [Compartment 2011, Outside Delhi 2011, DPS] [3 marks]

41
Homework Sheet 9.5
Theory
1. Draw a labelled ray diagram to show image formation by a refracting telescope. Write expression for its
magnifying power. [Foreign 2013] [2 marks]
2. Draw a schematic arrangement of a reflecting telescope (Cassegrain) showing how rays coming from a
distant object are received at the eye-piece. Write its two important advantages of reflecting telescope
over a refracting telescope.[Compartment 2013, Outside Delhi 2012,KV, Udgam, DPS, Board 2018]
[2 marks]
3. Draw a labeled ray diagram to show the formation of an image by a compound microscope. Write the
expression for its magnifying power. [Outside Delhi 2013, DPS] [2 marks]
Derivations
4. Draw a ray diagram to show the working of a compound microscope. Deduce an expression for the total
magnification when the final image is formed at the near point. [Delhi 2010, Board 2015] [3 marks]
Application Based
5. Why must both the objective and the eye-piece of a compound microscope have short focal lengths?
1
Ans. We know that, m∝ 𝑓 𝑓 . Hence, to increase magnification, focal length of eye-piece and objective
0 𝑒
must be small. [Outside Delhi 2010] [1 mark]
6. When viewing through a compound microscope, our eyes should be positioned not on the eyepiece but
a short distance away from it for best viewing. Why?
Ans. When we place our eyes too close to the eyepiece of a compound microscope, we are unable to
collect much refracted light. As a result, the field of view decreases substantially. Hence, the clarity of
the image gets blurred. The best position of the eye for viewing through a compound microscope is at
the eye-ring attached to the eyepiece. The precise location of the eye depends on the separation
between the objective lens and the eyepiece. [Udgam] [1 mark]
Numerical
7. How is the working of a telescope different from that of a microscope? The focal lengths of the objective
and eyepiece of a microscope are 1.25cm and 5cm respectively. Find the position of the object relative
to the objective in order to obtain an angular magnification of 30 in normal adjustment.
Ans. 1.5cm [Compartment 2012, Outside Delhi 2012, Udgam] [3 marks]
8. A compound microscope uses an objective lens of focal length 4 cm and eyepiece lens of focal length 10
cm. An object is placed at 6 cm from the objective lens. Calculate the magnifying power of the
compound microscope. Also calculate the length of the microscope. Ans.7, 19.14cm
[Compartment 2011, Outside Delhi 2011, KV] [3 marks]
9. In a compound microscope, an object is placed at a distance of 1.5 cm from the objective focal length
1.25cm. If the eye piece has a focal length of 5cm and the final image is formed at the near point,
estimate the magnifying power of the microscope. Ans. 30 [Delhi 2010] [2 marks]
10. A compound microscope consists of an objective lens of focal length 2.0cm and an eyepiece of focal
length 6.25cm separated by a distance of 15cm. How far from the objective should an object be placed
in order to obtain the final image at (a) the least distance of distinct vision (25cm), and (b) at infinity?
What is the magnifying power of the microscope in each case. Ans. -2.5cm, 20, -2.59cm, 13.5
[3 marks]
11. A small telescope has an objective lens of focal length 144cm and an eyepiece of focal length 6.0cm.
What is the magnifying power of the telescope? What is the separation between the objective and the
eyepiece? Ans. 24,150cm. [3 marks]

42
(Space for classroom notes)

43
Solved Examples
Eg 19: A convex lens of focal length 10cm is placed coaxially 5cm away from a concave lens of focal length
10cm. If an object is placed 30cm in front of the convex lens, find the position of the final image formed by
the combined system.
Sol: u = -25cm, v= -50cm ∴
1 1 1
𝑓
= 𝑣− 𝑢
1 1
= (−50) − (−25)
−1+2 1
= 50
= 50
1
∴ f = 50cm = 2 cm
1
∴ Power of the lens P =𝑓 = +2D

Eg 20: A Cassegrain telescope uses uses two mirrors. Such a telescope is built with the mirrors 20 mm
apart. If the radius of curvature of the large mirror is 220 mm and the small mirror is 140mm, where will the
final image of an object at infinity be?
Sol: The image formed by the larger (concave) mirror acts as a virtual object for the smaller (convex) mirror.
Parallel rays coming from the object at infinity will focus at 110mm from the larger mirror. The distance of
the virtual object for the smaller mirror =110-20 = 90mm.
For the small convex mirror, we have
u = -90mm, f = -70mm, v=?
1
Using mirror formula, =
𝑣
1 1 1 1 1
− = − = −
𝑓 𝑢 −70 −90 315
v = -315 mm
∴ Thus the image is formed at 315 mm from the smaller mirror.

Eg 21: A beam of light of wavelength 400nm is incident normally on a right angle


prism as shown in fig 52. It is observed that the light just grazes along the surface
AC after falling on it. Given that the refractive index of the material of the prism
𝑏
varies with the wavelength λ as per the relation, μ = 1.2 + 𝜆2 . Calculate the value
of b and the refractive index of the prism material for a wavelength λ = 5000𝐴̇.
Given θ = sin-1(0.625). [Nirman] [3 marks] Fig 52
Sol: As the light just grazes along the surface AC after falling on it, so angle of incidence is
equal to the critical angle.
⟹iC = θ = sin-1(0.625)
⟹sin ic= 0.625
𝟏
⟹μ =𝑠𝑖𝑛 𝑖 = 1.6
𝑐
Hence, for light of wavelength 400nm,
𝑏
μ = 1.2 + 𝜆2
𝑏
1.6 = 1.2 + (400×10−9 )2
⟹b=6.4×10-14m2
For wavelength 5000𝐴̇, the refractive index of the material is,
6.4×10−14
μ = 1.2 + (5000×10−10 )2 = 1.456
Ans. Refractive index is 1.456

44
Assignment Sheet
1. Define pole, radius of curvature, principle axis, principle focus, focal
length of a spherical mirror.
[2 marks]
2. A mobile phone lies along the principal axis of a concave mirror, as
shown in Fig 53. Show by suitable diagram, the formation of its image.
Explain why the magnification is not uniform. Will the distortion of
image depend on the location of the phone with respect to the mirror?
[Outside Delhi 2013] Fig 53
[3 marks]
3. Suppose while sitting in a parked car, you notice a jogger approaching towards you in the side view
mirror of R = 2 m. If the jogger is running at a speed of 5 m s-1, how fast the image of the jogger appear to
move when the jogger is (a) 39 m, (b) 29 m, (c) 19 m, and (d) 9 m away.
Ans. 1/280ms-1,1/150ms-1, 1/60ms-1, 1/10ms-1 [3 marks]
4. Two monochromatic rays of light are incident normally on the face AB of an
isosceles right-angled triangle prism as shown in figure 54. The refractive
indices for the two rays ‘1’ and ‘2’ are respectively 1.35 and 1.45. Trace the
path of these rays after entering through prism.
[Outside Delhi 2014] [2 marks]
5. What is normal shift? [1 mark]
6. State the factors on which refractive index of medium depends.
Fig 54
[1 mark]
7. A luminescent object is placed at a depth d in a optically denser medium of refractive index µ. Prove that
𝑑
radius r of the base of cone of light, from the object that can emerge out from the surface, is, r= .
√𝜇2 −1
[3 marks]
8. The apparent depth of an object at the bottom of a tank filled with a liquid of refractive index 1.3 is
7.7cm. What is the actual depth of the liquid in the tank? Ans. 10.01cm [2 marks]
9. Define one diopter. Ans. It is power of a lens of focal length 1m. [1 mark]
10. The image of a candle is formed by a convex lens on a screen. The lower half of the lens is painted black
to make it completely opaque. How will this image be different from the one obtained when the lens is
not painted black?
Ans. The image formed will be of half intensity than that of the original image but it will be formed at
same point as before. [Delhi 2013, KV][1 mark]
11. The image obtained with a convex lens is erect and its length is four times the length of the object. If the
focal length of the lens is 20cm, calculate the object and image distances. Ans. 15cm, 60cm
[Outside Delhi 2010, KV] [3 marks]
12. The radius of curvature of the faces of a double convex lens are 10 cm and 15 cm. If focal length of the
lens is 12cm, find the refractive index of the material of the lens. Ans. 3/2 [Delhi 2010] [2 marks]
13. Light from a point source in air falls on a spherical glass surface (n = 1.5 and radius of curvature = 20 cm).
The distance of the light source from the glass surface is 100 cm. At what position the image is formed?
Ans. 100cm [2 marks]
14. An object is placed at a distance of 1.5m from a screen and a convex lens in interposed between them.
The magnification produced is 4. What is the focal length of the lens? Ans. 0.24m [2 marks]
15. At what distance should an object be placed from convex lens of power 4D to obtain a real image three
times the size of the object? Ans. 33.33cm [2 marks]
16. A person with a normal near point (25cm) using a compound microscope with objective of focal length
8.0 mm and an eyepiece of focal length 2.5cm can bring an object placed at 9.0mm from the objective in
sharp focus. What is the separation between the two lenses? Calculate the magnifying power of the
microscope. Ans. 9.47cm,88 [3 marks]
17. Two lens of powers 1.25D & -3D are combined to form a lens. Calculate the equivalent focal length of the
combination. [Nirma] Ans. -57.14cm [2 marks]

45
18. Figure 55 shows an equiconvex lens (of refractive index 1.50) in
contact with a liquid layer on top of a plane mirror. A small
needle with its tip on the principal axis is moved along the axis
until its inverted image is found at the position of the needle.
The distance of the needle from the lens is measured to be Fig 55
45.0cm. The liquid is removed and the experiment is repeated.
The new distance is measured to be 30.0cm. What is the
refractive index of the liquid? [3 marks]

19. Figures 56(a) and (b) show refraction of a ray in air incident at 60° with the normal to a glass-air and
water-air interface, respectively. Predict the angle of refraction in glass when the angle of incidence in
water is 45° with the normal to a water-glass interface [Fig. 56(c)] [Textbook]

Fig 56

20. Double-convex lenses are to be manufactured from a glass of refractive index 1.55, with both faces of the
same radius of curvature. What is the radius of curvature required if the focal length is to be 20cm?
[Textbook]
21. (a) Figure 57 shows a cross-section of a ‘light pipe’ made of a glass fibre of refractive index 1.68. The
outer covering of the pipe is made of a material of refractive index 1.44. What is the range of the angles
of the incident rays with the axis of the pipe for which total reflections inside the pipe take place, as
shown in the figure. (b) What is the answer if there is no outer covering of the pipe?
[Textbook]

Fig 57
22. Answer the following questions: (a) You have learnt that plane and convex mirrors produce virtual images
of objects. Can they produce real images under some circumstances? Explain. (b) A virtual image, we
always say, cannot be caught on a screen. Yet when we ‘see’ a virtual image, we are obviously bringing it
on to the ‘screen’ (i.e., the retina) of our eye. Is there a contradiction? (c) A diver under water, looks
obliquely at a fisherman standing on the bank of a lake. Would the fisherman look taller or shorter to the
diver than what he actually is? (d) Does the apparent depth of a tank of water change if viewed
obliquely? If so, does the apparent depth increase or decrease? (e) The refractive index of diamond is
much greater than that of ordinary glass. Is this fact of some use to a diamond cutter?
[Textbook]
23. The image of a small electric bulb fixed on the wall of a room is to be obtained on the opposite wall 3m
away by means of a large convex lens. What is the maximum possible focal length of the lens required for
the purpose? [Textbook]

46
24. At what angle should a ray of light be incident on the face of a prism of refracting angle 60° so that it just
suffers total internal reflection at the other face? The refractive index of the material of the prism is
1.524. [Textbook]
25. Answer the following questions: (a) The angle subtended at the eye by an object is equal to the angle
subtended at the eye by the virtual image produced by a magnifying glass. In what sense then does a
magnifying glass provide angular magnification? (b) In viewing through a magnifying glass, one usually
positions one’s eyes very close to the lens. Does angular magnification change if the eye is moved back?
(c) Magnifying power of a simple microscope is inversely proportional to the focal length of the lens.
What then stops us from using a convex lens of smaller and smaller focal length and achieving greater
and greater magnifying power? (d) Why must both the objective and the eyepiece of a compound
microscope have short focal lengths? (e) When viewing through a compound microscope, our eyes
should be positioned not on the eyepiece but a short distance away from it for best viewing. Why? How
much should be that short distance between the eye and eyepiece?
[Textbook]
26. (a) For the telescope described in Exercise 9.28 (a), what is the separation between the objective lens and
the eyepiece? (b) If this telescope is used to view a 100 m tall tower 3 km away, what is the height of the
image of the tower formed by the objective lens? (c) What is the height of the final image of the tower if
it is formed at 25cm? [Textbook]
27. Light incident normally on a plane mirror attached to a galvanometer coil retraces backwards as shown in
Fig. 58. A current in the coil produces a deflection of 3.5o of the mirror. What is the displacement of the
reflected spot of light on a screen placed 1.5 m away? [Textbook]

Fig 58

28. An object is kept 20cm in front of a concave mirror of radius of curvature 60 cm. Find the nature and
position of the image formed. Ans: 60 cm [2 marks]
29. The focal length of an equi-concave lens is ¾ times of radius of curvature of its surfaces.Find the
refractive index of the material of the lens.Under what condition will this lens behaves as a converging
lens ? Ans : (µ- 1)< 0 or µ< 1 [2 marks]
30.

47
48
List of Formulas
Quantity Formula Quantity Formula

Mirror Formula 1 1 1 Combination of two 𝟏 𝟏 𝟏


𝑣
+𝑢=𝑓 =𝒇 +𝒇
𝒇 𝟏 𝟐
convex lens
P = P1 + P2

Linear magnification h2 −v Prism Formula 𝛿 +𝐴


sin( 𝑚 )
m = h1 = u
sin 𝑖
μ21 = sin 𝑟 = 2
𝐴
for mirror sin( )
2

Thin Lens Formula 1 1 1 Refractive index 𝑐 𝜆𝑣𝑎𝑐𝑢𝑢𝑚


- = 𝜇= =
𝑣 𝑢 𝑓
𝑣 𝜆𝑚𝑒𝑑𝑖𝑢𝑚
𝟏
µ=𝒔𝒊𝒏 𝒊
𝒄

1
𝜇21 = 𝜇
12

Lens Maker Formula 1 1 1 Simple Microscope 𝐷


= (n21 -1)[𝑅 − 𝑅 ] (i) At near point, m = 1 + 𝑓
𝑓 1 2
𝐷
(ii)At infinity, m = 𝑓

v 𝐿 𝐷
Linear magnification m= Compound Microscope (i) At near point, m = 𝑓 (1 + )
u 0 𝑓𝑒
for lens 𝐿 𝐷
(ii)At infinity, m = 𝑓 × 𝑓
0 𝑒

Power of lens 1 Telescope 𝑓


m = 𝑓0
𝑃(𝐷) = 𝑒
𝑓(𝑚)

Derivation List:
1. Mirror equation.
2. Lens maker’s formula.
3. Deviation through prism.
4. Simple microscope
5. Compound microscope
6. Telescope

49
(Space for classroom notes)

50
(Space for classroom notes)

51
52
Ch-10 Wave Optics

Session 10.1 (Polarisation)


1. Difference in ray and wave optics:
Ray Optics Wave Optics
It concerns with particle nature of light and is based on It concerns with wave nature of light and is based on
(i) straight motion of light, (ii) laws of reflection and phenomena like interference, diffraction and
refraction of light. polarisation of light.
It explains the formation of images in mirrors, In this, waves bend around obstacles of size 50cm to
aberrations of optical images and working of optical 1m, however they reflect back when come across huge
instruments. objects.
2. Transverse waves:
It is a moving wave that consists of oscillations occurring perpendicular to the direction of propagation of
wave.
If a transverse wave is moving in the positive x-axis, its oscillations are perpendicular to x-axis.
Example, propagation of EMW.
Light is an example of a transverse wave.
For transverse waves, the displacement of the medium is perpendicular to the direction of propagation of
the wave.
3. Longitudinal waves:
It is a moving wave that consists of oscillations occurring along the direction of propagation of wave.
Mechanical longitudinal waves produce compression and rarefaction when traveling through a medium.
Example, sound waves and seismic waves (earthquake waves).
4. Unpolarised waves:
A wave in which vibrations are present in all possible directions, perpendicular to direction of propagation, is
known as unpolarised wave.
➢ Pictorial representation of unpolarized light:
Figure 1(a) shows vibrations in all directions in the
transverse plane.
In fig 1(b), double arrows represent vibrations in the
plane of paper and small dots represent vibrations
perpendicular to the plane of paper.
5. Polarised waves:
If the vibrations of a wave are present in just one direction in a plane perpendicular to the direction of
propagation, the wave is said to be plane polarized wave or polarized wave or linearly polarized wave.
➢ Pictorial representation of polarized light:
In figure 2(a), vibrations (or oscillations) are parallel
to plane of paper
In figure 2(b), vibrations are perpendicular to the
plane of paper.
In case of transverse waves (light waves), propagation is
perpendicular to direction of oscillation and hence it can be polarised.
6. Polarization:
The phenomenon of restricting the oscillations of a wave to just one direction in the transverse plane is
called polarization of waves.

53
7. Polariser:
A device that plane-polarises the unpolarised light passing through it is
called a polariser.
Tourmaline crystal, Nicol prism and Polaroids are some commonly used
polarisers.
Polaroids:
Polaroids consists of long chain molecules aligned in a particular direction.
The electric field vectors (associated with the propagating light wave)
along the direction of the aligned molecules get absorbed.
Thus, if an unpolarised light wave is incident on such a Polaroid then the light wave will get linearly polarised
with the electric field vector oscillating along a direction perpendicular to the aligned molecules.
The direction perpendicular to the direction of the alignment of the molecules of the Polaroid is known as
the pass-axis of the polaroid.
When an unpolarised light of intensity I0 is passed through a Polaroid, then intensity of the transmitted light
becomes half i.e. I0/2.
The transmitted light is also known as linearly or plane polarized light.
8. Experiment to show polarisation of light (Transverse nature of light):
When unpolarised light from an ordinary source is passed
through a Polaroid sheet P1 (polarizer), its intensity is cut down
to half.
Rotating P1 has no effect on the transmitted beam and
transmitted intensity remains constant.
Now, let an identical piece of Polaroid P2 (analyzer) be placed
before P1. As expected, the light from the lamp is reduced in Fig 4
intensity on passing through P2 alone. But now on rotating P1 has
a dramatic effect on the light coming from P1. In one position, the intensity transmitted by P1 is nearly zero.
When turned by 90º from this position, P1 transmits nearly the full intensity emerging from P2.
Conclusion:
From the experiment, it is clearly understood that light
transmitted through P1 is polarized. When the axis of P2 is parallel
to that of P1, the polarized light passes completely through P2. But,
when P2 is perpendicular to P1, at that time vibrations of polarized
light are blocked by P2 and no light is transmitted further.
Fig 5
P2 is called analyzer because it analyses whether the light through
polarizer is polarized or not.
Hence, the intensity of polarized light through the analyzer (P2) depends on the angle between the pass-axis
of the polarizer and analyzer. If both the pass-axis are parallel then intensity of the transmitted light is
maximum and if perpendicular then intensity of transmitted light is minimum. This shows that light is
transverse in nature.

54
9. Law of Malus:
It states that “when a beam of completely plane polarised light is
passed through analyzer, the intensity ‘I’ of the transmitted light
varies directly as square of the cosine of the angle ‘θ’ between the
transmission directions of polarizer and analyzer”.
Mathematically, it is given by I=I0cos2θ, where I0 is the maximum Fig 6
intensity of transmitted light after passing through polarizer.
If IT is the intensity of unpolarized light, then I0 = IT/2.
Intensity graph for Malus law is given in figure 6.
❖ Explanation:
Let I0 be the intensity and ‘a’ be the amplitude of the light transmitted by
polarizer and Ө be the angle between the planes of the polarizer and the
analyser.
The amplitude a of the light incident on the analyzer has two rectangular
components:
Fig 7
(i) acosθ, along the plane of analyzer
(ii) asinθ, perpendicular to the plane of analyzer.
So only acosθ component is transmitted by the analyzer.
We know, Intensity ∝ (Amplitude)2.
Hence, Intensity of light through analyzer, I ∝ (acosθ)2 ⟹I = ka2cos2 θ
I = I0cos2θ, where I0=ka2 is the maximum intensity of light transmitted (θ=00).
❖ Note:
For partially polarized light, intensity of emerging ray out of analyzer does not have minimum value equal to
zero. This is because in partially polarized light, vibrations exist in all directions, but these vibrations are
more in some direction and less in other directions. Hence, for any axis there will exist electric field vectors.
10. Applications of Polarization:
(i) Polaroids are used in sunglasses, to reduce the intensity and glare by cutting down the horizontally
polarized light.
(ii) For 3-D view, special type of glasses are used which have polaroids with perpendicular axes.
(iii) Polarized light is used to determine size and shape of viruses.
(iv) Polaroids are used to eliminate the dazzle from the head lights of cars, buses etc.
(v) Polaroids are used as windows in train and aeroplanes.
11. Polarization by scattering:
When light is incident on small particles of dust, air molecule etc., it is
absorbed by the atoms of these particles and re-radiated in all
directions. This phenomenon is known as scattering.
Fig 8 shows unpolarized light incident on a small dust particle or air
molecule along X-axis. The dots show vibrations perpendicular to the
plane of paper i.e along Z-axis and double arrows shows vibrations in
plane of paper i.e. along Y-axis.
For an observer looking at 900 to the Sun, the electrons vibrating
parallel to the double arrows do not transverse any energy and hence
the light scattered by molecules contains only vibrations in dot
Fig 8
direction i.e. along Z-axis.
Hence, the light observed by the observer is linearly polarized perpendicular to the plane of paper.

55
12. Polarization by Reflection:
When an ordinary (unpolarised) light is incident on surface of a transparent medium (water or glass) some
part of light goes refracted and some reflected. Both the reflected and refracted beams of light are partially
polarised. The extent of polarization depends on the angle of incidence.
For a particular angle of incidence, the reflected light is completely polarized with its vibrations (oscillations)
perpendicular to the plane of incidence. At this angle of incidence, reflected ray becomes perpendicular to
the refracted ray.
The angle of incidence at which reflected light is completely plane polarised is called polarizing or Brewster’s
angle. It is denoted by ip.
As shown in figure 9, incident unpolarized light has vibrations in both directions, one perpendicular (dots)
and other parallel (double arrows) to the plane of
incidence. At polarizing angle of incidence (ip), the
reflected and refracted rays are at 900 to each other. It
is the electrons oscillating in transparent (denser)
medium, which produce reflected wave. These
vibrations move in two directions transverse to the
Fig 9
refracted wave represented by double arrows. As
arrows are parallel to direction of reflected wave, they
cannot send energy along the direction of reflected
wave. Hence the reflected light consists of vibrations perpendicular to the plane of incidence only.
Brewster’s Law:
Let ip be the polarizing angle and rp be the corresponding angle of refraction, then
ip+rp=900⟹ rp = 900 - ip
sin 𝑖𝑝 sin 𝑖𝑝 sin 𝑖𝑝
From Snell’s law, the refractive index of denser medium is, μ = = = = tan ip
sin 𝑟𝑝 sin 90−𝑖𝑝 cos 𝑖𝑝
μ = tan ip
This relation is known as Brewster law. The law states that the tangent of the polarizing angle of incidence of
a transparent medium is equal to its refractive index.
The value of Brewster angle depends on the nature of the transparent refracting medium and the
wavelength of the light used.

56
Solved Examples
Eg 1: Why sound waves cannot be polarized? [1 mark]
Ans: Sound waves cannot be polarised as they are longitudinal waves. In longitudinal waves (sound waves),
displacement and propagation of wave takes place in same direction, and in polarization, vibrations
perpendicular to the direction of propagation of wave are restricted to just one direction. Since there is no
oscillation perpendicular to propagation, hence, their polarization is not possible.

Eg 2: The angle between the pass axis of polarizer and the analyzer is 450, write the ratio of intensities of
original light and transmitted light after passing through the analyzer. [2 marks]
Sol: Let I0 be the intensity of unpolarized light.
Then intensity of polarized light through polarizer will be I0/2.
Now, the angle between axes of polarizer and analyzer is 450, henc e according to Law of Malus,
𝐼0 𝐼0 1 2 𝐼0
Intensity of final emergent ray, I=I0cos2θ = 2
cos245 = ( )
2 √2
= 4
𝐼
Ans. The ratio of intensities of incident ray (unpolarized light) to that of emergent ray is: 𝐼00 = 4:1
⁄4
Eg 3: Two polaroids are placed crossed to each other. If one of them is rotated through 600, then what
percentage of incident unpolarised light will be transmitted by polaroids? [2 marks]
Sol: Let intensity of unpolarized light be given by I0.
Then intensity of polarized light after passing through first polaroid will be I0/2.
Initially angle between pass axis of both the polaroids is 900. After rotation by 600, angle between both the
polaroids is 900-600 = 300.
2
𝐼0 𝐼0 √3 3𝐼0
Hence, intensity of light after passing through 2nd Polaroid is, I=I0cos2θ = 2
cos230 = (
2 2
) = 8
3𝐼0 /8
Ans. Hence, polarized light transmitted = 𝐼0
× 100 = 37.5%

Eg 4: Show that ic = sin-1(cot iP), where ic is the critical angle and ip is the polarizing angle. [2 marks]
1
Sol: We know that for critical angle ic, μ = sin 𝑖𝑐
→① and from Brewster law, μ = tan ip→②
1
Thus, from ① & ②, sin 𝑖 = tan ip
𝑐
⟹ sin ic = cot ip
⟹ ic = sin-1(cot ip)
Thus, ic =sin-1(cot ip)

57
Classroom Questions:
Level – 1
1. The angle between the pass axis of polarizer and the analyzer is 450, write the ratio of intensities of original
light and transmitted light after passing through the analyzer. [2 marks]
0
2. If the angle between the pass axis of polarizer and the analyzer is 45 , write the ratio of the intensities of
original light and the transmitted light after passing through the analyzer.
Ans. 4:1 [2 marks]

Level – 2
1. Two polaroids are placed crossed to each other. If one of them is rotated through 600, then what percentage
of incident unpolarised light will be transmitted by polaroids? [DPS]
Ans. 37.5% [2 marks]
2. Two polaroids P1 and P2 are placed 90° to each other. Find the transmitted intensity if a third polaroid P3 is
placed between P1 and P2 bisecting the angle between them.Take intensity of unpolarised light as I0
.[Outside Delhi 2013, Outside Delhi 2010, Foreign 2013, Foreign 2011] Ans. I0/8 [2 marks]
3. Three identical Polaroid sheets P1, P2 and P3 are oriented so that the pass axis of P2& P3 are inclined at angles
of 600& 900 respectively w.r.t. pass axis of P1. A monochromatic source (S) of intensity I0 is kept in front of P1.
Find intensity of this light as observed by observers O1, O2& O3. [Compartment 2013, Prakash]
Ans. I0/2, I0/8, 3I0/32 [3 marks]

Fig 10

58
Homework Sheet 10.1
Theory
1. What is meant by a linearly polarised light? Briefly explain method of scattering for producing polarised
light. [Outside Delhi 2013, Outside Delhi 2010] [3 marks]
2. Describe briefly, with the help of suitable diagram, how the transverse nature of light can be
demonstrated by the phenomenon of polarization. [Compartment 2011,2013]
[3 marks]
3. State Malus law. Draw a graph showing the dependence of intensity of transmitted light on the angle
between polarizer and analyser. [DPS] [2 marks]
4. Define polarizing angle. Derive relation between polarizing angle and refractive index.
[DPS] [3 marks]
5. What is an unpolarized light? Describe briefly, with the help of a necessary diagram, the polarization of
light by reflection from a transparent medium. [Outside Delhi 2012, Delhi 2010] [3 marks]
6. Which special characteristic of light is demonstrated only by the phenomenon of polarization?
Distinguish clearly between linearly polarized light and unpolarized light.
Ans. Transverse nature of light is characterized by phenomenon of polarization. Linearly polarized light is
one in which the direction of oscillation is restricted to one direction in a plane which is perpendicular to
plane of propagation whereas in unpolarized light in which light oscillates in all directions instead of one
direction. [DPS] [2 marks]
Application Based
7. Light waves can be polarized, sound waves cannot be. Why? [DPS] [1 mark]
8. Sunglasses are made of polaroids and not of coloured glasses. Why? Ans. To avoid glare, sunglasses are
made of polaroids so that light coming from Sun is polarized. [1 mark]
Numerical
9. What is the polarising angle for of a medium of refractive index √3? Ans. 600 [1 mark]
0
10. A ray of light strikes a glass plate at an angle of 60 . If the reflected and refracted rays are perpendicular
to each other, find the refractive index of the glass. Ans. 1.732 [1 mark]
0
11. Four polaroids are so placed that the transmission-axis of each is inclined at an angle of 30 from the axis
of previous Polaroid in the same direction. If unpolarised light-beam of intensity I0 falls on the first
Polaroid, then what will be the intensity of the light emerging out from the last Polaroid?
Ans. 0.21I0 [2 marks]

59
(Space for classroom notes)

60
Session 10.2 (Huygen’s Principle)
1. Some Important Terms:
(i) Wavelength (λ):
The distance between two successive crests or troughs of a wave is called wavelength.
SI unit is metres.
(ii) Frequency (ν):
Number of oscillations produced by a source in 1 sec is known as frequency.
SI unit is hertz (Hz).
Mathematically, νλ = c (Speed of light)
(iii) Amplitude (A or a):
The maximum displacement in either direction of the wave is known as amplitude.
(iv) Intensity (I):
It is the average power transferred per unit area.
SI unit is watt/m2.
Mathematically, I ∝ a2.
(v) Phase (θ or φ):
The argument of sine or cosine in the expression for displacement of a wave is defined as the phase.
For displacement y = a sin ωt ; term ωt = phase or instantaneous phase.
(vi) Phase Difference:
The difference between the phases of two waves at a point is called phase difference i.e. if y1 = a1sin
ωt and y2 = a2 sin(ωt + φ) so phase difference = φ.
(vii) Path difference (Δx):
The difference in path lengths of two waves meeting at a point is called path difference between
the waves at that point.
Relation between path difference and phase difference
λ
Path difference (Δx) = 2π  phase difference(φ)

𝑇𝑖𝑚𝑒 𝑑𝑖𝑓𝑓. 𝑃ℎ𝑎𝑠𝑒 𝑑𝑖𝑓𝑓. 𝑃𝑎𝑡ℎ 𝑑𝑖𝑓𝑓.


= =
𝑇 2𝜋 𝜆
ΔT Δφ Δλ
= =
𝑇 2𝜋 𝜆

2. Wavefront:
It is defined as continuous locus of all such particles which are vibrating in the same phase at a given instant.
The speed with which the wavefront moves outwards from the source
is called the speed of the wave.
Different types of wave fronts:
The shape of a wavefront is usually determined by the geometry of the
source.
(a) Spherical wavefront:
A point source has wavefronts that are spheres whose centers are Fig 12
Fig 11
at the point source. (Figure 11).
(b) Cylindrical wavefront:
If source of light is a slit or linear in shape then wave front is cylindrical in shape.
Example, a fluorescent tube would have wavefronts that are cylinders
concentric with the tube itself.(Figure 12).
(c) Plane wavefront:
If source of light is at infinity then wavefronts are planar in shape.
A spherical wavefront appears as plane wavefront after travelling a large
distance from the point source.
❖ Note:
The phase difference between any two points situated on the same wavefront is zero.

61
3. Ray of light:
It is an arrow drawn perpendicular to a wavefront in the direction of propagation of a wave.
Time taken for light to travel from one wavefront to another is the same along any ray.
In case of a plane wavefront, the rays are parallel. A group of parallel rays is called a beam of light.

4. Huygens’ Principle:
It is a geometrical method to find the position of a given
wavefront at any instant if its present position is known.
According to Huygens principle, “each point of the
wavefront is the source of a secondary disturbance and
the wavelets emanating from these points spread out in
all directions with the speed of the wave. These wavelets
emanating from the wavefront are usually referred to as
secondary wavelets and if we draw a common tangent to
all these spheres, we obtain the new position of the wavefront at a later time.”
Limitation of Huygen’s principle:
It is considered that there is no backward wave because there is no backward flow of energy when a wave
travels in forward direction. But when light travels, energy flows in all directions.
5. Reflection on the basis of wave theory (Must do – Examiner’s Favourite):
Consider a plane wavefront ‘AB’ incident at an angle ‘i’ on a
reflecting surface ‘MN’.
If ‘v’ is the speed of light in the medium and τ is the time taken by the waveform to advance from point ‘B’ to
‘C’, then distance BC=vτ.
According to Huygen’s principle, in order to construct the
reflected wavefront after time 𝜏, we draw a sphere of radius vτ
taking A as centre. Now draw a tangent to this sphere from C
which meet sphere at point ‘E’. Thus, CE is the reflected
wavefront at angle of refraction ‘r’ with surface ‘MN’.
So, AE = BC= vτ
[distance travelled by a wave in a medium in equal interval of time is equal]
In ΔABC & ΔAEC are congruent (by RHS congruency rule)
⟹i = r (by CPCT)
This is the law of reflection.
6. Refraction of a Plane wave in denser medium (Must do – Examiner’s Favourite):
Consider PP’ surface separating medium 1 and medium 2 as shown in figure 16.
Let v1 and v2 represent the speed of light in medium 1 and 2 respectively.
Let us assume a plane waveform AB propagating in the direction A’A
incident on the interface at an angle ‘i’.
Let τ be the time taken by the wavefront to travel the distance ‘BC’.
Thus, BC = v1τ
According to Huygen’s principle, draw a sphere of radius v2τ from a point A
in medium 2 and then draw a tangent from ‘C’ to sphere at point ‘E’.
So AE = v2τ
CE is refracted wavefront which makes angle ‘r’ with the PP’.
Now, for ΔABC & ΔAEC,
𝐵𝐶 𝑣1 𝜏 𝐴𝐸 𝑣2 𝜏
sin i = 𝐴𝐶 = 𝐴𝐶
& sin r = 𝐴𝐶 = 𝐴𝐶
.
sin 𝑖 𝑣
So, sin 𝑟 = 𝑣1 .
2
𝑐 𝑐
Let n1 and n2 be the refractive indexes of medium 1 & 2, hence, n1 = 𝑣 & n2 = 𝑣 .
1 2
𝑛2 𝑣1 𝑛2 sin 𝑖
Hence, 𝑛1
= 𝑣2
. Hence, 𝑛1
= sin 𝑟
⟹n1sin i = n2 sin r, which is Snell’s law of refraction.
Note: In going from medium 1 to medium 2, frequency of light does not change as it depends on source.
𝑣 𝜆 𝜈 𝜆
Thus, 𝑣1 = 𝜆1 𝜈 = 𝜆1 .
2 2 2
Since in going from rarer to denser medium, speed of light decreases wavelength also decreases. 62
7. Refraction of a Plane wave in rarer medium (Do it yourself):
Consider PP’ surface separating medium 1 and medium 2.
Let v1 and v2 represent the speed of light in medium 1 and 2 respectively.
Let us assume a plane waveform AB propagating in the direction A’A
incident on the interface at an angle ‘i’.
Let τ be the time taken by the wavefront to travel the distance ‘BC’.
Thus, BC = v1τ
According to Huygen’s principle, draw a sphere of radius v2τ from a point A
in medium 2 and then draw a tangent from ‘C’ to sphere at point ‘E’.
So AE = v2τ
CE is refracted wavefront which makes angle ‘r’ with the PP’.
Now, for ΔABC & ΔAEC,
𝐵𝐶 𝑣1 𝜏 𝐴𝐸 𝑣2 𝜏 Fig 17
sin i = 𝐴𝐶 = 𝐴𝐶
& sin r = 𝐴𝐶 = 𝐴𝐶
.
sin 𝑖 𝑣
So, = 1.
sin 𝑟 𝑣2
𝑐 𝑐
Let n1 and n2 be the refractive indexes of medium 1 & 2, hence, n1 = 𝑣 & n2 = 𝑣 .
1 2
𝑛 𝑣 𝑛 sin 𝑖
Hence, 𝑛2 = 𝑣1 . Hence, 𝑛2 = sin 𝑟 ⟹n1sin i = n2 sin r, which is Snell’s law of refraction.
1 2 1
Note: In going from medium 1 to medium 2, frequency of light does not change as it depends on source.
𝑣 𝜆 𝜈 𝜆
Thus, 𝑣1 = 𝜆1 𝜈 = 𝜆1 .
2 2 2
Since in going from denser to rarer medium, speed of light increases wavelength also increases.
8. Reflection and refraction of plane wavefronts:
(i) Prism:
Fig 18 shows refraction of a plane wavefront through a prism.
Since speed of light is slower in glass, thus lower part of
wavefront is slowed more since thickness is more and at upper
part it is slowed down least due to less thickness. This explains
tilting of wave front after refraction through glass prism.
Fig 18
(ii) Lens:
Fig 19 shows refraction of plane wavefront through convex lens. Since,
central part passes through greatest thickness, thus it is slowed down
most. Upper and lower part pass through least thickness, thus are slowed
down least. So, the emerging wavefront is spherical and converges to
focus.
Fig 19

(iii) Mirror:
Fig 20 shows reflection of plane wavefront through concave mirror.
Since the central part has to travel greatest distance, thus the reflected
wavefront’s central part is closer to mirror than other parts. Hence, the
reflected wavefront is spherical and converges to focus.

Fig 20

63
Classroom Questions:
Level – 1
1. Monochromatic light of wavelength 589 nm is incident from air on a water surface. What are the
wavelength, frequency and speed of (a) reflected, and (b) refracted light? Refractive index of water is
1.33.Ans. 589nm, 5.09×1014 Hz, 3×108m/s, 444nm, 5.09×1014 Hz, 2.26×108m/s. [3 marks]

64
Homework Sheet 10.2
Theory
1. State Huygens’ principle. [Delhi 2013, Outside Delhi 2012, Foreign 2011] [1 mark]
2. Define term ‘wave front’. [Board 2018] [1 mark]
3. What is phase difference between two points on a wavefront? Ans. 00 [1 mark]
4. What is geometrical shape of the wavefront emitted a light source in the form of (i) point (ii) narrow
slit? Ans. Spherical, Cylindrical [1 mark]
5. Verify, using Huygens principle, Snell’s law of refraction of a plane wave propagating from a rarer to a
denser medium. [Compartment 2013, Delhi 2013, Outside Delhi 2012, Foreign 2011] [3 marks]
6. Derive Snell’s law on the basis of huygen’s wave theory when light is travelling from a denser to a rarer
medium. [Board 2016] [3 marks]
7. How is a wave front different from a ray? Draw the geometrical shape of the wave fronts when (i) light
diverges from a point source, and (ii) light emerges out of a convex lens for incident plane wavefront.
[Delhi 2013, Outside Delhi 2012, Foreign 2011,DPS] [3 marks]
8. Deduce laws of reflection on the basis of Huygen’s wave theory. [Board 2018] [2 marks]
Diagram Based
9. Draw diagrams to show behaviour of pane wavefront when they pass through (a) a thin prism, (b) a thin
concave mirror. [2 marks]
Application Based
10. Use Huygens geometrical construction to show how a plane wave-front at t=0
propagates and produces a wave-front at a later time.
Ans. Huygens geometrical construction for a plane wave propagating to the right. F1 F2 is the
plane wavefront at t = 0 and G1G2 is the wavefront at a later time τ. The lines A1A2, B1B2 … etc,
are normal to both F1F2 and G1G2 and represent rays.
[Compartment 2013] [1 mark]
11. When monochromatic light is incident on a surface separating two media, why does
the refracted light have the same the same frequency as that of the incident light?
Ans. This is because the source of light remains same and hence frequency does not
change.[Delhi 2013, Outside Delhi 2012, Foreign 2011, Compartment 2011, Delhi 2011]
[1 mark]
12. When light travels from a rarer to a denser medium, the speed decreases. Does the
reduction in speed imply a reduction in the energy carried by the light wave?
Ans. No, because energy of light depends on frequency and frequency of light does not
change on changing medium [1 mark]
13. Show that a plane wavefront incident on a concave mirror is reflected as spherical
Fig 21
wave front. [1 mark]

65
66
Session 10.3 (Interference and Young’s Double Slit Experiment)
1. Principle of superposition:
When a number of waves travelling through a medium superpose on each other, the resultant displacement
at any point at a given instant is equal to the vector sum of the displacements due to individual waves at that
point.
If ⃗⃗⃗⃗
𝑦1 , ⃗⃗⃗⃗
𝑦2 , ⃗⃗⃗⃗
𝑦3 ,…., ⃗⃗⃗⃗
𝑦𝑛 be displacements due to different waves acting separately, then according to principle of
superposition, resultant wave y is given by, 𝑦=𝑦 ⃗⃗⃗⃗1 + ⃗⃗⃗⃗
𝑦2 + ⃗⃗⃗⃗
𝑦3 +….….+ ⃗⃗⃗⃗
𝑦𝑛
2. Polychromatic and monochromatic light:
Light source having only a single wavelength (or frequency) of light and composed of only one colour is
known as monochromatic light like light from sodium lamp.
Light source having several frequency (or wavelength) of light and can show different colours is known as
polychromatic light like sunlight.
3. Coherent and Incoherent sources:
(i) Two sources of light are said to be coherent if they emit light waves of same frequency (or
wavelength) with a zero or constant phase difference.
(ii) Two sources of light are said to be incoherent if they do not emit light waves with a constant phase
difference are called incoherent sources.
Two different sources can’t be coherent:
Light is emitted due to de-excitation of electrons from higher orbits of the various atoms of the light source.
The de-excitation can also be to different levels in different atoms resulting in emission of different
wavelengths. A light source has numerous atoms in it. When we take two light sources it is not possible for all
the atoms to get de-excited to the same shell at the same time (causing phase difference).
4. Interference of light:
When two light waves of the same frequency and having zero or constant phase difference (coherent
sources) travelling in the same direction superpose each other, the intensity in the region of superposition
gets redistributed, becoming maximum at some points and minimum at others. This phenomenon is called
interference of light.
For interference to take place, source of light should be monochromatic and coherent.
5. Intensity at a point in interference pattern:
Suppose the displacements of two light waves from two coherent sources S1 and S2 at point P at any
time t is given by, y1=acos ωt and y2 = acos(ωt+φ), where ‘a’ is the amplitude of the two waves, φ is the
constant phase difference between the two waves.
Let I0 be the intensity of light produced by each wave.
We know that, Intensity∝ (Amplitude)2
Hence, I0 ∝ a2
⟹ I0 = ka2⟶①
By the superposition principle, the resultant displacement at point P is,
y = y 1 + y2
⟹ y = a [cosωt + cos(ωt+φ)]
𝐷−𝐶 𝐶+𝐷
Using cos C + cos D = 2 cos 2
cos2
, we get,
𝜑 φ
⟹y = 2a cos 2 cos (ωt + 2 )
𝜑
In the above displacement expression, 2a cos 2 is the amplitude.
Now, Intensity∝ (Amplitude)2
𝜑
Hence, I ∝ (2a cos 2 )2
𝜑
⟹ I = 4ka2cos2 2
𝜑
⟹I = 4I0 cos2 2 [From ①]

6. Constructive and Destructive interference:


Wshen two waves of same wavelength superimpose on each other in phase, their amplitudes add up and we
get maximum intensity. This is called constructive interference.

67
When two waves of same wavelength superimpose on each other out of phase, their amplitudes gets
subtracted and we get minimum intensity and this is called destructive interference.
Condition for constructive interference:
Constructive interference occurs when resultant intensity is maximum.
𝜑
So cosΦ = 1. Thus, Φ=0, 2π, 4π, … [I = 4I0 cos2 2 ]
So phase difference should be integral multiple of 2π.
Now, for a phase difference of 2π, path difference between the
sources is λ.
Thus if Δx is path difference between two superposing waves, then,
2πΔx
𝜆
=0, 2π, 4π, ...
⟹Δx =0, λ, 2λ, … = nλ Fig 22
Thus, for constructive interference, path difference between two superposing waves should be an integral
multiple of λ.
The locus of all points for constructive interference is shown in Fig 22. As can be seen clearly, the locus of all
such points is a hyperbola.
Condition for destructive interference:
Destructive interference occurs when Resultant intensity is minimum.
𝜑
So cosΦ = -1. Thus, Φ= π, 3π, … [I = 4I0 cos2 2 ]
So phase difference should be odd multiple of π.
Path difference:
2πΔx
Thus, 𝜆
= π, 3π, 5π, ...
(2𝑛−1)𝜆
⟹Δx =λ/2, 3λ/2, … = 2
.
Thus, for destructive interference, path difference between two superposing waves should be an odd integral
of λ/2.

68
7. Young’s Double Slit Experiment(Must Do - Examiner Favourite):
As shown in fig 23, let S be a monochromatic source of light of wavelength
λ. S1 and S2 are two narrow slits at equal distances from S.
Since, source of both S1 and S2 are same, thus they act as two coherent
sources, separated by small distance d.
Interference fringes are obtained on a screen placed at a distance D from
sources S1 and S2.
Consider a point P on the screen at a distance x from the centre O(fig 24).
The nature of the interference at the point P depends on path difference,
Δx = S2P – S1P Fig 23
By Pythagoras theorem,
𝑑 𝑑
(S2P)2- (S1P)2=[D2+(x + 2 )2]- [D2+(x - 2 )2]
Thus, (S2P-S1P)(S2P+S1P) = 2xd
2𝑥𝑑
⟹ S2P-S1P =𝑆
2 + 𝑆1 𝑃
𝑃
In practice, the point P lies very close to O, therefore, S1P = S2P = D
2𝑥𝑑
Thus, Δx = S2P-S1P = 2𝐷
𝒙𝒅
⟹Δx = 𝑫 Fig 24
Positions for Bright Fringes:
For constructive interference, Δx = xd/D = nλ
⟹ x = nDλ/d.
For n=0, x0=0 (central bright fringe),
For n= 1, x1= Dλ/d (first bright fringe) and so on.
𝒏𝑫𝝀
Thus, for nth bright fringe, xn = 𝒅
.
Positions for Dark Fringes:
𝜆
For destructive interference, Δx = xd/D = (2n-1)
2
𝐷𝜆
⟹ x = (2n - 1)
2𝑑
𝐷𝜆
For n = 1, x1’ =
2𝑑
3𝐷𝜆
For n = 2, x2’ =
2𝑑
𝑫𝝀
For n = n, xn’ = (2n - 1)𝟐𝒅
Fringe Width (β):
It is the separation between two successive dark or bright fringes.
Thus, width of a fringe, β = xn-xn-1= nDλ/d - (n-1)Dλ/d or (2n-1)Dλ/2d - (2n-3)Dλ/2d= Dλ/d.
Thus, β = Dλ/d.
The locus of the point P lying in the x-y plane such that
S2P – S1P (= Δ) is a constant, is a hyperbola.
Thus the fringe pattern will strictly be a hyperbola;
however, if the distance D is very large compared
to the fringe width, the fringes will be very nearly
straight lines.
Here, λ/d is known as angular separation.
The intensity distribution curve is given in fig 25. Fig 25
Condition for Interference:
Let s be the size of source and S is its distance from two slits.
For interference pattern to be observed,
𝒔 𝝀
Necessary condition, 𝑺<𝒅 should be observed.

69
8. Conservation of energy:
When two waves superimpose out of phase at a point, destructive interference takes place. Thus, the
intensity of light and hence the energy at that point is zero. It means, the light energy at that point is
completely destroyed. Since energy cannot be destroyed, so the energy disappearing at the point of
destructive interference appears at the point of constructive interference. So, we can say that energy is
simply re-distributed in the interference pattern. Thus, the law of conservation of energy holds good during
the phenomenon of interference.
9. Ratio of intensities in maxima and minima :
Let a and b be the amplitudes of the waves emitted from S1 and S2 respectively and Φ be the phase
difference between them. Let Ia and Ib be the intensities of the two sources.
Amplitude of the resultant wave will be given by
𝐴 = √𝑎2 + 𝑏 2 + 2𝑎𝑏 cos 𝛷
Intensity of the resultant wave will be given by
Ir = Ia + Ib + 2√Ia Ib cos 𝛷
We know that I1/I2=a12/a22, where I1 and I2 are the intensities of the two sources.
Max amplitude is given by a1+a2 and minimum by a1 - a2.
Thus, the ratio of intensities at maxima and minima is given by,
𝑟+1 2
Imax/Imin = (a1+a2)2/ (a1-a2)2 = (𝑟−1) , where r = a1/a2.
10. Displacement of interference fringes:
When a thin transparent sheet of thickness t and refractive index µ is inserted in the path of the interfering
beams, then extra path difference is given by, Δp=(µ-1)t.
𝐷𝑝
Suppose the present position of the particular fringe is, x = 𝑑
𝐷
Then the new position of the same fringe will be, x’= (p+Δp)
𝑑
𝐷𝛥𝑝
Hence, the lateral displacement of the particular fringe on the screen is, Δx = x’ – x = 𝑑
𝑫 𝜷 𝐷𝜆
Δx = (μ - 1)t = (μ - 1)t [β = ]
𝒅 𝝀 𝑑

70
Solved Examples
Eg 5: In Young’s double slit experiment, the two slits 0.15mm apart are illuminated by monochromatic light
of wavelength 450nm.The screen is 1.0m away from the slits. (a) Find the distance of the second (i) bright
fringe, (ii) dark fringe from the central maximum, (b) How will the fringe pattern change if the screen is
moved away from the slits? [Outside Delhi 2010] [3 marks]
-4
Sol: Distance between the two sources d = 0.15 mm = 1.5 × 10 m
Wavelength λ = 450nm = 4.5 × 10-7m
Distance of screen from source D = 1m
(i) The distance of nth order bright fringe from the central fringe is given by
𝐷𝑛𝜆
yn = 𝑑
For second bright fringe,
2𝐷𝜆
y2 = 𝑑
2 ×1 ×4.5 × 10−7
y2 = 1.5 × 10−4
= 6 × 10-7+4= 6 × 10-3m
The distance of the second bright fringe
y2 = 6mm
(ii) The distance of nth order dark fringe from central fringe is given by
𝐷𝜆
y’n = (2n-1)2𝑑
For second dark fringe n = 2
𝐷𝜆 3𝐷𝜆
y’n= (2 × 2-1) =
2𝑑 2𝑑
3 1 ×4.5 × 10−7
y’n = 2 × 1.5 × 10−4
The distance of the second dark fringe
y’n = 4.5 mm
𝐷𝜆
(b) Fringe width increases as β = 𝑑
𝑜𝑟 𝛽 ∝ 𝐷

Eg 6: A beam of light, consisting of two wavelengths 560nm and 420 nm, is used to obtain interference
fringes in a Young’s double slit experiment. Find the least distanced from the central maximum, where the
bright fringes, due to both the wavelengths coincide. The distance between the two slits is 4.0 mm and the
screen is at a distance of 1.0m from the slits. [3 marks]
-3
Sol: Given, D =1m, d=4 × 10 m, λ1 = 560nm, and λ2 = 420nm
Let nth order bright fringe of λ1 coincide with (n+1)th order bright fringe of λ2.
𝐷𝑛𝜆1 𝐷(𝑛+1)𝜆
2
𝑑
= 𝑑
(λ1> λ2)
nλ1 = (n+1)λ2
𝑛+1 𝜆
𝑛
= 𝜆1
2
1 560 × 10−9
1+ 𝑛 = 420 × 10−9
1 4
1+𝑛 = 3
1 1
𝑛
= 3
n=3
∴ Least distance from the central fringe where bright fringe of two wavelengths coincides
= distance of 3rd order bright fringe of λ1
3𝐷𝜆1
yn = 𝑑
3 ×1 ×560 × 10−9
= 4 × 10−3
yn = 420 × 10-6m
=0.42 × 10-3m
yn = 0.42mm
Ans. 3rd bright fringe of λ1 and 4th bright fringe of λ2 coincide at 0.42mm from the central fringe.

71
Eg 7: In a Young’s double slit experiment, monochromatic light of wavelength 630nm illuminates the pair of
slits and produces an interference pattern in which two consecutive bright fringes are separated by 8.1mm.
Another source of monochromatic light produces the interference pattern in which the two consecutive
bright fringes are separated by 7.2mm. Find the wavelength of light from the second source. [2 marks]
𝐷𝜆
Sol: Fringe width Β =
𝑑
For given young’s double slit experiment (YDSE) D and d are constants.
𝛽1 𝜆1
𝛽2
= 𝜆2
𝐷
As 𝑑 = constant
Here, β1 = 8.1 × 10-3m
λ1 = 630nm = 630 × 10-9m
β2 = 7.2 × 10-3m
𝛽1 𝜆1
=
𝛽2 𝜆2
Wavelength of light from the second source
𝛽 7.2 × 10−3
λ2 = 𝛽2 × 𝜆1 = 8.1 × 10−3
× 630 × 10−9
1
8
= 9 × 630 × 10-9
= 560 × 10-9m
Ans. λ2 = 560nm

Eg 8: How does the fringe width of interference fringes change, when the whole apparatus of Young’s
experiment is kept in a liquid of refractive index 1.3? [1 mark]
1
Sol: The new fringe width becomes times of original fringe width as
1.3
𝛽𝑎𝑖𝑟 𝛽𝑎𝑖𝑟
=  => βmed =
𝛽𝑚𝑒𝑑 

Eg 9: The intensity at the central maxima (O) in a Young’s double slit experiment is Io. If the distance OP
equals one-third of fringe width of the pattern, show that the intensity at point P would be Io/4. [2 marks]
Sol: Given, OP =yn
The distance OP equals one-third of fringe width of the pattern
𝛽 1 𝐷𝜆 𝐷𝜆
i.e., yn = 3 = ( ) = 3𝑑
3 𝑑
𝑑𝑦𝑛 𝜆
𝐷
= 3
𝑑𝑦 𝜆
Path difference S2P – S1P = 𝐷𝑛 = 3
∴ Phase difference corresponding to path difference, is,
2𝜋 𝜆
Phase difference = 𝜆
× 3
2𝜋
⟹Phase difference = 3
If intensity at central fringe is Io, then intensity at a point P where phase difference is φ, is given by
I = Iocos2(φ/2)
2𝜋 𝜋
I = Io(cos 2×3)2 = Io(𝑐𝑜𝑠 3 )2
1 𝐼𝑜
= Io(2)2 = 4
𝐼
Ans. Hence, the intensity at point P would be 4𝑜
Eg 10: In a Young’s double slit experiment, the two slits are kept 2mm apart and the screen is positioned
140cm away from the plane of the slits. The slits are illuminated with light of wavelength 600nm. Find the
distance of the third bright fringe, from the central maximum, in the interference pattern obtained on the
screen. If the wavelength of the incident light wave changed to 480nm, find out the shift in the position of
third bright fringe from the central maximum. [2 marks]
-3
Sol: Given, d=2 × 10 m, D =140cm = 1.4m
and λ = 600 × 10-9m
𝐷𝑛𝜆
The separation of nth order bright fringe from central fringe is given by yn = 𝑑
,
For 3rd order bright fringe
1.4 ×3 ×600 × 10−9
y3 = 2 × 10−3
= 1.26 × 10-3m

72
y3 = 1.26 mm
For wavelength λ = 480nm,
𝐷𝑛𝜆
For shift of fringe y =
𝑑
Where, λ = (480-600)nm = -120 nm
= -120 × 10-9m
Negative sign indicates that shift take place toward central fringe. The magnitude of shift is given by
1.4 ×3 ×120 ×10−9
y = 2 × 10−3
= 252 × 10-6
Ans. y = 0.252 mm

Eg11: In Young’s double slit experiment, interference fringes are observed on a screen, kept at D from the
slits. If the screen is moved towards the slits by 5 × 10-2m, the change in fringe width is found to be 3 × 10-
5
m. If the separation between the slits is 10-3 m, calculate the wavelength of the light used. [2 marks]
Sol: Given, d=10-3m, β = −3 × 10-5m,
D = −5 × 10-2m
𝐷𝜆
∵ Fringe width β = ,
𝑑
Negative sign indicate that fringe width and D decreases.
𝜆
β = 𝑑 𝐷
𝑑× 𝛽
λ=
𝐷
10−3 ×(−3 × 10−5 )
where, λ = (−5 × 10−2 )
λ = 600 × 10-9m
Ans. Wavelength of light λ =600nm

Eg 12: What is the effect on the interference fringes in a Young’s double-slit experiment due to each of the
following operations: [5 marks]
(a) the screen is moved away from the plane of the slits;
Ans. When screen is moved away from the slits (D increases), angular separation (λ/d)remains constant. But,
𝐷𝜆
the fringe width 𝛽 = 𝑑 ,increases.
(b) the (monochromatic) source is replaced by another (monochromatic) source of shorter wavelength;
𝐷𝜆
Ans. When light is replaced by another light of shorter wavelength, then the fringe width 𝛽 = 𝑑
and angular
width λ/d both decrease.
(c) the separation between the two slits is increased;
𝐷𝜆
Ans. When the separation between two slits (d) is increased, then fringewidth 𝛽 = 𝑑 and angular width λ/d
both decrease.
(d) the source slit is moved closer to the double-slit plane;
Ans. Let s be the size of the source and S its distance from the plane of the two slits. For interference fringes
to be seen, the condition s/S < λ/d should be satisfied; otherwise, interference patterns produced by
different parts of the source overlap and no fringes are seen. Thus, as S decreases (i.e., the source slit is
brought closer), the interference pattern gets less and less sharp, and when the source is brought too close
for this condition to be valid, the fringes disappear. Till this happens, the fringe separation remains fixed.
(e) the width of the source slit is increased,
Ans.As the source slit width increases, fringe pattern gets less and less sharp. When the source slit is so wide
that the condition s/S S < λ /d is not satisfied, the interference pattern disappears.
(f ) the monochromatic source is replaced by a source of white light?
Ans.The interference patterns due to different component colours of white light overlap (incoherently). The
central bright fringes for different colours are at the same position. Therefore, the central
fringe is white. The fringe closest on either side of the central white fringe is red and the farthest will appear
blue. After a few fringes, no clear fringe pattern is seen.

73
Classroom Questions:
Level – 1
1. In Young’s double slit experiment using mono-chromatic light of wavelength λ, the intensity of light at a
point on the screen where path difference is λ, is K units. Find out the intensity of light at a point where path
difference is 2λ/3. [Compartment 2012] Ans. K/4 [2 marks]
2. Yellow light of wavelength 6000𝐴̇ produces fringe of width 0.8mm in Young’s double slit experiment. What
will be the fringe width if instead wavelength of 7500𝐴̇ is used and separation is doubled?
Ans. 0.5mm [2 marks]
Level – 2
1 A beam of light, consisting of two wavelengths 560nm and 420 nm, is used to obtain interference fringes in a
Young’s double slit experiment. Find the least distanced from the central maximum, where the bright
fringes, due to both the wavelengths coincide. The distance between the two slits is 4.0 mm and the screen
is at a distance of 1.0m from the slits. [3 marks]
2 A beam of light consisting of two wavelengths, 800 nm and 600nm is used to obtain the interference fringes
in a Young’s double slit experiment on a screen place 1.4m away. If the two slits are separated by 0.28mm,
calculate the least distance from the central bright maximum where the bright fringes of the two
wavelengths coincide. [Outside Delhi 2012] Ans. 1.2×10-2m [2 marks]
3 The intensity at the central maxima in Young’s double slit experiment is 𝐼0 . Find intensity at the point on
𝜆 𝜆 𝜆 3 𝐼0 𝐼0
which path difference is 6 , 4 𝑎𝑛𝑑 3. Ans. I1 = 4 𝐼0 , I2 = 2 3
,I = 4
[Board 2016] [3 marks]

74
Homework Sheet 10.3
Theory
1. What are coherent sources? State conditions for two light sources to be coherent. [DPS] [2 marks]
2. Describe Young's double slit experiment to produce interference pattern due to a monochromatic
source of light. Deduce the expression for the fringe width.
[Outside Delhi 2014, Compartment 2011, Delhi 2011, Prakash] [5 marks]
Application Based
3. Why are coherent sources necessary to produce a sustained interference pattern? Ans. Two source of
emitting light waves of same frequency or wavelength and of constant phase difference are required
two see interference pattern and we can obtain such nature of light waves from coherent sources only.
So, we require coherent sources to produce the interference of light.[Compartment 2012] [1 mark]
4. What is the effect on the interference fringes in a Young’s double slit experiment when (i) the
separation between the two slits is decreased? (ii) The width of the source slit is increased? (iii) The
monochromatic source is replaced by a source of white light? Justify your answer in each case.
[Outside Delhi 2012, DPS, Udgam] [3 marks]
5. How does the fringe width get affected, if the entire experimental apparatus of Young is immersed in
water? [Compartment 2011, Outside Delhi 2011] [1 mark]
6. The intensity at the central maxima (O) in a Young’s double slit experiment is I0. If the distance OP
equals one-third of the fringe width of the pattern, show that the intensity at point P would be I0/4.
[Foreign 2011, Delhi 2010] [2 marks]
7. Why no interference pattern is observed when 2 coherent sources are (i) too close, (ii) very far apart.
Ans. (i) In this case, fringe width becomes very large. Even a single fringe may occupy the entire screen.
The interference pattern is not observable. (ii) In this case, the fringe width goes on increasing. At very
large separation, it becomes too small to be detected. The interference pattern cannot be observed.
[Prakash] [2 marks]
Numerical
8. In Young’s double slit experiment, the two slits 0.15mm apart are illuminated by monochromatic light
of wavelength 450nm.The screen is 1.0m away from the slits. (a) Find the distance of the second (i)
bright fringe, (ii) dark fringe from the central maximum, (b) How will the fringe pattern change if the
screen is moved away from the slits? [Outside Delhi 2010] Ans. 6×10-3m, 7.5×10-3m[3 marks]
9. Two coherent sources have intensities in the ratio 25:16. Find the ratio of intensities of maxima to
minima? Ans. 81:1 [2 marks]
10. In Young’s double slit experiment, two coherent sources are 1.5mm apart and fringes are obtained at a
distance of 2.5m from them. If wavelength of source light is 589.3nm, then find the number of fringe
patterns, which is 4.9×10-3m wide. Ans. 5 [2 marks]

11. (a) If one of two identical slits producing interference in Young’s experiment is covered with glass, so
that the light intensity passing through it is reduced 50%, find the ratio of the maximum and minimum
intensity of the fringe in the interference pattern.
(b) What kind of fringes do you expect to observe if white light is used instead of monochromatic light?
[Board 2018] [3 marks]

75
76
Session 10.4 (Diffraction)
1. Diffraction:
The phenomenon of bending of light around the corners of small obstacles or apertures and in consequent
spreading into regions of geometrical shadow is called diffraction of light.
2. Single Slit Experiment(Must Do - Examiner Favourite):

In single slit experiment, a monochromatic source illuminates a slit (LN) of size ‘a’ when a planar wavefront from
source S reaches the slit.
According to Huygens principle, ach point on the slit acts as secondary source of disturbance and emanates
wavelets in all directions. When these wavelets superpose on a screen kept at a distance D (D>>a) from slit plane,
then a diffraction pattern is observed on a screen in which central region is of very high intensity (central
maxima). On both sides of central maxima, there are alternate dark and bright regions such that intensity of these
bright regions (secondary maxima’s) keeps on decreasing as we move away from central maxima on either side.
To understand these observations, we consider the fig 26.

Fig 26

Here, LN represents a slit of size ‘a’. M is the mid-point of the slit. C is the central point of the screen and a normal
on slit is drawn at point M which meets the screen at point C. Now, position of any point on screen is taken by
taking angle with this line MC.
Consider a point P whose angular position is θ. Since size of slit is very small, waves from all the points on slit are
approximated parallel to each other.
To find the path difference between the waves emanated from pts. L and N, we draw a perpendicular LQ on NP.
Now, NQ is the path difference between the two waves which are emanated from L and N and is represented by
∆x.
Now, in ∆LNQ,
𝑁𝑄 ∆𝑥
sin θ = = ,
𝐿𝑁 𝑎
∆𝑥
θ≈ 𝑎 [For very small angles, sinθ≈θ]
∆x = aθ ⟶①

Central maxima: At point C, angular position θ = 0


From equ. ①
∆x = 0
i.e. the path difference between the waves striking at C is zero,
Therefore, all waves emanated from various points of slit superpose in same phase at C. Thus, intensity of light is
maximum at C.

77
Minima’s:
𝑛𝜆
Minima’s are located on screen at where n≠ 0
𝑎
To understand his observation, consider a slit in two equal parts such that for every point source in upper half,
𝑎
there is a corresponding pt. source in lower half such that these pairs are separated by a distance of 2.
𝜆
Now, for position of minima, light from these pairs must superpose destructively i.e ∆x = 2
From equ ①,
𝑎 𝜆
2
𝜃 =2
𝜆
θ=𝑎
𝑛𝜆
Similarly, all other minima’s can obtained by splitting slit into equal parts and hence generalised as θ = 𝑎
where n ≠ 0
Secondary maxima’s:
𝜆
Secondary maxima’s are located at (2n + 1)
2𝑎
To understand this observation, divide the slit into 3 equal parts, for every point source in first one-third, there
𝑎
will be a corresponding point in second one-third such that their separation is
3
𝜆
If these pairs superpose destructively i.e., ∆x = 2 then from equ. ①
𝑎 𝜆
3
θ =2
3𝜆
θ=
2𝑎
And hence this position will be illuminated by the third one-third of slit.
For higher values of n i.e, 2,3,4,… illumination will be due to one-fifth, one-seventh, one-ninth… part of the slit.
Hence, as we move away from central maxima, intensity of secondary maxima’s keeps on decreasing.
Angular size and linear size of central maxima:
Angular separation between first minima’s on either side of central maxima’s is its angular size.
2𝜆
∴ Angular size of central maxima = 𝑎
2𝜆𝐷
Linear size of central maxima = 𝑎
𝜆 𝑎𝑛𝑔𝑢𝑙𝑎𝑟 𝑠𝑖𝑧𝑒 𝑜𝑓 𝑐𝑒𝑛𝑡𝑟𝑎𝑙 𝑚𝑎𝑥𝑖𝑚𝑎
Angular size of secondary maxima = 𝑎 = 2
𝝀𝑫
Linear size of secondary maxima = 𝒂
NOTE: When the converging lens is placed after the slits and the screen is placed at the focus of it, then the
fringes are formed at a larger distance of the screen. This arrangement gives more intensity than placing the
screen far away.
𝝀
Linear size = f, where f is the focal length of
𝒂
converging lens.

Fig
27

78
3. Difference between Interference and Diffraction:
Diffraction Interference
1. Diffraction is due to the superposition of 1. Interference is due to the superposition of two
secondary wavelets from the different parts of the different wave trains coming from coherent
same wavefront. sources.
2. Width of central maxima is twice the width of 2. All bright and dark fringes are of equal width.
any secondary maxima.
3. The maxima are of varying intensities. 3. All the maxima have the same intensity.
4. There is a poor contrast between the maxima 4. There is a good contrast between the maxima
and minima. and minima.

79
Classroom Questions:
Level – 1
1. Light of wavelength 600nm falls normally on a slit of width 1.2µm producing diffraction pattern on a screen.
Calculate the angular position of the first minimum and the angular width of the central maximum.
Ans. 300, 600 [2 marks]
2. The slit of width ‘d’ is illuminated by a light of wavelength 5000𝐴̇. For what value of ‘a’ will the first
maximum fall at angle of diffraction of 300? Ans. 1.5×10-6m [2 marks]
Level – 2
1 A parallel beam of monochromatic light of wavelength 500nm falls normally on a narrow slit and the
resulting diffraction pattern is obtained on a screen 1m away. It is observed that the first minimum distance
of 2.5mm from the centre of the screen. Find (i) the width of the slit, (ii) the distance of the second
maximum from the centre of the screen, (iii) the width of the central maximum.
[Foreign 2010] Ans. 2×10-4m, 0.5×10-3m, 0.4×10-3m [3 marks]

80
Homework Sheet 10.4
Theory
1. What is diffraction of light? [1 mark]
2. Give difference between interference and diffraction of light. [2 marks]
3. Using Huygens’ principle, draw a diagram to show propagation of a wave-front originating from a
monochromatic point source. Describe diffraction of light due to a single slit. Explain formation of a
pattern of fringes obtained on the screen and plot showing variation of intensity with angle (θ) in single
slit diffraction. [Outside Delhi 2014, 2013, 2012, 2011, Compartment 2011, Delhi 2010] [5 marks]
Application Based
4. When the width of the slit is made double the original width, how would this affect the size and
intensity of the central diffraction band in diffraction pattern?
1
Ans. Angular width∝ 𝑎. Hence, if a is doubled, then size of central diffraction is halved and intensity
becomes 4 times the initial. [Compartment 2012, Outside Delhi 2012] [1 mark]
5. How does the angular separation between fringes in single-slit diffraction experiment change when the
𝜆
distance of separation between the slit a screen is doubled? Ans. No change as angular separation =
𝑑
[Outside Delhi 2012] [1 mark]
6. Why secondary maxima are less intense that the central maxima?
Ans. Central maxima receives wavelets from all parts of the slit while the secondary maxima receive
wavelets from only a part of the slit. The wavelets from the remaining part of the slit cancel each
other’s effect. [Prakash, DPS] [1 mark]
7. One of the 2 slits in Young’s double slit experiment is covered with transparent paper such that only
half of light intensity is transmitted. How will the intensity of maxima & minima be affected.
Ans. Intensity of maxima will decrease whereas that of minima will increase. [DPS] [1 mark]
8. Write one feature which distinguishes the observed pattern in single slit diffraction pattern from the
double slit interference pattern. How would the diffraction pattern of single slit be affected when: (i)
width of slit is decreased? (ii) light of smaller visible wavelength is used?
Ans. (i) Angular width increases, (ii)angular width decreases. [3 marks]
9. A parallel beam of monochromatic light falls normally on a narrow slit and the light, coming out the slit,
is obtained on the screen, kept behind, parallel to the slit plane. What kind of pattern do we observe on
the screen and why? How does the i) angular width ii) linear width of the principle maximum, in this
pattern change when the screen is moved, parallel to itself, away from the slit plane?
Ans. Pattern observed is a diffraction pattern with central maxima having maximum intensity and
corresponding maximas with decreasing intensities. (i) No change in angular width, (ii) Linear width∝ D,
hence linear width increases. [All India 2008] [3 marks]
10. In a single slit diffraction experiment, when a tiny circular obstacle is placed in the path of light from a
distant source, a bright spot is seen at the centre of the shadow of the obstacle. Explain why?
Ans. This is because the light rays flare into the shadow region of the circular object as they pass the
edge of the tiny circular object. The lights from all the edges of the tiny circular object are in phase with
each other. Thus, a bright spot is seen at the centre of the shadow of the obstacle.
[Outside Delhi 2010] [1 mark]

81
82
Session 10.5 (Fresnel Distance, Resolving Power)
1. Rayleigh’s criterion for resolution:
According to Rayleigh’s criterion, the image of
the two point objects is just resolved when the
central maximum of the diffraction pattern of
one falls over the first minimum of the diffraction
pattern of the other. Fig 28
2. Resolving Power of Optical instruments:
(i) Limit of Resolution:
The smallest linear or angular separation between two point object at which they can be just
separately seen or resolved by an optical instrument is called the limit of resolution of that
instrument.
(ii) Resolving power:
The resolving power of an instrument is its ability to resolve or separate the image of two nearby
point object so that they can be distinctly seen. Numerically, it is equal to reciprocal of the limit of
resolution of the instrument.
3. Resolving power of microscope:
The resolving power (R.P.) of the microscope is defined as reciprocal of the smallest distance between two
point object at which they can be just resolved when seen through the microscope.

1.22λ
The limit of resolution for microscope is given by d = 2μsinθ
1 2μsinθ
So, resolving power of microscope is given by RP = =
d 1.22λ
Here, λ is wavelength of light used, θ is half of the angle subtended by object on objective, µ is the refractive
index of medium between object and objective.
The product µsinθ is called numerical aperture.
So to increase resolving power of microscope, an oil of high refractive index (µ) is used between object and
objective of microscope.
Such an arrangement is called “oil immersed objective”.
4. Resolving power of telescope:
The resolving power (R.P.) of the telescope is defined as reciprocal of the smallest angular separation
between two objects whose images can be just resolved through the telescope.
1.22λ
The limit of resolution for telescope is given by Δθ = D
1 D
Thus, resolving power of telescope RP = Δθ = 1.22λ
Here, λ is wavelength of light, D is the diameter of the objective, Δθ is the angle subtended by two distant
object at the objective.
Larger the aperture of the objective and smaller the wavelength of the light used, the greater will be the
resolving power of the telescope.
5. Fresnel Distance:
A parallel beam of light of wavelength λ on passing through an aperture of size d gets diffracted into a beam
of angular width, θ= λ/d. This is the angular size of the bright central maximum.
If a screen is placed at distance D, then beam spread over a linear width, x = λD/d.
If diffraction x is less than or equal to aperture d then till this distance concept of ray optics is valid. Beyond x
greater than d ray optics concept is not valid.
The distance at which the diffraction spread of a beam is equal to the size of the aperture is called Fresnel’s
distance (ZF).
At this, x=d, D=ZF. Thus, d=ZFλ/d
⟹ZF=d2/λ.

83
If D<ZF, then there will not be too much broadening by diffraction, i.e., the light will travel along straight lines
and the concept of ray optics will be valid.
d2
As D<ZF⟹ D < λ ⟹ d >√λD
For a given value of D, the quantity √λD is called the size of Fresnel zone and is denoted by dF.
i.e., dF = √λD.
Hence the concept of ray optics can be conveniently used without introducing any appreciable error if the
size of the aperture is greater than the size of the Fresnel zone, i.e., d>dF.
7. Theories on nature of light (Not to be asked in board exams):
(i) Corpuscular theory of light: This theory was given by Newton in 1675. According to him, light
consists of tiny particles called corpuscles which are shot out at high speed by a luminous object. His
theory could explain reflection, refraction and straight line motion of light.
(ii) Wave theory of light: This theory was proposed by Christian Huygens in 1678. He suggested that
light travels in form of longitudinal waves similar to sound. His theory of luminiferous ether (a
hypothetical medium present everywhere) was discarded due to contradictory properties. However,
his theory was not accepted initially due to acceptance of Newton’s theory and also because light
travels through vacuum and it was felt that a wave always requires a medium to propagate.
(iii) However, Fresnel and Young showed that light propagates as a transverse wave by performing
interference experiment. This theory was able to explain reflection, refraction as well interference,
diffraction and polarisation of light waves. In this chapter, we are going to study this theory of light.
(iv) However, a major problem with this theory was that it could not explain travelling of a wave in
vacuum since a wave requires a medium for travelling. This was explained by Maxwell in his
electromagnetic theory of wave.
(v) Maxwell theory of Electromagnetic wave: Maxwell proposed that light propagates as electric and
magnetic field oscillations. These waves are called electromagnetic waves and require no medium to
traverse. Also, these waves are transverse in nature. In 1905, Albert Einstein used his theory to
explain photoelectric effect. Thus, it was proved that light has dual nature, i.e., it behaves as
particles as well as waves.

84
Solved Examples
Eg 13: An electron microscope uses electrons accelerated by voltage of 50kV. Taking other factors such as
numerical aperture, etc to be same, how does the resolving power of an electron microscope compare with
that of an optical microscope which uses yellow light? [Outside Delhi 2014] [2 marks]
1.227×10−9 1.227×10−9
Sol: For an electron, λ = = = 2.29×10-13m≈10-13m
√𝑉 √5×104
For visible light, λ ≈10-9m
1
Resolving power of microscope ∝ 𝜆
Since, λ is smaller for electron microscope by a factor of 104 than for optical microscope, hence, its Resolving
power is greater by a factor of 104.

85
Classroom Questions:
Level – 1
1. An electron microscope uses electrons accelerated by voltage of 50kV. Taking other factors such as
numerical aperture, etc to be same, how does the resolving power of an electron microscope compare with
that of an optical microscope which uses yellow light? [Outside Delhi 2014] [2 marks]

86
Homework Sheet 10.5
Theory
1. What is Fresnel’s distance? [1 mark]
2. Define resolving power of an optical instrument. [1 mark]
3. What is meant by the term angular resolution or resolving power of a telescope? [1 mark]

4. How are resolving power and limit of resolution of an optical instrument is related?
1
Ans. Resolving Power = 𝐿𝑖𝑚𝑖𝑡 𝑜𝑓 𝑅𝑒𝑠𝑜𝑙𝑢𝑡𝑖𝑜𝑛 [1 mark]
5. Define resolving power of microscope. [1 mark]
6. How can we increase resolving power of a microscope? [1 mark]
7. What are the factors on which the resolving power of a telescope depends? [1 mark]
Application Based
8. How does the resolving power of a compound microscope change on (i) decreasing the wavelength of
the light used, (ii) decreasing the diameter of the objective lens?
2𝜇𝑠𝑖𝑛𝜃
Ans. Resolving power of a compound microscope = 1.22𝜆 Hence, (i) R.P. increases on decreasing
wavelength, (ii) R.P. decreases on decreasing diameter of the objective lens, because the semi vertical
angle θ decreases with decreasing diameter. [Outside Delhi 2008] [2 marks]
9. How does the resolving power of telescope change if (i)size of the aperture of the objective lens is
increased, (ii) the focal length of the objective lens is decreased?
𝐷
Ans. Resolving power of telescope is given by 1.22𝜆. Hence, (i) R.P. increases, (ii) No change
[Outside Delhi 2008] [2 marks]
10. Explain with reason, how the resolving power of an astronomical telescope will change when, (i)
frequency of the incident light is increased, (ii) focal length of objective lens is increased, (iii) aperture of
the objective lens is halved.
𝐷 𝐷𝜈
Ans. Resolving power of telescope is given by = (i) R.P. increases, (ii) No change, (iii) R.P. is
1.22𝜆 1.22𝑐
halved [Outside Delhi 2002] [3 marks]

87
88
Assignment Sheet
1. Two plane monochromatic waves propagating in the same direction with amplitude A & 2A and differing
in phase by π/3 superpose. Calculate the amplitude of the resultant wave. [Nirma]
2. Which plane is defined as the plane of polarization in a polarized electromagnetic wave? [Nirman]
3. Explain the statement ‘light added to light can produce darkness’ [Nirman]
4. Find the ratio of intensities of two points P and Q on the screen in the Young’s double slit experiment
when the waves from two sources S1 and S2 have phase difference of (i) 𝜋/3 and (ii) 𝜋/2 [Shanti Asiatic
school]
5. The phase difference between two light waves emerging from the slits of young’s experiment is 𝜋
radians. Will the central fringe be bright or dark? [Nirma]
6. In young’s double experiment, the waves emerging from the two slits are in phase. Why? [Nirma]
7. What is the convincing proof of wave nature of light? [Nirma]
8. What change is observed in interference pattern in young’s double slit experiment, if one of the two slits
is painted so that it transmits half the light intensity of the other? [Nirma]
9. What will happen to the interference pattern in young’s experiment, if the source is not exactly on the
center line between the slits? [Nirma]
10. In an experiment using Young’s slits, the distance between the center of the interference pattern and
the tenth bright fringe on either side is 3.44 cm and the distance between the slit and the screen is
2.00m. If the wavelength of light used 5890 Å, determine the slit separation. [Nirma]
11. The distance between the first fringe and the 21st fringe in Young’s double slit experiment was found to
be 2.7 mm. the slit separation was 1.0 mm and the distance from the slits to the plane of the fringes was
25cm. what was the wavelength of light? [Nirma]
12. At what angle should the axes of two Polaroid be placed so as to reduce the intensity of incident
unpolarised light to 1/8? [Nirma]
13. In a double- slit- experiment the angular width of a fringe is found to be 0.2˚ on a screen place 1m away.
The wavelength of light used is 600nm. What will be the angular width of the fringe if the entire
experimental apparatus is immersed in water? Take refractive index of water to be 4/3. [DPS]
14. Light from the sodium lamp is passed to Polaroid sheets, P1 and P2 kept one after the other. Keeping P1
fixed, P2 is rotated so that its ‘pass-axis’ can be at different angles, , with respect to the pass-axis of P1.
An experimentalist records the following data for the intensity of light coming out of P2 as a function of
the angle . [I0 = Intensity of beam falling on P1]
S. No. 1 2 3 4 5
 (Angle between the pass-axis 0 30 45 60 90
of the two polaroids)
I (Intensity of light coming out of I0/2 3/8 I0 I0/22 I0/8 0
P2)
One of these observations is not in agreement with the expected theoretical variation of I. Identify this
𝐼0
observation and write the correct expression. [Outside Delhi 2008] Ans. For θ = 450, 4
[2 marks]
15. When unpolarized light passes from air to a transparent medium, under what condition does the
reflected light get polarized? Ans. When refracted and reflected light are perpendicular to each other
[Compartment 2011] [1 mark]
16. The intensity at the central maxima in Young’s double slit experiment set-up is I0. Show that the intensity
at a point where the path difference is λ/3 is I0/4. [Outside Delhi 2012] [2 marks]
17. In Young’s double slit experiment, separation between slits is 0.24mm. The screen is 1.2m away and
fringe width is 0.3cm. Calculate wavelength of light used. Ans. 6000𝐴̇ [2 marks]

89
List of Formulas
Quantity Formula Quantity Formula
Malus Law 2
I = I0cos θ Diffraction (2𝑛+1)𝜆𝐷
Bright Fringe: xn =
2𝑎
𝑛𝜆𝐷
Dark Fringe: xn = 𝑎
Brewster Law μ = tanip Resolving Power of 2𝜇𝑠𝑖𝑛𝜃
R.P. = 1.22𝜆
Microscope
Interference 𝑛𝐷𝜆 Resolving Power of 𝐷
Bright Fringe: xn = R.P. =
𝑑 1.22𝜆
(2𝑛−1)𝐷𝜆 Telescope
Dark Fringe: xn = 2𝑑
𝐷𝜆
Fringe Width: β = 𝑑

List of Derivations:
1. Brewster Law
2. Reflection & Refraction using Huygen’s Principle
3. Young’s Double Slit Experiment
4. Single Slit Experiment

90
(Space for classroom notes)

91
(Space for classroom notes)

92
93
CH-11 DUAL NATURE OF RADIATION AND MATTER
Session 11.1 (Photoelectric effect)

1. Electron Emission:
The process of emission of electrons from a metal surface is called
electron emission.There are three types of electron emission:
(i) Thermionic Emission:
The process of emission of electrons when a metal is heated is known as thermionic emission.
(ii) Field Emission:
The process of emission of free electrons when a strong electric field is applied across
the metal surface is known as field emission.
(iii) Photoelectric emission:
The process of emission of electrons when light of suitable frequency is incident on a metal
surface is known as photoelectric emission.
2. Mechanism of electron emission:
Electrons are bound to nucleus through electric forces. These forces make surface barrier for
electrons. When they are supplied energy externally sufficient to overcome this surface barrier,
they escape from the metallic surfaces.
3. Work function (Φ):
The minimum amount of energy required by an electron to just escape from metallic surface is
known as work function of metal. Work function of a material depends on its nature of losing
electron. Generally alkali metals have low 𝜑 like Cs(2.14 eV), whereas noble metals have high
values of 𝜑 like Pt(5.65 eV), lowest and highest values among metals respectively.
Note: SI unit of energy is joule but scale of energy is so low that Φ is conveniently expressed
in eV. 1eV is energy possessed by electron accelerated through potential difference of 1 V.

Conversion factor for numerical ⇒ 1J = 6.25 ×


1018eV
4. Photoelectric effect : 1eV = 1 6 × 10-19J
When light (EM radiation) of suitable frequency (depending on nature of metal) illuminates
surface, electrons on surface absorbs the energy of radiation & escape the metal surface by
overcoming nuclear attraction. This phenomenon is called photoelectric effect. Electrons
emitted in this phenomenon are called photoelectrons.
Applications of Photoelectric Effect:
1. Digital cameras and night-vision scopes convert light energy into electric signal to get image.
2. Photoelectric effect establishes particle nature of radiation. This revolutionalized the study of
physics leading it to quantum physics.

94
5. Experimental study of photoelectric effect by Hallwach and Lenard:
First observation about photoelectric effect was recorded by Hertz during his experiment of
producing electromagnetic wave. He observed that the sparks he was producing in
experiment were stronger when UVlight was used instead of ordinary light. Since this was
before discovery of electron, he couldn’t conclude this observation.
Wilhelm Hallwach and Philpp Lenard are credited for detailed study of
photoelectric effect. In this experimental setup,
Evacuated glass tube:
An evacuated glass tube (to minimize the collisions between
electrons and air molecules) is fitted with 2 plates.Plate
C(cathode) is a photosensitive emitter and Plate A(anode) is
collector plate. This tube has side window made up of
transparent Quartz through which light (radiation) is irradiated
on plate C. Through the window, the color of light(wavelength or
frequency) and amount of light (intensity) can becontrolled
using filters.
[Note: Glass doesn’t allow UV rays to pass through it. Therefore,
Quartz window is used]
External Circuit:
Plate A and C are connected to battery through a commutator
(reversible switched). Using commutator, polarity of the plates can be reversed. When A is positive w.r.t
C, potential difference is called accelerating potential where as when A is negative w.r.t C, potential
difference between the plates is called retarding voltage. These voltage values are varied using
Rheostat in circuit.
Micro-Ammeter:
When plate C is irradiated with sufficiently high frequency, emitted photoelectrons in tube
constitutes photoelectric current in external circuit which is measured by microammeter.
Experimental Observations: During this experiment four parameters were varied and their effect
on photoelectric effect was recorded.
(i) Intensity of radiations:
At higher intensities, higher photocurrent was observed. IP and intensity
had a linearrelationship.
(ii) Potential difference between C and A at different intensities of light:
When plate A is made positive w.r.t C: The potential difference is called
accelerating voltage. As this voltage is increased, photocurrent increases
non-linearly and eventuallybecomes steady. Once current saturates,
increase in accelerating potential difference doesn’t increase IP.
When plate A is made negative w.r.t C: The P.D applied is called retarding
voltage. In this polarity arrangement, plate A repels and
opposes the photoelectrons coming from C. As this voltage is
increased, photocurrent in the circuit starts to decrease. At a
specific retarding voltage, photoelectric current becomes zero
as photoelectrons are no longer flowing in circuit.

The retarding P.D at which photo current becomes zero is


known as stopping potential (V0).(graph 2)
V0 (stopping potential) does negative work on electron towards
A (qV= eV0 in this case). By work energy theorem, Kmax= eV0, we
can interpret the maximum kinetic energy possessed by
photoelectron.

95
When P.D is varied at different intensities:

As intensity is increased, saturation current increases but there


is no change in stopping voltage (thus Kmax). This clearly
implies, change in intensity can change photocurrent (no. of
photo electrons emitted) but change in intensity has no effect
whatsoever on kinetic energy with which electrons are emitted.
(iii) Potential difference between C and A at different
frequencies(colours) of light:
It is observed that there is no change in saturation current
with increase in frequency, but as frequency
increases stopping potential(V0) gets higher and
higher showing that electrons are emitted with
higher K.E at higher frequencies.
(iv) Material of emitter plate:
For a given material, photoelectric emission starts at
a certain minimum frequency of light. Below this
frequency, there is no photoelectric effect. The
minimum frequency at which photoelectric
emission starts is known as threshold frequency (ν0)
of the material.
Beyond threshold frequency, increase in frequency
shows linear increase in stopping potential (thus K.E
of electrons). For different materials, threshold
frequencies are different depending on their nature
to lose electrons (i.e. work function).

6. Laws of photoelectric emission (Conclusions of photoelectric experiments):


(i) Below threshold frequency of the material, whatsoever is the intensity of radiation, there is no
photoelectric effect.
(ii) Above threshold frequency of the material, increase in intensity of radiation results in increase of
photoelectrons emitted (photocurrent), though increase in intensity has no effect on K.E of electron
(stopping potential).
(iii) On increasing frequency of radiation, K.E (stopping potential) of photoelectrons increases, but
photocurrent is independent of change in frequency.
(iv) Photoelectric effect is instantaneous. There is no lag between irradiation of light and
photoelectricemission.

96
7. Failure of wave theory to explain observation of photoelectric emission experiment:

Wave theory prediction Actual observations


1.Intensity According to wave theory, intensity K.E of photoelectrons depends on
oflight of wave∝(Amplitude)2.Higher frequency of radiation. Increase in
intensity implies higher amplitude, intensityhas no effect on energy of
thus higher energy. So wave theory emitted electrons.
expects that athigher intensities,
photoelectrons must
be emitted at a higher energy.
2. Energy of wave depends on its Below certain minimum frequency
Frequency amplitude, thus sufficiently (calledthreshold frequency), the
of light intensebeam should cause photoelectric effect doesn’t take place
photoelectric howsoever
emission at whatsoever frequency. intense the light beam is.
3.Time delay Energy of wave is continuous and Photoelectric effect is instantaneous.
absorbed by all the electrons at its
wavefront. Hence, energy absorbed
perelectron is very small. So the
electrons will require time to acquire
enough energy to break away from
metal
surface.

8. Einstein’s photoelectric equation:


Based on Planck’s Quantum Theory, Einstein proposed all together a radical view on behavior of
radiation during photoelectric effect. According to Einstein’s Quantum theory, when radiation is
interacting with metal surface during photoelectric effect, energy of radiation is not continuous,
but it is in the form of discrete packets called quanta (for light –photon). Energy of this quanta is
directly proportional to frequency of radiation (E=h𝜈 ).
When light strikes the metal surface, each photon interacts with one electron only and
transfers its whole energy to the electron. If frequency of radiation is greater than threshold
frequency, energy of incident photon exceeds the work function, hence electron is emitted
with K.E given by the following equation:-
Kmax = hν – 𝜑
For minimum frequency, Kmax=0 (Kmax cannot be negative)
hν0 – 𝜑 = 0
v0 =
 (threshold frequency)
h
Intensity of radiation in Quantum theory: The no. of photons striking per unit time per unit area is
defined asintensity of radiation.
Expression of photoelectric effect:
According to law of conservation of energy,
Energy of incident photon = Maximum K.E. of photoelectron + Work function
h m ax 
1
mv 0
2
  , where  
0
2
1
Hence, K m ax
 2
m v m ax  h   0
2
Substituting  h , 0 0

Kmax = ½ mv2max = h   h v  h (   ) 
2 0 0

Equation ① and ② are known as Einstein’s photoelectric equations.

97
Now, if V0 is the stopping potential, then Kmax or ½ mv2 = eV0
 eV 0  h v  0
h  
 V0   v  0
e e

Comparing this equation with y = mx + c, we get slope (m) = h


and (c) = 
0

e e
V0 v/s v graph.
Thus, h = slope × e and   c  e. Hence, values of h and
0
 can be
0

determined from the V0 v/s v graph.

Explanation of photoelectric effect observations through Einstein’s equations:


(i) Intensity of radiation is a measure of photons hitting the surface per unit time per unit area. So
higher intensity means larger no. of photons hitting the surface, emitting larger no. of photoelectrons,
thus higher amount of current generated.
(ii) Kinetic energy of emitted electrons is equal to energy of photon minus the work function. Photons
of higher frequency have higher energy. Electrons emitted by such photons will have higher kinetic
energy. But clearly, intensity has no role deciding the energy of electrons. In the same way, frequency
has no role in deciding the no. of electrons emitted (photocurrent).
(iii) Since interaction between photon and electrons is elastic collision and transfer of energy and
emission of electron is instantaneous
9. Photon:
A photon is quanta of light radiation or packet of energy.
hc
Energy of a photon is E  h , where h is the Planck’s constant, v is the frequency of the radiation or

photon, c is the speed of light (e.m. wave) and λ is the wavelength.
10. Properties of photons:
8
i) A photon travels at a speed of light in vacuum. (i.e. 3 x 10 m/s)
ii) Photon does not have any charge so they are electrically neutral. So not deflected in
electric and magnetic field.
iii) Rest mass of photon is zero, i.e. the photon cannot exist at rest.
iv) The momentum of a photon is, p =E/c =h /λ.
v) Photons travel in a straight line.
vi) Energy of a photon depends upon frequency of the photon; so the energy of the photon
does not change when photon travels from one medium to another.
vii) The speed of a photon changes as it travels through different media due to
change in its wavelength.
viii) A photon collides elastically with a material particle.
E h
ix) The kinetic mass of a photon is, m   .
c
2
c

98
E
Classroom Questions : -
Level – 1
1. Does the 'stopping potential' in photoelectric emission depend upon (i) the intensity of the incident
radiation in a photocell? (ii) The frequency of the incident radiation?
Ans. (i) No, as it depends on number ofphotoelectrons, (ii) increases with increase in frequency.
[Delhi 2013] [1
mark]
2. The following graph shows the variation of stopping potential V0 with the frequency  of
the incidentradiation for two photosensitive metals X and Y:
(i) which of the metals has larger threshold wavelength? Give reason.
Ans: As threshold frequency of X is less, hence, it has more threshold
1
wavelength as 

(ii) Explain, giving reason, which metal gives out electrons,
having larger kinetic energy, for the same wavelength of the
incident radiation.
Ans: Since threshold wavelength of X is more, thus, it will
have lower threshold frequency and hence it will have higher
kinetic energy.
(iii) If the distance between the light source and metal X is
halved, how will the kinetic energy of electrons emitted from
it change? Give reason.
Ans: No change in kinetic energy as distance remains has no effect on kinetic energy.
[Outside Delhi 2008, DPS, Prakash, Udgam] [3 marks]
3. The wavelength of light in the visible region is about 390 nm for violet colour, about 550 nm (average
wavelength) for yellow-green colour and about 760 nm for red colour.
What are the energies of photons in (eV) at the (i) violet end, (ii) average wavelength, yellow-green
colour, and (iii) red end of the visible spectrum? (Take h = 6.63 × 10–34 J s ad 1 eV = 1.6 × 10–19 J.)

Level – 2
1. Two metals M1 and M2 have work functions 2eV and 4eV respectively. Which of the two metals has a
lower threshold wavelength for protoelectric emission?
hc 1
Ans. We know that, 0  h v 0  . Thus, 0  . Hence, for 2eV, threshold wavelength is more.
0

0

[Foreign 2013] [1
mark]
2. An -particle and a proton are accelerated from rest through the same potential difference V.
Find the ratio of de-Broglie wavelengths associated with them. [Outside Delhi – 2005, 2010, 2009,
2011]
Ans. 2√2:1 [2 marks]
15
3. Radiation of frequency 10 Hz are incident on two photosensitive surfaces A and B, following
observations are recorded:
Surface A: No photo-emission takes place.
Surface B: Photo-emission takes place but photo-electrons

99
have zeroenergy.
Explain the above observations on the basis of Einstein’s
photoelectric equation. How will the observation with surface
B change when the wavelength of incident radiations is
decreased?
Ans. The above observations explain that, energy of incident light is less than work function of surface
A, whereas in case of surface B, energy of incident light is equal to work function of B. If the
wavelength of incident radiation is decreased, then incident energy will increase and hence electrons
emitted from surface B will contain some energy.
[Outside Delhi 2007, DPS] [2 marks]

4. The work function of cesium metal is 2.14 eV. When light of frequency 6 ×1014Hz is incident
on the metal surface, photoemission of electrons occurs. What is the (a) maximum kinetic
energy of the emitted electrons, (b) Stopping potential, (c) maximum speed of the emitted
photoelectrons?
Ans. 0.34eV, 0.34V, 345.8kms1. [3 marks]

5. Light of wavelength 2000𝐴 ̇ on a metal surface of work functions 4.2eV. What is the kinetic
energy (in eV) ofthe fastest electrons emitted from the surface? (i) What will be the change in
the energy of the emitted electrons if the intensity of light with same wavelength is doubled? (ii)
If the same light falls on another surface of work functions 6.5eV, what will be the energy of
emitted electrons? [Foreign 2011]
Ans. 2eV, No emission in both cases [3 marks]

6. When a given photosensitive material is irradiated with light of frequency v,


the maximum speed of emittedphotoelectrons equals vmax. the square of vmax
i.e, v2 is observed to vary with v as per graph shown here. Obtain
ml
(i) Planck’s constant and Ans. 2 n
h= 6.67 × 10–34 Js
(ii) Work functions of photosensitive material in terms of parameter I,nand
mass m of the electron.
(iii) The following table gives the values of work function for a few
sensitive metals.
Sr no. Metal Work
Function
(eV)
1 Na 1.92

2 K 2.15

3 Mo 4.17

If each of these metals is exposed to radiations of wavelength 300 nm, Which of these will not
emit photoelectrons and why?

100
101
Homework Sheet 11.1
Theory
1. What is photoelectric effect? [DPS] [1 mark]
2. Write four laws of Einstein’s photo electric effect. [MAV] [2 marks]
3. Use Einstein’s photoelectric equation to define the terms (i) stopping potential and (ii) threshold frequency.
[Compartment 2013, Compartment 2011, DPS] [2 marks]
4. Write Einstein’s photoelectric equation. State clearly how this equation is obtained using the photon picture of
electromagnetic radiation. Write three salient features observed in photoelectric effect which can explainusing
this equation. [Compartment 2012, Delhi 2012, Outside Delhi 2012, Outside Delhi 2010, DPS, Udgam]
[3 marks]
5. Using Einstein’s photoelectric equation, show how the cut-off voltage and threshold frequency for a given
photosensitive material can be determined with the help of a suitable plot/graph.
[Outside Delhi 2012, Foreign 2011] [2
marks]
6. Explain briefly the reasons why wave theory of light is not able to explain the observed features in
photoelectric effect. [Foreign 2010] [2 marks]
7. Draw graphs showing the variation of photoelectric current with anode potential of a photocell for (i) the same
frequency but different intensities I1>I2>I3 of incident radiation. Explain why the saturation current is
independent of the anode potential. [2 marks]
8. Draw a plot showing the variation of photoelectric current with collector plate potential for two different
frequencies, ν1>ν2, of incident radiation having the same intensity. In which case will the stopping potential be
higher? Justify your answer. [Compartment 2011, Outside Delhi 2011, DPS] [2 marks]
9. Show graphically how the stopping potential for a given photosensitive surface varies with the frequency of
the incident radiation. [Outside Delhi 2013] [1 mark]
Application Based
10. Does the 'stopping potential' in photoelectric emission depend upon (i) the intensity of the incident radiation in
a photocell? (ii) The frequency of the incident radiation? Ans. (i) No, as it depends on number of
photoelectrons,(ii) increases with increase in frequency. [Delhi 2013] [1 mark]
11. State two important properties of photon which are used to write Einstein’s photoelectric equation. Define
(i) stopping potential and (ii) threshold frequency using Einstein’s equation and drawing ecessary plot
between relevant quantities.
12. Electrons are emitted from a photosensitive surface when it is illuminated by green light but electron emission
does not take place by yellow light. Will the electrons be emitted when the surface is illuminatedby (i) red light,
and (ii) blue light? Ans. (i) As frequency of red light is less than that of green light, thus, no electrons will be
emitted, (ii) as frequency of blue light is more than that of green light, thus, electrons will be emitted. [Delhi
2013] [1 mark]
13. Alkali metals are more suitable for photo emission. Why? Ans. This is because work function of alkali metalsis
low, thus, photo emission is easier. [1 mark]
14. If the frequency of incident light on a metal surface is doubled, will the kinetic energy of the photoelectrons be
doubled? Give reason. Ans. No, it will not be doubled but will be more than double. [DPS, Udgam] [1 mark]
15. If the intensity of the incident radiation on a photosensitive surface is doubled, how does the kinetic energy of
the emitted electrons get affected? Ans. No effect on kinetic energy as kinetic energy is independent of
intensity of light. [Foreign 2013] [1 mark]
16. Ultraviolet light is incident on two photosensitive materials having work functions W1 and W2 (W1> W2). In
which case will the kinetic energy of the emitted electrons be greater? Why? Ans. Kinetic energy will be more
in case of W2 as Kmax = hν – W. Thus, lesser is the work function, more is the kinetic energy.
[Outside Delhi 2013, DPS] [1 mark]

102
17. Do all photons have same mass? If not, why? Ans. No, because mass of photons depends on their velocity
since they do not have rest mass. [Nirman] [1
mark]
18. If the frequency of the incident radiation on a photocell is doubled for the same intensity, what changes will
you observe in (i) K.E. of photoelectrons emitted, (ii) photoelectric current, (iii) stopping potential. Justify.
[Prakash] [2 marks]
Numerical
19. The photoelectric cut-off voltage in a certain experiment is 1.5 V. What is the maximum kinetic energy of
-19
photoelectrons emitted? Ans. 2.4×10 J [2 marks]
20. Ultraviolet light of wavelength 2271𝐴 ̇ from a 100 W mercury source radiates a photo cell made of
molybdenum metal. If the stopping potential is 1.3 V, estimate the work function of the metal. How would the
photo cell respond to high intensity (105 Wm-2) red light of wavelength 6328𝐴 ̇ produced by a He - Ne laser?
[Delhi 2013]
Ans. 6.678×10-19J. [2 marks]
21. Light of wavelength 2500𝐴 ̇ falls on a metal surface of work function 3.5eV. What is the kinetic energy
(in eV0) of (i) the fastest and (ii) the slowest electron emitted from the surface? If the same light falls on
another surface of work function 5.5eV, what will be the energy of emitted electrons? [Foreign 2011]
Ans. 1.4725eV, Zero [3 marks]
22. Calculate the maximum kinetic energy of electrons emitted from photosensitive surface of work function 3.2
eV, for the incident radiation if wavelength 300 nm. Ans. 0.94375eV [2 marks]

103
Session – 11.2
(Dual Nature of Matter)

1. Dual Nature of Radiation:


Wave theory of electromagnetic radiations explained the phenomenon of interference,
diffraction and polarization. On the other hand, quantum theory of e.m. radiations successfully
explained the photoelectric effect, Compton effect, black body radiations, X- ray spectra, etc.
Thus, radiations have dual nature. , i.e., wave and particle nature.
Louis de Broglie suggested that the particles like electrons, protons, neutrons, etc have also dual
nature, i.e.,they also can have particle as well as wave nature.
2. De-Broglie’s theory was based on following considerations:
i) The two main physical quantities of universe are mass and energy. By Einstein’s
mass relation equivalence, we see that mass and energy are in complete
equivalence. Thus, there is mutual symmetry between mass and radiation.
ii) From symmetry considerations, de-Broglie predicted that matter also possess dual nature.
De Broglie’s waves: The waves associated with material particles in motion are called matter
or de Brogliewaves and their corresponding wavelengths are called de Broglie wavelengths.
hc
According to quantum theory, the energy of the photon is E  h 

According to Einstein’s theory, the energy of the photon is E = mc2.
h h
So,    , where p = mc is momentum of a photon.
mc p
If instead of a photon, we have a material particle of mass m moving with velocity v, then the equation
h
becomes  which is the expression for de Broglie wavelength.
mv
3. Conclusions of de Broglie’s equation:
a. De Broglie wavelength is inversely proportional to the velocity of the particle. If the particle
moves faster,then the wavelength will be smaller and vice versa.
b. If the particle is at rest, then the de Broglie wavelength is infinite. Such a wave cannot be
visualized.
c. De Broglie wavelength is inversely proportional to the mass of the particle. The wavelength
associatedwith a heavier particle is smaller than that with a lighter particle.
Note: Matter waves are not electromagnetic waves because they are not produced by
accelerated charges. Matter waves are probability waves, amplitude of which gives the
probability of existence of the particle at the point.
For macroscopic particles, λ is very small. Thus, they do not show wave-like properties.

104
4. De Broglie wavelength of electron:
Consider an electron (mass m, charge e) accelerated from rest through a potential V.

The kinetic energy K of the electron equals the work done (eV0) on it by the electric field : K =
eV.
2
p
Now, K = ½ m v
2
 , so that p  2 m K  2 m e V.
2m
The de Broglie wavelength  of the electron is then   h  h

h
.
p 2mk 2 m eV

Substituting the numerical values of h, m, e we get   1 .2 2 7 nm, here V is the magnitude of


V
accelerating potential in volts.
For a 120 V accelerating potential, we get   0 .1 1 2 nm. This wavelength is of the same order
as the spacing between the atomic planes in crystals. This suggests that matter waves
associated with an electron could be verified by crystal diffraction experiments analogous to
X-ray diffraction.

Heisenberg Uncertainty principle:


The matter–wave picture elegantly incorporated the
Heisenberg’s uncertainty principle. According to the principle, it
is not possible to measure both the position and momentum of
an electron (or any otherparticle) at the same time exactly.
There is always some uncertainty
(Δ x) in the specification of position and some uncertainty (Δp)
in thespecification of momentum.
The product of Δx and Δp is of the order of h/2π, i.e.,
Δx Δp ≈ h/2π. Now, if an electron has a definite momentum p,
(i.e. Δ p = 0), by the de Broglie relation, it has a definite
wavelength λ. A wave of definite (single)wavelength extends all
over space. By Born’s probability interpretation this means that
the electron is not localised in any finite region of space. That is,
its position uncertainty is infinite (Δx → ∞), which is consistent
with the uncertainty principle (Fig 7).
i) In general, the matter wave associated with the electron is
not extended all over space. It is a wavepacket extending over
some finite region of space (Fig 8).

105
Davisson and Germer Experiment:(Verification of De-Broglie Hypothesis)
Davisson and Germer used the diffraction effect of electron radiation scattered by crystals. The
experimentset up used is shown in figure. Important
parts of setup are:
(i) Electron gun: it consists of a tungsten filament F
coated with barium oxide to decrease the work
function.This filament act as an emitter. It is
heated by low tension (L.T.) battery to give
thermionic emission. The emitted electrons are
accelerated with the help of high tension (H.T.)
source. A guided cylinder with a hole arranges
these electrons into a collimated beam.
(ii) Nickel crystal: the beam of accelerated electrons
is incident on the nickel crystal. Atom of crystal
scatters thebeam of electron in all directions.
(iii) Detector or collector: it can be moved on a
circular scale.It is further connected to a sensitive
galvanometer to detect current, which is
proportional to intensity of the scattered
electron beam.

Working:
A collimated beam of electron accelerated at desired velocityis made to fall on nickel crystal. Electron are
scattered in all direction are received by the detector which can be positioned at any angle by rotating it on
circular scale.The energy of incident electron can also be varied by changing the applied to the electron gun.
The intensity of scattered beam of electrons is measured as as the function of the angle of scattering.
According to classical physics, the intensity of scattered beam of electron at all scattering angle will be
same.

But Davisson and Germer found that the intensity of scattered beam of electron was not the same but
different at different angle of scattering and different voltage. As shown in graph:

So at accelerated voltage 54V, the intensity of scattered beam was maximum at scattering angle ϴ= 50°
and a strong peak was obtained in this case. Using Bragg’s equation for maxima in diffraction pattern they
found that wavelength ofelectron λ= 1.65 Å .
According to de-Broglie hypothesis, the wavelength of electron accelerated through a potential difference V
volt is given by,
12 .27
12. Å = Å =1.67Å.
27 √54
λ=
√V

Above results are close. So theoretical and experimental values are equal, hence Davisson-Germer
experiment proves De- Broglie hypothesis of wave nature of moving particle.

106
Class Room Questions : -
Level – 1
1. A proton and electron have same velocity. Which one has greater de-Broglie wavelength and why?
Ans.   h  h
. Since velocity of both of them is same, thus, electron having lower mass will have
p mv

higher wavelength. [Outside Delhi 2012] [1 mark]


2. An electron is accelerated through a potential difference of 100 volts. What is the de-Broglie wavelength
associated with it? To which part of electromagnetic spectrum does this value of wavelength
correspond?[Delhi 2010] Ans. 1.227𝐴 ̇ [1 mark]

Level – 2
1. An electron, an   particle, and a proton have the same kinetic energy. Which of these particles has the
shortest de Broglie wavelength?
h h 1
Ans. We know that,    . As kinetic energy is same for all of them, thus,   . Since, mass
p 2mE m

of   particle is highest, thus it has the shortest de Broglie wavelength. [Outside Delhi 2007] [1 mark]
2. An   particle ad a proton are accelerated from rest through the same potential difference V. Find the
ratio of de-Broglie wavelengths associated with them. [Outside Delhi – 2005, 2010, 2009, 2011]
Ans. 2 2 :1 [2 marks]
3. An electromagnetic wave of wavelength  is incident on a photosensitive surface of negligible work
function. If the photoelectrons emitted from this surface have de-Broglie wavelength  , then
1

 2mc  2 [2 marks]
   1 .
 h 
4. The ratio, between the de-Broglie wavelengths, associated with protons, accelerated through a potential
of 512 V and   particle, accelerated through a potential of x volts, is found to be one. Find the value of
x. [Delhi 2011] Ans. 64 V [1 mark]
–10
5. For what kinetic energy a neutron, will the associated de-Broglie wavelength be 1.32 × 10 m?
–21
[Outside Delhi 2008] Ans. 7.5 × 10 J [1 mark]
6. Monochromatic light of frequency 6.0 × 10 Hz is produced by a laser. The power emitted is 2.0 × 10–3
14

W. (a) what is the energy of a photon in the light beam? (b) How many photons per second, on a an
-19 15
average, are emitted by the source? Ans. 3.98 × 10 J, 5 × 10 photons per second [2 marks]
–10
7. (a) For what kinetic energy of a neutron will be associated de Broglie wavelength be 1.40 × 10
m? (b) Also find the de Broglie wavelength of a neutron, in thermal equilibrium with matter,
having an average kinetic energy of (3/2) k T at 300 K. Ans. 0.146 m
8. What is the de Broglie wavelength of a nitrogen molecule in air at 300 K? Assume that the
molecule is moving with the root-mean-square speed of molecules at this temperature. (Atomic
mass of nitrogen = 14 u) Ans. 0.028 nm

107
Homework Sheet 11.2
Theory
1. Write the relationship of de-Broglie wavelength  associated with particle of mass m in terms of its
kinetic energy E. [Foreign 2011] [1
mark]
2. Draw a schematic diagram of the experimental arrangement used by Davisson and Germer to establish
wave nature of electrons and explain it. Explain briefly ho de Broglie relation was experimentally verified
in the case of electron. [DPS, Delhi 2010]
[5 marks]
Application Based
3. A proton and an  particle are accelerated through the same potential difference. Which one of the two
has (iv) greater de-Broglie wavelength, and (ii) less kinetic energy? Justify your answer. [All india 2016]
[2 marks]
4. De-Broglie wavelength associated with an electron accelerated through a potential difference V is  .
What will be its wavelength wen the accelerating potential is increased to 4V? [Delhi 2006]
[1 mark]
5. Show that the wavelength of electromagnetic radiation is equal to the de Broglie wavelength of its
quantum (photon).
6. Draw a plot showing the variation of the De-Broglie wavelength of electron as a function of K.E. [1
mark]
Numerical
7. An electron microscope uses electrons accelerated by a voltage of 50kV. Determine the de-Broglie
wavelength associated with the electrons. [Outside Delhi 2014] Ans. 54.87pm [2 marks]
8. What is the de Broglie wavelength associated with (a) an electron moving with a speed of 5.4×106 m/s,
and
-34
(b) a ball of mass 150 g travelling at 30.0 m/s? Ans. 0.135nm, 1.47×10 m. [2 marks]
9. For what kinetic energy of a neutron will the associated de-Broglie wavelength be 140 × 1010 cm?
Ans. 4.219 × 102 eV [2
marks]
10. Find the (a) maximum frequency, and (b) minimum wavelength of X-rays produced by 30 kV electrons.
Ans. 7.24 × 1018Hz, 0.0414 nm. [2 marks]
11. What is the (a) Momentum, (b) Speed, and (c) de Broglie wavelength of an electron with kinetic energy
–24 6
of 120 eV. Ans. 5.91 × 10 kgm/s, 6.496 × 10 m/s, 0.112 nm.
[2 marks]
12. Calculate the (a) momentum, and (b) de Broglie wavelength of the electrons accelerated through a
–24
potential difference of 56 V. Ans. 4.04 × 10 kg m/s, 0.1639 nm
[2 marks]
13. The wavelength of light from the spectral emission line of sodium is 589 m. Find the kinetic energy at
which (a) an electron, and (b) a neutron, would have the same de Broglie wavelength.
[2 marks]
Ans. 4.31  eV , 2.360 eV
14. What is the de Broglie wavelength of [2 marks]
(a) a bullet of mass 0.040 kg travelling at the speed of 1.0 km/s,
(b) a ball of mass 0.060 kg moving at a speed of 1.0 m/s, and
(c) a dust particle of mass 1.0 × 10–9 kg drifting with a speed of 2.2 m/s?
Ans. 1.655 × 10–35 m, 1.1 × 10–32 m, 3 × 10–23 m
15. An electron and a photon each have a wavelength of 1.00 nm. Find [2
marks]
(a) their momenta.
(b) the energy of the photon, and

108
(c) the kinetic energy of electron.
Ans. 6.62 × 10–25 kg m/s, 1.243 KeV, 1.51 eV

109
Derivation List :
Einstein’s photoelectric equation

Units and Dimensions


Physical Quantity SI Unit Scalar/Vector Dimension
Work Function Joule Scalar [M1L2T–2]

Formula List
Physical Quantity Formula
Einstein’s photoelectric k m ax  hv  1
m v m ax
2
 h(v  v 0 )
equation 2

Energy of photon E  hv 
hc

De-Broglie wavelength h h
 
mv p
Kinetic Mass of a E
m 
Photon c
2

Momentum of a Photon E h
p  
c 
De-Broglie wavelength 
h
2M K
De-Broglie wavelength 
h

1 2 .2 7
A 
for Electron 2 M eV V

Work Function hc
  hv  0
 0

Number of photons P
n 
E

110
(Space for classroom notes)

111
Chapter - 12 (Atoms)

112
8. Hans Geiger and Ernest Marsden experiment of a particles:
In 1909, under the instruction of Rutherford, Geiger and Marsden performed gold foil experiment. They fired a
beam of a particle (radiated by Radium) on a gold foil (a few atoms thick).
If the plum pudding model of atom were correct, coulombic repulsion on a particles would have been
extremely small and they would've pierced the gold and r
\ .iliCUU.Tn

travelled almost undeviated. That happened to most of a


particles. However, not satisfied with this observation,
Rutherford made a hunch and instructed Geiger and
Marsden to repeat the experiment and observe a particles •
Sow.rceof
in other directions too. Disregarding initial disappointment, (l•p;IJUcklll

the experiment was conducted again and to their great


surprise they found out that 1 out of 8000 a particles were
°
getting deflected by more than 90 , while the rest travelled
Abool I lfl 8000
straight with almost no deviation. I rdb:tcd cl!;
Rutherford compared this large deflection to a bullet shot Fig. 2
on tissue paper bouncing off the surface. He did his calculations and eventually in 1911, he gave the nuclear
model of atom.

.\

] 10'
',� ----- -
E
Scattertng angle 6 (in degree)
z 10
0 20 40 60 80 100 120 140 160 180

Graph between number of scattered a particle and scattering angle 0 is shown fig 3

9. Impact parameter (b):


The perpendicular distance of the velocity vector ( v ) of the a-particle
from the centre of the nucleus when it is far away from the nucleus is
known as impact parameter. It is given as
b= ze2 cot(O 12) ⇒ b oc
(½mv2 j
cot(0 I 2)
4m, 0

For large b, a particles will go undeviated and for small b the a-particle will suffer
large scattering.

a-particle

(energy E)

113
10. Distance of closest approach:
When an a-particle with certain velocity approaches nucleus of an atom, then it experiences Columbic
repulsions and its kinetic energy is converted into electrostatic potential energy. At a certain distance ro from
the nucleus , the a-particle stops for a moment and retraces its path i.e. it is scattered through an angle of
°
180 . The distance ro is known as distance of closest approach.
At ro, the entire kinetic energy is converted into electric potential energy.
At the distance of closest approach:
(i) scattering angle is 180°.
(ii) kinetic energy is converted in potential energy.
(iii) impact parameter is zero.
(iv) distance of closest approach is inversely proportional to kinetic energy of a- particle.
Expression for ro:

In the given experiment, let q1 be the charge of a-particle and q 2 be the charge on nucleus of atom
(Atomic number= Z). Hence, q1= 2e and q 2= Ze.
· · kq1q2 k(2e)(Ze) k(2Z)e 2
Potential energy at distance ro, U = -- = -----'---'- = --
To To To
At distance of closest approach (ro), Ka = U (By conservation of energy)
_ k(2Z)e 2
Hence, Ka--­
To
2kZe 2 4kZe 2
==>ro= -- =-- mv2
Ka
In Rutherford's experiment, Ka=S.SMeV and Z= 79 for gold. Thus, ro= 41.3fm which is larger than radius of
gold nucleus (6 fm).

11. Rutherford's Nuclear Model of Atom:


(i)Almost all the mass and positive charge of an atom is concentrated in a very small volume called nucleus.
(ii) Size of nucleus is about 10-5 times the size of atom.
(iii) Most of the atom (99.99%) is empty. Negatively charged electrons revolve around nucleus around
coulombic attraction (acting as centripetal force).
Note:
a-particle is 1He 2+ Mass= 4u Charge= +2e

Rutherford won Nobel prize for Radioactivity Investigations in 1908

He was also called Father of Nuclear Physics

Element 104 named after him in periodic table-Rutherfordium.

12. Drawbacks of Rutherford's atomic model:


(1) Stability of atom:
It could not explain stability of atom because according to classical electrodynamics theory an accelerated
charged particle should continuously radiate energy. Thus an electron moving in a circular path around the
nucleus should also radiate energy and thus move into smaller and smaller orbits of gradually decreasing
radius and it should ultimately fall ir

Instability of atom

(2) According to this model the spectrum of atom must be continuous whereas practically it is a line spectrum.
(3) It did not explain the distribution of electrons outside the nucleus.

114
13. Bohr's atomic model:
In 1913, Neils Bohr, a student of Rutherford, with his help corrected the atomic model further using quantum
concepts. This model could specifically explain the stability of atom as well as atomic spectrum.
Postulates of Bohr's Atomic Model:
(i)Energy level:
Electron revolves around the nucleus in a circular path without losing or gaining energy during this process.
These circular paths were called Energy levels or Stationary orbits.
(ii)Quantization of Angular Momentum:
Only those energy levels are allowed in the atom which angular momentum of electron remains integral
multiple of h/2rr.
h
Ln= mvnrn= n - ,
2rr

where n = 1, 2, 3, ..... each value of n corresponds to a permitted value of the orbit radius.
rn = Radius of d h orbit, Vn = corresponding speed of electron.
(iii) Transition of electrons:
Whenever electron makes a transition from one energy level to another, it absorbs or emits the photon of
energy equal to difference of energies of the 2 levels which it transited.
fJE = E;- Et [if E;>Et photon is emitted]
hv= E;- Et [if E;<Et photon is absorbed]

Distance r= radius of nth orbit


Charge on electron=e
Nm 2
Coulomb's constant= Kc=9 x 10 9
c2

Charge of nucleus=e

14. Hydrogen atom in Bohr's Model:-


1 n hydrogen, Electrostatic force between Electron and Nucleus in nth orbit can be given as

Fn= K}e)(2 e)
rn
This force provides required Centripetal force.Hence,
2
e
Kc2
Tn
2
e
¢ rn= Kc-- or Vn =
mvn2 n

Using Bohr's second postulate of quantisation of Angular momentum.


nh
m.vnrn=
2rr
On substituting values of rn in above equation
Kc e2 nh
mvn(--z) =
mvn 2rr

_ Kc 2 2rr _.!_ 9x109 (l.6x10- 1 9 2 x2 x3.14


Vn- e - ( ) )
nh n 6.63x10- 34
2.19 X 10 6 _1 C
:---ms-- / [-]
n n 137

115
Similarly

K e2 Kc e 2n2
rn=_c_ = -----
m v� m (2.18 x 106)2

n
Kinetic energy:-
K e2 K e2
Kn - ! m vn2 - ! m c - ! c 1
En = - Un - Kn
-2 - 2 m rn - 2 rn 2
=

Potential energy:-
_ Kc e(-e) = _ Kc e 2 Kn = - En
Un
rn Tn
Total energy :-
Kc e 2 - Kc e 2 =! Kc e 2
En=Kn+Un = !
2 rn Tn 2 rn
__ � 9x109 (1.6x10- 19 ) -2.rnx10- 18 = -2.rnx10- 18
2

- J eV
2 (0.529X10-10)n2 n2 n 2 .( 1.6x10- 19 )

OR

Hydrogen atom in Bohr's Model :-


(i) Radius of orbit: For an electron around a stationary nucleus the electrostatics force of attraction provides the
necessary centripetal force

i.e. _1 _ (Z�)e = mv2


.... (i)
4w 0 r r

nh
also mvr = - .... (ii)
2:rr

From equation (i) and (ii) substitute value v from (ii) to (i) then radius of n th orbit

n 2h2 2 h2 2 l
rn = =� = 0.53 �
Z
A (k = --)
2
4:rr kZme2 mnZe 2 4w 0
2
n
⇒r. oc-
n Z
(ii) Speed of electron: From equation (i) and (ii) substitute value r from (ii) to (i) then speed of electron innth orbit

speed of electron in n th orbit can be calculated as

=
2:rrkZe2
=
Ze2
=(
c z
) = 2.2x 10 6 nm/ sec z
Vn -------,;;;- 260 nh 137 n

where (c = speed of light 3 x 10 8m/s)

Energy
(i) Potential energy: An electron possesses some potential energy because it is found in the field of nucleus
= (Ze)(-e ) = - kZe2
potential energy of electron in n th orbit of radius rn is given by U k.
rn rn

116
(ii) Kinetic energy: Electron posses kinetic energy because of it's motion. Closer orbits have greater kinetic
energy than outer ones.

mil = k. (Z (
As we know � e)
rn rn

kZd- = �
⇒ Kinetic energy K =
2rn 2

(iii) Total energy: Total energy (f) is the sum of potential energy and kinetic energy i.e.E= K + U

⇒E=--- also rn =--i. [for derivation use equation (i)and (ii) to replace value ofrnl
kZ d- n 2 h2 c
2rn mnze

me4 z2 =- --me4 z2
Hence E=- ( --2 ) - ( Jch 2
-
s/5h n2 c c
8 5 fr n

2 2
z z
=-Rc h-=
2
-13.6-eV
2
n n

me4
where R= � = Rydberg's constant = 1.09 x 10 7per m.
8c0 crr

Note:- Negative sign of total energy indicates that electron is bound to


atom. When energy of electron is either zero or positive, it implies
electron has escaped the atom.

15. De-Broglie's explanation of Bohr's second postulate:-


For a standing wave to sustain to sustain on circular path, 2m(circumference) must be equal to nA. Now
according to De-Broglie's matter wave hypothesis, motion of electron in an atom can be treated as wave
motion who's wavelength '"A'.
A= !!:.
p mvn
=-h-

Applying the stationary wave condition


h
21rr n= nA = n--
mvn
nh
mv nr n=
2rr
nh
Ln= , which is Bohr's 2 nd postulate.
2rr

Fig. 6

117
16. Rydberg's formula:-
On 5th November 1888,Swedish physicist Johannes Rydberg, presented a formula based on his studies of
spectral lines of hydrogen atom and Balmer's equation which could describe spectral lines. Balmer, a school
teacher, in 1885 first observed series of visible wavelengths. Now which are called as Balmer series.
Rydberg developed an empirical formula which could give all the wavelength(visible, ultraviolet , infrared) of
hydrogen spectrum.
1 V 1 1
-;- = - = R [2 - 2] , where Rydberg constant R= 1.097 x 107 m- 1
,._ c n; n
1
1. Lyman Series:

1 1 1
'i = R [12 - nj] nt= 2,3,4,.... These A were in UV region

2. Balmer Series:

½ = R[ \ - : ]nt= 3,4,5,.... These A were in Visible region


2 1

3. Paschen Series:
1
½ = R[ 2 - : ]nt= 4,5,6,...These A were in Infrared region
3 1

4. Brackett Series:

½ = R[:2 - : ]nt= 5,6,7,...These A were in Infrared region


1

5. Pfund Series:

½ = R[:2 - : ]nt=6,7,8,...These A were in Infrared region


7

).
?
C
1.1
[l l]
For Higher to Lower energy !eve I - - - - R. :-i - --
Df Di

J v [l l]
For Lower to Higher ene rgv !eve I - "" - .. R :-1 - =,-
;. C Dl Dj

118
17. Energy level diagram and Spectral lines/ emission spectrum of hydrogen atom:
According to the third postulate of Bohr's model, when an atom makes a transition from the higher energy state
with quantum number n i to the lower energy state with quantum number n1 (n1 <n i ), the difference of energy is
carried away by a photon of frequency v if such that

....
We know that, energy of electron at a given state is given by,

E n= -13.6
n2
eV • 0
-0.85
-l,5 .r1 •3
Thus, difference between two energy states is equal to, ..____..,
l'm•cl""rl
sc:rl.,,,.

■ 2
..:i.40f-""-"- -�.L.L-'L-'-''---�-!11_es____
tiE = Ei - Er
.,.--+

= -13.6 _ (-13.6)ev= 13_6(2_ _ 2_)


nf n} n} nf

For electron transition

from first excited state to ground state or (2 nd to i st orbit)

6 6
tiE = E2- E1 = -�:- - (-::· ) = -3.4 + 13.6 = 10.2eV n•
_t.i�
·-13.6 ,__--;!-':===:;;==:=------G_m_un_d_,. _t _
Fig. 7
from second excited to ground state,(3 rd to l51 orbit)

6 6
tiE = E2- E1 = -�:- - (-::· ) = -1.5 + 13.6 = 12.leV

For electron from third excited state to ground state,

6 6
tiE = E2- E1 = -::· - (-::· ) = -0.85 + 13.6 = 12.75eV

18. Some important terms for energy levels:


(i) Excitation Energy:
It is defined as the energy required by its electron to jump from the ground state to any one of the
excited states.
First excitation energy of hydrogen, = E2- E1 = -3.4 - (-13.6) = 10.2eV
Second excitation energy of hydrogen, = E3- E1 = -1.5 - (-13.6) = 12.leV
(ii) Excitation Potential:
It is that accelerating potential which gives an electron in ground state, sufficient energy to excite
the electron to a certain higher state.
First excitation potential of hydrogen, = E2- E1 = -3.4 - (-13.6) = 10.2V
Second excitation potential of hydrogen, = Er E1 = -1.5 - (-13.6) = 12.1V
(iii) Ionisation Energy:
It is defined as the energy required to knock an electron completely out of the atom, i.e., energy
required to take an electron from ground state to outermost orbit(n=oo).
Ionisation energy of hydrogen = E 00 - E1 = 0 - (-13.6) = 13.6eV
(iv) Ionisation Potential:
It is that accelerating potential which gives electron sufficient energy to knockout electron in ground
state completely out of atom.
Ionisation potential of hydrogen = E00 - E1 = 0 - (-13.6) = 13.6V

119
19. Limitations of Bohr's Theory:
(i) It is valid only for one electron atoms, e.g. :H, He +, u+2, Na+letc.
(ii) Orbits were taken as circular but according to Sommerfield these are elliptical.
(iii) Intensity of spectral lines could not be explained.
(iv) Nucleus was taken as stationary but it also rotates on its own axis.
(v) It could not be explained the minute structure in spectrum line.
(vi) This does not explain the Zeeman effect (splitting up of spectral lines in magnetic field) and Stark effect
(splitting up in electric field)

Solved Examples
Eg 1: A12.75 eV electron beam is used to bombard gaseous hydrogen at room temperature. What series of
wavelengths will be emitted?
Sol: Here �E = 12. 75eV
-0.85 ev·
4
- 1.51 eV
3
Energy of electron in nth orbit of hydrogen atom is
En= _ 13.6 eV
-3.4eV
2

n2
In ground state, n= 1,
1
E1= - :/ = -13.6 eV
Energy of electron in the excited state after absorbing of12.75 eV
energy becomes
En= - 13.6+12.45=-0.85 eV ,r
13 6 13 6 ,,_ -13.6eV

n2=- . = - . =16 or n= 4
En -0 85
.
thus the electron gets excited to n=4 state.
n(n-1) 4x3
. .
Tota I number of wave I engths .in em1ss1on spectrum= - - = =6
2 -2-
The possible emission lines are shown in fig.
Emitted wavelength,
· he 6.6x10- 34 x 3x108 19.8x10-26
A if= m
E;-EJ E;-EJ E;-EJ
19.8xio-26 19.8x10-7
A 43= = = 28.409 X10-7= 28409A
(-0.85+1.51)x1.6x10- 19 0.6 6x1.6
19'8xl0-26 19'8xio-?
A 42= = 4.8529x10-7= 4852.9A
(-0.85+3.4)x1.6x10- 19 2.5 5*1.6
19.8xio-26 19.8xio-7
A 41= = 0.9706 X10-7= 970.6A
(-0.85+13.6)x1.6x10- 19 12.75x1.6
19.8x10-26 19.8xio-7
A 32= = 6 .5476 X10-7= 6547 .6A
(-1.51+3.4)x1.6x10- 19 1.89x1.6
19.8xio-26 19.8xio-7
A 31= = 1.0236 X10-7= 1023.6A
(-1.51+13.6)x1.6x10- 19 12.09x1.6
- 19.8x10-26 _19.8x10-7 .
A 21- 1 2132 x10-7=1213.2A
(-3.4+13.6)x1.6x10- 19 - 10.2x1.6

For rl h energy level principle quantum number is n

For nth excited state principle quantum number is n + 1

120
Classroom Questions : -

Level-1
1. In the Rutherford scattering experiment, the distance of the closest approach for an a-particle is do. If
a-particle is replaced by a proton, how much kinetic energy in comparison to a-particle will it require
to have the same distance of closest approach do? [Foreign 2009] Ans. Half
[1 mark]
2. When an electron in H atom jumps from 3 exited state to ground state, how would the de Broglie
rd

wavelength associated with electron changes? [Delhi 2015] [2 marks]


3. In a Geiger-Marsden experiment, calculate the distance of closest approach to the nucleus of Z=80,
when an a-particle of 8 MeV energy impinges on it before it comes momentarily to rest and reverses
its direction. How will the distance of closest approach be affected when the kinetic energy of the a-
particle is doubled? [Outside Delhi 2012] Ans. 28.8fm, 14.4fm
[3 marks]
4. Use Rydberg formula to determine the wavelength of Ha line.(Given Rydberg constant, R= 1.03
x10 7 m-1) [CBSE 2015] Ans. 699nm [2 marks]
5. The ground state energy of hydrogen atom is - 13.6 eV. What are the kinetic and potential energies of
the electron in this state? [Compartment 2011, Outside Delhi 2011, Outside Delhi 2010, DPS, MAV]
Ans. 13.6eV, -27.2eV [1 mark]
6. The ground state energy of hydrogen atom is -13.6 eV. i.) What is the kinetic energy of an electron in
the 2 nd excited state? ii.) If the electron jumps to the ground state from the second excited state,
calculate the wavelength of photon emitted. [Outside Delhi 2008, 2011] Ans. l.5eV, 1030A
[2 marks]
7. What is the shortest wavelength present in the Paschen series of spectral lines?
Ans. 8201A [2 marks]

Level-2
1. Using the postulates of Bohr's model of hydrogen atom, obtain an expression for the frequency of
radiation emitted when atom make a transition from the higher energy state with quantum number n;

to the lower energy state with quantum number nt (nt<n;). Ans. v = 3.288x10 15 [� - �]
nf ni
[Foreign 2011, MAV] [2 marks]
2. In the ground state of hydrogen atom, its Bohr radius is given as 5.3x10-11m. The atom is excited such
that the radius becomes 21.2x10-11m. Find (i) the value of the principal quantum number (n) and (ii)
the total energy of the atom in this excited state. [Compartment 2013] Ans. 2, -3.4eV [2 marks]
3. The energy levels of a hypothetical atom are as shown below. Which of the shown transitions will
result in the emission of a photon of wavelength 275 nm? Which ----------ocv
of this transition corresponds to emission of radiation of (i)
maximum and (ii) minimum wavelength? Ans. transition B, (i) A
Al
----,..-.....--..----,�v

(ii) D
[3 marks]
[Delhi 2009, Compartment 2011, Prakash]
---
8
--'..,.l-....... _--·4.5eV

b
4. Find the ratio of energies of photons produced due to transition
-----------!Ocll
of an electron of hydrogen atom from its: (i) second permitted
energy level to the first level, and (ii) the highest permitted
Fig. a
energy level to the first permitted level. [Outside Delhi 2010, MAV] Ans. 3:4
[2 marks]

121
Homework Sheet

Theory
1. State the postulates of Bohr's theory of hydrogen atom. [Outside Delhi 2010] [1 mark]
2. Define ionization energy. What is its value for a hydrogen atom? [Delhi 2010] [1 mark]
3. Write the expression for Bohr's radius in hydrogen atom. [1 mark]
4. What is the significance of total negative energy possessed by the electron?
[Outside Delhi 2014] [1 mark]
5. State Bohr's postulate to define stable orbits in hydrogen atom. How does de Broglie's hypothesis
explain the stability of these orbits? [Board 2018] [2 marks]
6. Draw a schematic arrangement of the Geiger-Marsden experiment for studying a-particle scattering by a
thin foil of gold. Give Rutherford's observations and discuss the significance of this experiment. Obtain
the expression which help us to get an idea of the size of nucleus, using these observations
[Foreign 2010, Board 2019] [S marks]
7. State the basic assumption of the Rutherford model of the atom. Explain, in brief why this model cannot
account for the stability of an atom? [Delhi 2010] [3 marks]
8. (a) In an experiment on a - particle scattering by a thin foil, draw a plot showing the number of particles
scattered verses the scattering angle 0.
(b) why is it that a very small fraction of the particle is scattered at 0 > 90 °.
(c) write two important conclusion that can be drawn regarding the structure of the atom from the study
of this experiment. [3 marks]
Derivations
9. Using Bohr's second postulate of quantization of orbital angular momentum show that the
circumference of the electron in the nth orbital state in hydrogen atom is n times the de Broglie
wavelength associated with it. [Compartment 2012, Delhi 2012, Outside Delhi 2012] [2 marks]
10. Derive the expression for radius of the ground state orbit of hydrogen atom, using Bohr's postulates.
[DPS,MAV] [2 marks]
11. Using Bohr's postulates for hydrogen atom, show that the total energy (E) of the electron in the
stationary states can be expressed as the sum of kinetic energy (K) and potential energy (U), where K=­
U/2 . Hence deduce the expression for the total energy in the nth energy level of hydrogen atom.
[Outside Delhi 2012, Outside Delhi 2014, Prakash, MAV] [3 marks]
Application Based
12. When is the Ha line of the Balmer series in the emission spectrum of hydrogen atom is obtained?
Ans. 3 rd to 2 nd orbit transition. [2 marks]
13. In hydrogen atom, if the electron is replaced by a particle which is 200 times heavier but has the same
charge, how would its radius change?
Ans. Radius is inversely proportional to mass of electron so will become � time
2 0
[Foreign 2008] [1 mark]
14. What is the energy possessed by an e for n= oo? Ans. zero [1 mark]
15. The short wavelength limits of the Lyman, Paschen and Balmer Series, in the hydrogen spectrum, are
denoted by AL, Ap and As respectively. Arrange these wavelengths in increasing order.
Ans. AL<As<Ap [1 mark]
16. What is the angle of scattering for zero impact parameter? Ans. 180° [1 mark]
17. When is Ha line of Balmer series in the emission spectrum of hydrogen atom obtained?
Ans. When electron Jump from 3 rd to 2nd orbit [Compartment 2013] [1 mark]
Numerical
18. An electron is in nth orbit of hydrogen atom. An electromagnetic wave of wavelength 90nm is used to
ionize the atom. If the kinetic energy of ejected electron is 10.4eV, calculate the value of n.
Ans. n=2 [2 marks]
19. A hydrogen atom initially in the ground state absorbs a photon which excites it to the n = 4 level.
Estimate the frequency of the photon. [Board 2018] [2 marks]

122
20. What is the maximum possible number of spectral lines observed when the hydrogen atom is in its 2 nd
excited state? Justify your answer. Calculate the ratio of the maximum and minimum wavelengths of the
radiation emitted in this process. [Outside Delhi 2010] Ans. 3, 32:5 [3 marks]
21. Calculate the frequency of the photon, which can excite the electron to first excited state from ground
state. [DPS] Ans. 2.46x10 15 Hz [1 mark]
22. The electron in hydrogen atom is initially in the third excited state. What is the maximum number of
spectral lines which can be emitted when it finally moves to the ground state?
[Compartment 2012, Delhi 2012, Outside Delhi 2012] Ans. 6 [1 mark]
23. The ground state energy of hydrogen atom is -13.6eV. If an electron makes a transition from an energy
level-0.85eV to - 3.4eV, calculate the wavelength of the spectral line emitted. To which series of
hydrogen spectrum does this wavelength belong?
[Outside Delhi 2012, Udgam, MAV] Ans. 484.8nm, Balmer [3 marks]
24. What is the radii of the orbits corresponding to first excited state and ground state in a hydrogen atom?
[Delhi 2010] Ans. 2.12 x10-10m , 5.3x10-11m [2 marks]
25. A difference of 2.3 eV separates two energy levels in an atom. What is the frequency of radiation
emitted when the atom make a transition from the upper level to the lower level? Ans. 5.6x10 14 Hz
[2 marks]

123
Physical Quantity SI Unit Scalar/Vector Dimension

Angular kgm2 s-' Vector Scalar [¥1 er1 l


Momentum

Rydberg's constant m-' Scalar [_llAo l;T o 7

List of Formulas
Quantity Formula Quantity Formula

Radius of Atom 2KZe2 4KZe2 Potential Energy - Kc e2


r=--=-- Un =
ka mv2 rn
0

Angular nh Rydberg's formula


Ln=mvnrn=- ±= �=
Momentum 2lf R[;; - �/

Kinetic Energy 2 Energy Difference LiE= Ei -Et


2 e
K = .!..mv= _!_ mkc
D2 Il2ffifll

Derivation List:
1. Expression for ro.
2. De Broglie's explaination of Bohr's second postulate.

124
SUMMARY

125
CH-13 Nuclei
Session 13.1 (Nuclear forces, Binding Energy)
1. Nucleus:
Rutherford's 𝛼-scattering experiment established that the mass of atom is concentrated with smallpositively
charged region at the centre which is called 'nucleus'.
Radius of nucleus is of the approx. 10−15 𝑚 (1 femto).
Nuclei are made up of proton and neutron.
Atomic number:
The number of protons in a nucleus (called the atomic number or proton number) is represented by the
symbol Z.
The number of neutrons (neutron number) is represented by N.
Mass number:
The total number of neutrons and protons in a nucleus is called it's mass number A so A = Z + N.
The following terms are used to describe the composition of atomic nucleus: -
Nucleons:
Protons and neutrons which are present in nucleus of an atom are collectively called nucleons.
An atomic nucleus is designated by symbol 𝐴𝑍𝑋, where X is chemical element symbol.
2. Different type of nuclei:
The nuclei have been classified on the basis of the number of protons (atomic number) or the total number
of nucleons (mass number) as follows:
(i)Isotopes:
The atoms of an element having same atomic number but different mass number are called isotopes.
Eg. 6C12 and 6C14 are isotopes of carbon, 1H1, 1H2 (deuterium) and 1H3(tritium) are isotopes of hydrogen.
(ii)Isobars:
The atoms having same mass number but different atomic number are called isobars.
Eg. 1H3 and 2He3, 17Cl37 and 16S37.
(iii)Isotones:
Atoms of different elements having same number of neutrons are called isotones.
Eg., 17Cl37 and 19K39, having no of neutrons=20.
(iv)Isomers:
These are the nuclei with same atomic number and same mass number but different energy states, like: an
atom in ground state and an atom in excited states.
3. Atomic mass unit:
1 𝑡ℎ
One atomic mass unit (amu) is defined as mass of mass of carbon-12 ( 126𝐶 ) atom. 1amu=1.67×10-27kg.
12
Energy equivalent of amu: By Einstein’s mass-equivalence relation, E=mc2 and m=1.67×10−27kg and
1.67 ×10−27 ×9 ×1016
c=3×108m/s, E= 106 ×1.6×10−19
=931MeV. Thus, 1amu=931.5MeV.
4. Relation between nuclear size and mass number:
The volume of a nucleus is directly proportional to its mass number. If R is the radius of a nucleus having
4
mass number A, then πR3∝ A.
3
Thus, R ∝ A1/3⟹ R=R0A1/3

126
5. Nuclear Density:
The density of a nuclear matter is the ratio of mass of a nucleus to its volume. As the volume of nucleus is
directly proportional to its mass number A, so the density of nuclear matter is independent of the size of the
nucleus. It is numerically equal to 2.3×1017kg/m3.
Proof of nuclear density is constant:
Let A be the mass number and R be the radius of a nucleus. If m is the average mass of a nucleon, then mass of
nucleus=mA.
4 4 4
Volume of nucleus = πR3 = π(R0A1/3)3= πR03A, where R0 is constant.
3 3 3
𝑚𝑎𝑠𝑠 𝑚𝐴 3𝑚
So, nuclear density, ρnu = =4 =
4𝜋𝑅03
𝑣𝑜𝑙𝑢𝑚𝑒 𝜋𝑅03 𝐴
3
Taking m = 1.67×10−27 kg, R0 = 1.2×10−15 m, we get ρnu = 1017 kgm-3, which is a constant.

6. Chadwick Experiment for discovery of neutrons:


The neutrons were discovered by James Chadwick in 1932.
(i) Chadwick performed an experiment in which α-particles from a radioactive source were used to
bombard Beryllium nuclei.
(ii) Highly penetrating rays were found to come out of the beryllium metal, which could not be deflected by
electric and magnetic fields (thus, these rays contained chargeless particles).
(iii) These radiations were used to bombard hydrocarbons like paraffin wax.
(iv) High energy protons were knocked out from the paraffin wax.
(v) By using the laws of conservation of energy and momentum, Chadwick concluded that the penetrating
radiation consisted of neutral particles, each having a mass nearly that of proton. These particles were
called neutrons.
The above experiment has the reaction given by, 2He4 + 4Be9→0n1 + 6C12.
Thermal neutrons: Fast neutrons can be converted into slow neutrons by certain materials called
moderator's (Paraffin wax, heavy water, graphite). when fast moving neutrons pass through a moderator,
they collide with the molecules of the moderator, as a result of this, the energy of moving neutron decreases
while that of the molecules of the moderator increases. After sometime they both attains same energy. The
neutrons are then in thermal equilibrium with the molecules of the moderator and are called thermal
neutrons.

127
7. Nuclear Force:
Force that keep the nucleons (neutron and proton) bound in the nucleus is called nuclear force.
Properties:
(i) Strongest interactive force:
Nuclear forces are the strongest forces in nature.
Ratio of gravitational, coulombic and nuclear force is:
1:1036:1038.
(ii) Short range forces:
It operates only upto a very short distance of about 2-3fm
from the nucleon.
(iii) Charge independent:
The nuclear force does not depend on the charge of the
particles or nucleons.
(iv) Non central forces:
The nuclear force between two nucleons does not act along the line joining their centres.
(v) Variation with Distance:
Figure 1 shows a graph of the potential energy of a pair of nucleon as a function of their separation.
Graph indicates that the attractive force between two nucleons is strongest at a separation of R0=
0.8fm.
For a separation greater than R0, the force is attractive and for separation less than R0, the force is
strongly repulsive.
(vi) Saturation effect:
Nuclear forces show saturation effect, i.e., a nucleon interacts only with its neighbouring nucleon.
8. Mass Energy Relationship:
Einstein showed from his theory of special relativity that it is necessary to treat mass as another form of
energy. Before the advent of this theory of special relativity it was presumed that mass and energy were
conserved separately in a reaction.
However, Einstein showed that mass is another form of energy and one can convert mass-energy into other
forms of energy, say kinetic energy and vice-versa.
Einstein gave the famous mass-energy equivalence relation E = mc2.
9. Mass defect:
It is found that the mass of a stable nucleus is always less than the sum of the masses of its constituent
protons and neutrons in their free state.
The difference between the rest mass of a nucleus and the sum of the rest mass of its constituent nucleons
is called its mass defect. It is denoted by Δm.
Consider the nucleus ZXA. It has Z protons and (A−Z) neutrons.
Hence, Δm = Zmp +(A-Z)mn− m, where mp, mn and m are the rest mass of proton, neutron and nucleus.
In nuclear reaction mass defect Δm = mass of reactant – mass of product.
If mass defect is positive then energy will release and reaction will be spontaneous. Otherwise non-
spontaneous.
10. Binding Energy:
The binding energy of a nucleus may be defined as the energy required to break up a nucleus into its
constituent protons and neutrons and to separate them to such a large distance that they may not interact
with each other.
Expression for Binding energy: ΔEb=Δmc2=[Zmp+(A−Z)mn−m]c2. ΔEb=Δm×931eV, where Δm is in amu
(atomic mass unit).

128
11. Binding Energy per nucleon:
The binding energy per nucleon is the average energy required to extract one nucleon from the nucleus.
∆𝐄𝐛
ΔEbn= .
𝐀
The binding energy per nucleon gives a measure of force which binds the nucleus together inside a nucleus.
More the average binding energy, higher the stability of nucleus.
12. Binding Energy Curve:
Figure 2 shows the graph of binding energy per nucleon drawn against mass number A.
Important features of Binding Energy Curve:-
(i) Binding energy is
small for light nuclei
like 1H1, 1H2, 1H3.
(ii) In the mass number
range 2-20, there are
well defined maxima
and minima on the
curve. The maxima
are at even atomic
number (2He, 6C, 8O)
and minima at odd
atomic number (3Li,
7N).
(iii) The curve has value
8.5MeV/nucleon in
mass number range from 30-150. It has peak value of 8.8MeV/nucleon for 26F56.
(iv) As the mass number increases further, the B.E./nucleon shows a gradual decrease and drop to
7.6MeV/nucleon for 92U238.
Important Conclusions from Binding Energy Curve:-
(i) The force is attractive and sufficiently strong to produce a binding energy of a few MeV per nucleon.
(ii) The constancy of the binding energy in the range 30 < A < 170 is a consequence of the fact that the
nuclear force is short-ranged.
(a) Consider a particular nucleon inside a sufficiently large nucleus. It will be under the influence of
only some of its neighbours, which come within the range of the nuclear force. If any other
nucleon is at a distance more than the range of the nuclear force from the particular nucleon it
will have no influence on the binding energy of the nucleon under consideration.
(b) If a nucleon can have a maximum of p neighbours within the range of nuclear force, its binding
energy would be proportional to p. Let the binding energy of the nucleus be pk, where k is a
constant having the dimensions of energy.
(c) If we increase A by adding nucleons they will not change the binding energy of a nucleon inside.
Since most of the nucleons in a large nucleus reside inside it and not on the surface, the change
in binding energy per nucleon would be small.
(d) The binding energy per nucleon is a constant and is approximately equal to pk.
(e) The property that a given nucleon influences only nucleons close to it is also referred to as
saturation property of the nuclear force.

129
Importance of Binding Energy Curve:
Binding energy can be used to explain nuclear fission and nuclear fusion.
(i) Nuclear Fission:
Binding energy per nucleon is smaller for heavier nuclei than the middle ones, so nucleus is less
stable. When a heavier nucleus split into lighter nuclei, the B.E./nucleon changes from about 7.6MeV
to 8.4MeV. Due to greater binding energy of daughter nuclei, stability is more. And increase in B.E.
result in liberation of energy that what happens in atom bomb.
(ii) Nuclear Fusion:
Binding energy per nucleon is small for lighter nuclei, so they are less stable. So, when two nuclei
combine to form a heavier nucleus, the B.E/nucleon increase which result in the release of energy.
This is what happens in a nuclear fusion which is the basis of Hydrogen bomb.

Solved Examples
Eg 1: Obtain the binding energy (in MeV) of a nitrogen nucleus (147N). Given m(147N)=14.00307 u,
m(1H1)=1.00783 u, mass of n=1.00867u.
Sol: The 147N nucleus contains 7 protons and 7 neutrons.
Mass of 7 protons = 7×1.00783 = 7.05481u
Mass of 7 neutrons = 7×1.00867 = 7.06069u
Thus, total mass = 7.005481+7.06069 = 14.11550u
Mass of 147N nucleus = 14.00307u
Mass defect, Δm = 14.11550 – 14.00307 = 0.11243u
Ans. Binding Energy of nitrogen nucleus = 0.11243×931 = 104.67MeV
Eg 2: (a) Two stable isotopes of lithium 36 Li and 37 Li have respective abundances of 7.5% an 92.5%. These
isotopes have masses 6.01512 u and 7.01600 u, respectively. Find the atomic mass of lithium.
(b) Boron has two stable isotopes 10
5 B
and 115 B. Their respective masses are 10.01294 u and 11.00931 u, and
the atomic mass of boron is 10.811 u. Find the abundances of 105 B and 115 B.
Sol: Abundance of 36 Li is 7.5% and abundance of 37 Li is 92.5%.
Hence atomic mass of lithium,
7.5(6.01512u) + 92.5(7.01600u)
A=
100
451134 + 648.98
=A = u 6.941u
100
(b) Let abundance of 105 B x % the abundance of 115 B will be (100 – x)%.
Atomic mass of boron
x[10.01294u] + (100 − x)[11.00931u]
=
100
⇒ 100 ×10.811
= u 1100.931 u − 0.99637x u
Solving we=get, x = 19.831
19.9%
0.99637

So, relative abundance of 105 B isotope = 19.9%

Relative abundance of 11
5 B
isotope = 80.1%

Eg 3: Calculate the energy equivalent of 1 g of substance.


Sol: Energy, E = 10–3 × (3 × 108)2 J

E = 10–3 × 9 × 1016 = 9 × 1013 J

Thus, if one gram of matter is converted to energy, there is a release of enormous amount of energy.

130
Classroom Questions : -
Level – 1
1. Calculate the amount of energy released during the ∝-decay of 238 234 4
92𝑈 → 90𝑇ℎ+ 2𝐻𝑒 .
Given : atomic mass of 238
92𝑈 = 238.05079 u, atomic mass of
234
90𝑇ℎ = 234.04363 u atomic mass of
4 2
2𝐻𝑒 = 4.00260 u, 1 u = 931.5 MeV/c ] Is the decay spontaneous? Give reasons. [Delhi 2007]
Ans. 4.25MeV [3 marks]
2. Calculate binding energy per nucleon of 20Ca40 nucleus. Given, mass of 20Ca40=39.96u, mass of
proton=1.007825u, mass of neutron=1.008665u and 1u=931MeV/c2. [Outside Delhi 2007]
Ans. 8.5468MeV/nuclei [2 marks]

Level – 2
1. Calculate the amount of energy released in following reaction: 1H2 + 1H3→2He4 + n. m(1H2)=2.014u,
m(1H3)=3.016u, m(2He4)=4.0026u,mn= 1.00867u, 1u=931.5MeV. [Foreign 2007]
Ans. 17.584MeV [2 marks]
2. A given coin has a mass of 3.0 g. Calculate the nuclear energy that would be required to separate all
the neutrons and protons from each other. For simplicity assume that the coin is entirely made of 63
29 Cu

atoms (of mass 62.92960 u). [Outside Delhi 2005] Ans. 1.6 × 1025 [3 marks]

131
Homework Sheet 13.1
Theory
1. What is Einstein’s mass-energy equivalence? What is its importance? [2 marks]
2. What are thermal neutrons . [Board exam] [1 mark]
3. Draw a graph showing the variation of potential energy of a pair of nucleons as a function of their
separation. Indicate the regions in which nuclear force is (i) attractive, and (ii) repulsive.
[Delhi 2013, Outside Delhi 2012, 2010, 2007, 2005] [1 mark]
4. Write four characteristic features of nuclear force which distinguish it from the Coulomb force. [Delhi
2013, Outside Delhi 2012, Outside Delhi 2010,Compartment 2011, Outside Delhi 2011] [2 marks]
5. Draw a plot showing the variation of binding energy per nucleon with mass number, A. Write two
important conclusions which you can draw from this plot. Explain with the help of this plot, the release
in energy in the processes of nuclear fusion and fission.
[Foreign 2013, Compartment 2012, Outside Delhi 2010] [3 marks]
6. What characteristic property of nuclear force explains the constancy of binding energy per nucleon
(BE/A) in the range of mass number ‘A’ lying 30<A<170? Ans. Saturation property of nuclear force.
[Compartment 2012] [1 mark]
7. If the nucleons bound in a nucleus are separated apart from each other, the sum of their masses is
greater than the mass of the nucleus. Where does this mass difference go? Explain briefly.
[Ans: The mass of the nucleons is greater than the mass of nucleus. When the nucleons are brought near
each other, then nuclear forces start to act, and due to these forces, mass difference is converted into
binding energy using Einstein’s mass-energy relationship. Since nuclear forces are attractive forces, thus
they decrease the potential energy and hence this energy arises due to mass difference and is finally
converted to binding energy.] [Outside Delhi 2007] [2 marks]
Derivations
8. Show that the density of nucleus over a wide range of nuclei is constant-independent of mass number A.
[Compartment 2012] [2 marks]
Application Based
9. Two nuclei have mass number in the ratio of 8:125, what is the ratio of their nuclear radii? [1 mark]
10. If the nuclei, with lower binding energy per nucleon, transform to nuclei with greater binding energy per
nucleon, would the reaction be endothermic or exothermic? Justify your answer and write two examples
to support your answer. [Ans:The reaction will be exothermic because when some nuclei with lower
binding energy is converted to greater binding energy, then the nuclei attains more stability and hence it
releases energy. Two examples are nuclear fission. Write 2 nuclear fission reactions.] [2 marks]
Numerical
11. A heavy nucleus X of mass number 240 and binding energy per nucleon 7.6MeV is split into two
fragments 110 and 130. The binding energy per nucleons in Y and Z is 8.5MeV per nucleon. Calculate the
energy released per fission in MeV. [Delhi 2010] Ans. 216MeV [2 marks]
12. When four hydrogen nuclei combine to form a helium nucleus, estimate the amount of energy in MeV
released in the process of fusion (Neglect the masses of electrons and neutrinos). Given: (i) mass of
1
H1=1.007825u, (ii) mass of helium nucleus=4.002603u 1u=931MeV/c2.
[Foreign 2011] Ans. 26.7MeV [2 marks]
13. Obtain the binding energy (in MeV) of a nitrogen nucleus (147N). Given m(147N)=14.00307 u,
m(1H1)=1.00783 u, mass of n=1.00867u. [2 marks]
14. The three stable isotopes of neon : 10 Ne, 10 Ne and 10 Ne have respective abundances of 90.51%, 0.27%
20 21 22

and 9.22%. The atomic masses of the three isotopes are 19.99 u, 20.99 u and 21.99 u, respectively.
Obtain the average atomic mass of neon.
15. Obtain the binding energy of the nuclei 56
26 Fe
and 83
209
Bi in units of MeV from the following data:

26 Fe) = 55.934939
m(56 u 209
m(83 Bi) = 208.980388 u

132
(Space for classroom notes)

133
Session 13.2 (Radioactive Decay, Nuclear fission, Nuclear fusion)
1. Radioactivity:
It is the phenomenon of spontaneous disintegration of the nucleus of an atom with the emission of one or
more penetrating radiations like α-particles, β-particles or γ-rays.
α-particle is Helium nucleus, i.e., He+2, β-particle is high speed electron and γ-rays are electromagnetic
waves. The substances which spontaneously emit penetrating radiations are called radioactive substances.
2. Law of radioactive decay:
The number of nuclei disintegrating per second of a radioactive sample at any instant is directly proportional
to the number of undecayed nuclei present in the sample at that instant.
Let N0 be number of radioactive nuclei present initially, N be radioactive nuclei present at time t and dN be the
number of radioactive nuclei disintegrated in time interval dt.
−𝑑𝑁
Thus, ∝N [−ve sign show that number of active nuclei are decreasing with time]
𝑑𝑡
−𝑑𝑁
⟹ = λN, where λ is decay constant.
𝑑𝑡
−𝑑𝑁
Now, = λdt
𝑁
Integrating on both sides, we get,
1
− ∫ 𝑑𝑁 = ∫ 𝜆 𝑑𝑡
𝑁
loge N = −λt + C, where C is integration constant.
At t = 0, N = N0. Hence, C = logeN0
Thus, logeN = −λt + logeN0
Hence, we get, N = N0e–𝛌𝐭 which represents radioactive decay law.

The graph between N and t tells following features: N


(i) The number of active nuclei in a radioactive sample decreases
N0
exponentially with time. Initially disintegration is fast and
becomes slower consecutively. Half life = T

(ii) The larger value of decay constant λ indicates larger rate of N0/2
disintegration. N0/4
–𝛌𝐭 1
Decay constant: In equation, N=N0e , putting t = , we get
𝜆 0 1 2T 3T
𝑁0 t
N= 𝑒
= 0.368N0 = 36.8%N0.
So, the radioactive decay constant is defined as the reciprocal of time
interval during which the number of active nuclei in a given radioactive sample reduces to 36.8% of its initial
value.
3. Half Life:
The time interval in which one-half of the radioactive nuclei originally present in radioactive sample
disintegrate is called half-life of the radioactive substance. It is denoted by T1/2.
Relation between half life and decay constant:

Let N0 be number of radioactive nuclei initially and N be radioactive nuclei left at time t.
At t=T1/2, N=N0/2
Using radioactive decay equation, we get 𝑒 𝜆𝑇1/2 = 2
Taking natural log on both sides, we get λT1/2 = loge2
log𝑒 2
⟹T1/2 = 𝜆
𝟎.𝟔𝟗𝟑
⟹T1/2 = 𝝀

134
Let N0 be the initial radioactive nuclei and N be the radioactive nuclei left after ‘n’ half life disintegrations.
𝑵 𝟏 𝒏
Then, N=N0(½)n or =� �
𝑵𝟎 𝟐
*FRACTION of active/decayed atom at different time:

Time (t) Remaining fraction of active atoms Fraction of atoms decayed (N0 – N)
(N/N0) probability of survival /N0 probability of decay
t=0 1 (100%) 0
t = T1/2 1
(50%)
1
(50%)
2 2

t = 2(T1/2) 1
(25%)
3
(75%)
4 4

t = 3(T1/2) 1
(12.5%)
7
(87.5%)
8 8

t = 10 (T1/2) 1
10
≈ 99.9%
  ≈ 0.1%
 2

t = n (N1/2) 1
n
  1 n 
  1 −   
 2   2  

4. Mean Life:
The average time for which the nuclei of a radioactive sample exist is called mean life or average life of that
1
sample. It is denoted by τ. Mathematically, τ = . Hence, τ = 1.44T1/2.
λ
𝑆𝑢𝑚 𝑜𝑓 𝑡ℎ𝑒 𝑙𝑖𝑣𝑒𝑠 𝑜𝑓 𝑎𝑙𝑙 𝑡ℎ𝑒 𝑛𝑢𝑐𝑙𝑒𝑖
Mean life ‘τ’=
𝑇𝑜𝑡𝑎𝑙 𝑛𝑢𝑚𝑏𝑒𝑟 𝑜𝑓 𝑛𝑢𝑐𝑙𝑒𝑖
5. Activity(R) or decay rate of a radioactive sample:
It is defined as the number of radioactive disintegrations taking place per second in the sample.

Suppose, in a time interval dt, the decay constant measured is ΔN.


𝑑𝑁
Then, dN = −ΔN. Hence, R = − 𝑑𝑡
We know by radioactive law, N = N0𝑒 –λt
Differentiating above equation w.r.t. time, we get,
𝑑𝑁 𝑑
⟹ = (N0𝑒 –λt )
𝑑𝑡 𝑑𝑡
⟹R = λN0𝑒 –λt
⟹ R = R0𝑒 –λt , where R0 = λN0.
The decay rate R at a certain time t and the number of undecayed nuclei N at the same time are related
by R=λN

6. Units of Radioactivity:
(i) Becquerel: It is the SI unit of activity. It is defined as the decay rate of one disintegration per second.
1 Becquerel=1Bq=1 decay per second.
(ii) Curie: It is decay rate of 3.7×1010 disintegrations per second. 1curie=1Ci=3.7×1010Bq.

135
7. Q -value or energy of nuclear reaction:
The energy absorbed or released during nuclear reaction is known as Q-value of nuclear reaction.
Q-value = (Mass of reactants – mass of products)𝑐 2 Joules
= (Mass of reactants – mass of products) amu
If Q < 0, The nuclear reaction is known as endothermic. (The energy is absorbed in the reaction)
If Q > 0, The nuclear reaction is known as exothermic (The energy is released in the reaction)
8. Alpha Decay:
It is a process in which an unstable nucleus transforms itself into a new nucleus by emitting an alpha particle
(an helium nucleus). It is expressed by the equation: ZXA→Z-2YA –4+2He4+Q. Here Q is the energy released in
the process and can be determined by Einstein’s mass-energy relation which is Q=[mX− mY−mHe]c2. This
energy is shared by X and α-particle.
Eg. 92U238→90Th234+2He4+Q.
Energy Q is the kinetic energy of the released α-particles, which on striking a metal body is released as heat.
Extra Example:
In the reaction AZ X → AZ−−24 Y + 24 He + Q of the nucleus X at rest, taking the ratio of mass of
α − particle Mα and mass of Y-nucleus MY as Mα = 4 , show that the Q-value of the reaction
MY A−4
is given by Q = Kα  A  , where Kα = kinetic energy of α − particle.
 
 A−4 
Sol. Q-value of reaction = energy equivalent of mass-difference.
= (MX – MY – Mα ) c2
= increase in kinetic energy
= (Kα + K Y ) − 0 ( X was steady)
= 1 Mα vα2 + 1 M Y v Y2 (1)
2 2
From conservation of momentum,
M Y v Y + Mα vα =
0
Mα vα (in magnitude)
∴ M Y vY = (2)
 Mα 
∴ vY =
  vα
 MY 
Substituting this value in equation (1),
2
1 1 M 
=Q Mα vα2 + M Y  α  vα2
2 2  MY 
1  Mα   4 
= Mα vα2  = + 1 Kα  + 1
2  Y 
M  A − 4 
 A 
= Kα  
A−4

9. Beta Decay:
The process of spontaneous emission of an electron e− or
positron e+ from a nucleus is called β-decay.

In beta minus (β ) decay, the mass number of radio-active
nucleus remains unchanged but its atomic number increase
by one. Like, 15P32→16S32 + β− +ν� , where ν� is antineutrino.
In β+ decay, the mass number of the radio-active nucleus
remains same but its atomic number decreases by one.
Like, 11Na22→10Ne22 +𝛃+ +ν, where ν is neutrino.
In other words, in β- decay a neutron converts itself into a
proton, n→p+𝛃− +ν� ,
and in β+ decay a proton converts itself into a neutron, p→n + 𝛃+ +ν.

136
Continuous energy spectrum for beta rays:
In both α- and β- decay, the disintegration energy Q depends on the nature of radionuclide, energy emitted
by α-particle has a definite amount of kinetic energy. However, in β-decay, the disintegration energy is
shared in all proportions between the three particles - daughter nucleus, electron (or positron) and
antineutrino (or neutrino). As a result, K.E. of β-ray have a continuous energy spectrum.
Fig. 4 shows energy division between an electron and antineutrino (or neutrino) in a β-decay reaction.
10. Gamma decay:
The process of emission gamma ray photon during the radio-
active disintegration of a nucleus is called gamma ray. γ-ray
have no matter and no charge. So, during gamma decay mass
number and atomic number remains unchanged. So, no new
nucleus is formed. Gamma ray can be expressed as
A A
ZX (excited state) →ZX (ground state)+ 𝛄.

Eg. In Fig. 5, an unstable 27Co60 nucleus is transformed in a β-


decay into an excited 28Ni60 nucleus, which in turn reaches the
stable ground state by emitting photons of energies 1.17MeV
and 1.33MeV, in two successive 𝛾 −decay processes.

11. Nuclear Energy:


The energy released during a nuclear reaction is called nuclear energy.
For the lighter nuclei region, A < 30, and for the heavier nuclei region, A > 170, the binding energy per
nucleon is less than 8.0 MeV, as we have noted earlier. This feature of the binding energy curve means that
nuclei in the middle region 30 ≤ A≤ 170 are more tightly bound than nuclei with A < 30 and A > 170. Energy
then can be released if less tightly bound nuclei are transmuted into more tightly bound nuclei.
Two types of nuclear reactions are: (i) Nuclear fission in which a heavy nucleus splits into two small nuclei,
(ii) Nuclear fusion in which two smaller nuclei fuse together to form a large nucleus.
12. Nuclear Fission:
The phenomenon in which a large nucleus (A > 230) when excited splits into two smaller nuclei of nearly
comparable masses is called nuclear fission. Nuclear fission for Uranium is as given:
235
92U + 0n 1 → 92U
236
→ 56Ba
141
+ 36Kr92+ 30n1+ Q.
(unstable)
Here, the value of Q for a nuclear fission is 216MeV.
The same reaction can produce other pairs of intermediate mass fragments
0 n + 92 U → 92 U → 56 Ba + 36 Kr + 30 n
1 235 236 144 89 1

1
0 n+ 235
92 U→ 236
92 U→ Sb +
133
51
99
41 Nb + 410 n
Or, as another example,
0 n + 92 U →54 Xe + 38Sr + 20 n
1 235 140 94 1

137
13. Nuclear Reactor:
(i) When 92U235 undergoes fission after bombarded by a neutron, it also releases a neutron. This
neutron is then available for initiating fission of another 92U235 nucleus. In fact, on an average, 3
neutrons per fission of uranium nucleus are released. The fact that more neutrons are produced in
fission than are consumed raises the possibility of a chain reaction with each neutron that is
produced triggering another fission.
(ii) The chain reaction can be either uncontrolled and rapid (as in a nuclear bomb) or controlled and
steady (as in a nuclear reactor). The first leads to destruction while the latter can be harnessed to
generate electric power.The average
energy of a neutron produced in
fission of 92U235 is 216 MeV.
(iii) These neutrons unless slowed down
will escape from the reactor without
interacting with the uranium nuclei,
unless a very large amount of
fissionable material is used for
sustaining the chain reaction. What
one needs to do is to slow down the
fast neutrons by elastic scattering
with light nuclei.
(iv) In reactors, light nuclei called
moderators are provided along with
the fissionable nuclei for slowing down fast neutrons. The moderators commonly used are water,
heavy water (D2O) and graphite.
(v) Because of the use of moderator, it is possible that the ratio, K, of number of fission produced by a
given generation of neutrons to the number of fission of the preceding generation may be greater
than one. This ratio is called the multiplication factor; it is the measure of the growth rate of the
neutrons in the reactor. For K = 1, the operation of the reactor is said to be critical, which is what we
wish it to be for steady power operation.
(vi) If K becomes greater than one, the reaction rate and the reactor power increases exponentially.
Unless the factor K is brought down very close to unity, the reactor will become supercritical and can
even explode. The reaction rate is controlled through control-rods made out of neutron-absorbing
material such as cadmium.
(vii) In addition to control rods, reactors are provided with safety rods which, when required, can be
inserted into the reactor and K can be reduced rapidly to less than unity. The broad outlines of a
typical nuclear power plant based on pressurised-water reactor are shown in Fig 6.
(viii) Coolants are the materials used to cool fuel rods and the moderator and is capable of carrying large
amount of heat produced in the fission process. The coolant transfers heat to the working liquid like
water and produces steam. The steam drives a turbine which, in turn, runs a generator to generate
electric power. The coolant must have high boiling point and high specific heat. Heavy water and
liquid sodium are good coolants.
(ix) In such a reactor, water is used both as the moderator and as the heat transfer medium. In the
primary-loop, water is circulated through the reactor vessel and transfers energy at high
temperature and pressure (at about 600 K and 150atm) to the steam generator, which is part of the
secondary-loop. In the steam generator, evaporation provides high-pressure steam to operate the
turbine that drives the electric generator. The low-pressure steam from the turbine is cooled and
condensed to water and forced back into the steam generator.

138
(x) The energy released in nuclear reactions is a million times larger than in chemical reactions.
Therefore, the nuclear reactors need fuel a million times less in mass than used in the chemical
reactors of the same power capacity. A kilogram of 92U235 on complete fission generates about 3 ×
104 MW.
(xi) However, in nuclear reactions highly radioactive elements are continuously produced. Therefore, an
unavoidable feature of reactor operation is the accumulation of radioactive waste, including both
fission products and heavy transuranic elements such as plutonium and americium.
Nuclear disaster: On April 26, 1986, the world’s worst nuclear power plant accident occurs at the
Chernobyl nuclear power station in the Soviet Union. Thirty-two people died and dozens more
suffered radiation burns in the opening days of the crisis.
14. Nuclear Fusion:
The process in which two light nuclei combine (at extremely high temperature) to form a heavier nucleus is
called nuclear fusion. For example, two protons combine to form a deuteron. 1H1 + 1H1→ 1H2 +e++ 0.42MeV.
(i) We find that two positively charged particles combine to form a larger nucleus. It must be realised
that such a process is hindered by the Coulomb repulsion that acts to prevent the two positively
charged particles from getting close enough to be within the range of their attractive nuclear forces
and thus ‘fusing’.
(ii) The height of this Coulomb barrier depends on the charges and the radii of the two interacting
nuclei. To generate useful amount of energy, nuclear fusion must occur in bulk matter. What is
needed is to raise the temperature of the material until the particles have enough energy – due to
their thermal motions alone – to penetrate the coulomb barrier. This process is called
thermonuclear fusion. The temperature of the core of the sun is only about 1.5×107K.
(iii) Therefore, even in the sun if the fusion is to take place, it must involve protons whose energies are
far above the average energy.
(iv) Thus, for thermonuclear fusion to take place, extreme conditions of temperature and pressure are
required, which are available only in the interiors of stars including sun. The energy generation in
stars takes place via thermonuclear fusion. The fusion reaction in the sun is a multi-step process in
which hydrogen is burned into helium, hydrogen being the ‘fuel’ and helium the ‘ashes’.
(v) The proton-proton cycle by which these occurs are as follows:
1 1 2 +
1H + 1H →1H + e + ν + 0.42MeV
+
e + e−→ 𝛾 + 𝛾 + 1.02MeV
2 1 3
1H + 1H →2He + 𝛾 + 5.49MeV
3 3 4 1 1
2H + 2H →2He + 1H + 1H + 12.86MeV
The net reaction is given by: 4 1H1 + 2e-→2He4 + 2ν + 6γ + 26.7MeV

Solved Examples
Eg 1:The number of nuclei of a given radioactive sample at a time t=0 and t=T are N0 and N0/n respectively.
Obtain an expression for the half-life (T1/2) of the nucleus in terms of n and T. [Compartment 2013]
[2 marks]
-λt
Sol: We know that, N=N0e
For, t=T, N=N0/n.
𝑁0
Hence,
𝑛
= N0e-λT
⟹ eλT = n
⟹ λT = logen
log𝑒 𝑛
⟹λ=
𝑇
0.693
We know that, T1/2 = 𝜆
log𝑒 𝑛
Substituting λ = , we get,
𝑇
𝟎.𝟔𝟗𝟑𝑻
Ans. T1/2 =
𝐥𝐨𝐠 𝒆 𝒏

139
Eg 2:In a given sampler, two radioisotopes, A and B, are initially present in the ratio of 1:4. The half lives of A and
B are respectively 100 years and 50 years. Find the time after which the amounts of A and B become equal.
[Outside Delhi 2012][2 marks]
Sol: Let NA, NB and TA, TB be the initial samples and half lives of A and B.
Hence, NA:NB = 1:4, TA = 100yrs and TB = 50yrs.
We know that, N= N0e-λt
Here, it is given that, NA’ = NB’
𝑁𝐴 𝑒 −𝜆𝐵 𝑡
⟹ =
𝑁𝐵 𝑒 −𝜆𝐴𝑡
1
⟹ = 𝑒 −𝑡(𝜆𝐵 −𝜆𝐴 )
4
⟹ log( ¼ ) = t (λA –λB)
0.693 0.693
⟹ -2(log2) = t ( - )
𝑇𝐴 𝑇𝐵
0.693 0.693
⟹ 2(0.693) = t( - )
50 100
2(100)
⟹t= = 200yrs
1
Ans. Amounts of A and B will be equal after 200yrs.
Eg 3: The half-life of 92 U
238
undergoing α − decay is 4.5 × 109 years. What is the activity of 1g sample of 92 U?
238

Sol: T1/2 = 4.5 × 109y


= 4.5 × 109 y x 3.16 x 107 s/y
= 1.42 × 1017 s
One k mol of any isotope contains Avogadro’s number of atoms, and so 1g of 92 U
238
contains
26
1 kmol × 6.025 × 10 atoms/kmol
238 ×10−3
= 25.3 × 1020 atoms.
The decay rate R is
R = λN
0.693 0.693 × 25.3 ×1020 −1
= = N s
T1/2 1.42 ×1017
= 1.23 × 104 s −1
= 1.23 × 104 Bq
Eg 4: Obtain the amount of 16
27 Co necessary to provide a radioactive source of 8.0 mCi strength. The half-life of
16
27 Co is 5.3 years.
Sol: Here rate of disintegration required
R = 8.0 mCi
= 8.0 × 10–3 × 3.7 × 1010 diss–1
= 29.6 × 107 diss–1
Half life T1/2 = 5.3 years = 5.3 × 3.16 × 107 s
But =R λ= N
0.693
.N
T1/2
No. of atoms for given rate required,
RT1/2
N=
0.693
29.6 ×107 × 5.3 × 3.16 ×107
=
0.693
= 7.15 ×1016 atoms
As 1 mole i.e., 60 g of cobalt contains 6.023 × 1023 atoms, so, the mass of cobalt required for given rate of
disintegration
60 × 7.15 ×1016
= = 7.123 ×10−6 g.
6.023 ×1023

140
Eg 3: The normal activity of living carbon-containing matter is found to be about 15 decays per minute for every
gram of carbon. This activity arises from the small proportion of radioactive 146 C present with the stable carbon
isotope 126 C. When the organism is dead, its interaction with the atmosphere (which maintains the above
equilibrium activity) ceases and its activity begins to drop. From the known half-life (5730 years) of 146 C, and the
measured activity, the age of the specimen can be approximately estimated. This is the principle of 146 C dating
used in archaeology. Suppose a specimen from Mohenjodaro gives an activity of 9 decays per minute per gram of
carbon. Estimate the approximate age of the Indus-Valley civilization.
Sol: In order to estimate age, let us first find the activity ratio in form of time ‘t’. Given normal activity, R0 = 15
decays min-1 Present activity, R = 9 decays min-1, Tin = 5730 years Since activity is proportional to the number of
radioactive atoms, therefore,
R N N 0e−λ t
= = = e−λ t
R 0 N0 N0
or 9
= e−λ t or eλ t =
15
15 9
Taking natural logarithm,
15
log e eλ t = log e
9
or λ t log
= ee 2.303log=
10
5
2.303 × 0.2218
3
or t = 0.5109 …(i) [ loge e = 1]
λ
Now we know, half life, T1/2 = 0.693
λ
0.5109 0.5109
∴=t = × T1/2
0.693 / T1/2 0.693
0.5109 × 5730 years = 4224 years.
=
0.693

141
Classroom Questions : -
Level – 1
1. Find the Q-value and the kinetic energy of the emitted α − particle in the α − decay of (a) 88 Ra
226
and (b)
220
86 Rn.
Given m (88226 Ra) = 226.02540 u, m (86222 Rn) = 222.01750 u,
m (86222 Rn) = 220.01137 u. m (84
216
Po) = 216.00189 u.
2. The radionuclide 11C decays according to
+
11
6 C →11
5 B+e + v: T1/2 = 20.3 min
The maximum energy of the emitted positron is 0.960 MeV.
Given the mass values:
m(11
6 C)
= 11.011434 u and m (116 B) = 11,009305 u,
calculate Q and compare it with the maximum energy of the positron emitted.

Level – 2
1. A 1000 MW fission reactor consumes half of its fuel in 5.00 y. How much 235
92 U did it contain initially?
Assume that the reactor operates 80% of the time, that all the energy generated arises from the fission
of 92 U
235
and that this nuclide is consumed only by the fission process.
2. For the β + (positron) emission from a nucleus, there is another competing process known as electron
capture (electron from an inner orbit, say, the K-shell, is capture by the nucleus and a neutrino is
emitted).
e + + AZ X → AZ−1Y + v
Show that if β + emission is energetically allowed, electron capture is necessarily allowed but not vice-
versa.
3. Calculate the height of the potential barrier for a head on collision of two deuterons. (Hint: The height
of the potential barrier is given by the Coulomb repulsion between the two deuterons when they just
touch each other. Assume that they can be taken as hard spheres of radius 2.0 fm.)

142
Homework Sheet 13.2
Theory
1. Write the basic nuclear process underlying β- decay of a given radioactive nucleus.
[Ans: n→p + e- + ν, where ν is antineutrino] [Compartment 2013] [1 mark]
2. Define the activity of a radioactive nucleus and state its S.I. unit. [Foreign 2013] [1 mark]
3. How is the mean life of a radioactive sample related to its half life? [Ans:τ = 1.44T1/2]
[All India 2016] [Foreign 2011] [1 mark]
4. Draw the energy level diagram showing the emission of β-particles followed by γ-rays by a 60Co27
nucleus.[Outside Delhi 2013] [2 marks]
5. What are moderators? Give examples. [1 mark]
Derivations
-λt
6. Derive the law of radioactive decay, viz. N = N0 e . [Outside Delhi 2011, All India2016 ] [2 marks]
7. Show that the decay ratio 'R' of a sample of a radionuclide is related to the number of radioactive nuclei
'N' at the same instant by the expression R =λN. [Delhi 2013] [2 marks]
Application Based
8. State the reason, why heavy water is generally used as a moderator in a nuclear reactor.
Ans: The basic principle of mechanics is that momentum transfer is maximum when the mass of colliding
particle and target particle are equal. Heavy water has negligible absorption cross-section for neutrons
and its mass is small; so heavy water molecules do not absorb fast neutorns; but simply slow them.
9. If both the number of protons and the number of neutrons are conserved in a nuclear reaction like
12
C6+12C6 → 20Ne10+4He2, in what way is mass converted into energy? Explain.
[Ans: Here, in the given nuclear reaction, mass defect arises when reaction takes place. When carbon
and carbon react to give Neon and Helium, mass of reactant nucleons and product nucleons is not equal.
The difference in mass of reactant and product nucleons is converted to energy by Einstein’s mass-
energy relation.] [Foreign 2010] [2 marks]
10. Explain, with the help of a nucleus reaction in each of the following case how the neutron to proton ratio
change during (i) alpha-decay, (ii) beta-decay. [Ans:(i) In alpha-decay, since mass decreases by 4 and
charge by 2, thus, decrease in proton and neutron is same=2. Thus, ratio remains same as before. (ii) In
beta-decay, number of proton is increased by 1 and there is no change in no of neutrons. Thus, ratio
becomes n/p+1 from n/p.] [Delhi 2006] [2 marks]
11. Why is it necessary to slow down the neutrons, produced through the fission of 235 92𝑈 nuclei (by
neutrons), to sustain a chain reaction? What type of nuclei are (preferably) needed for slowing down fast
neutrons? [Outside Delhi 2008] [2 marks]
12. A nucleus undergoes β − − decay. How does its (i) mass number (ii) atomic number change ? (AI 2013C)
[1 mark]
13. Answer the following points. (AI 2012 C) [3 marks]
(i) Why is the B.E/nucleons found to be constant for nuclei in the range of mass number (A) lying
between 30 and 170?
(ii) When a heavy nucleus with mass number A = 240 breaks into two nuclei, A = 120 energy is released
in this process.

Numerical
14. There are 4√2 × 106 radioactive nuclei in a given radioactive sample. If the half life of the sample is 20
s, how many nuclei will decay in 10 s ?

15. The half-life of 238U92 against α-decay is 1.5 × 1017 s. What is the activity of a sample of 238U92 having 25 ×
1020 atoms? [Delhi 2013] Ans. 11.55×103 disintegrations/sec [2 marks]

143
16. A radioactive nucleus ‘A’ undergoes a series of decays according to the following scheme:
𝛼 𝛽− 𝛼 𝛾
A→A1��A2→A3→A4
The mass number and atomic number of A are 180 and 72 respectively. What are these numbers for A4?
Ans. 172 and 69 [2 marks]
17. The half-life of 38 Sr is 28 years. What is the disintegration rate of 15 mg of this isotope?
90

18. The nucleus 23


10 Ne decays by β − emission. Write down the β − decay equation and determine the
maximum kinetic energy of the electrons emitted. Given that:
23
m(10 Ne) = 22.994466 u
23
m(11 Na) = 22.089770 u.

144
Extra Topic :

145
(Space for classroom notes)

146
(Space for classroom notes)

147
(Space for classroom notes)

148
Assignment Sheet
1. What is the ratio of the nuclear densities of two nuclei having mass number in the ratio 1:4? [1 mark]
[Ans. 1 : 1]
2. Write two characteristic of Nuclear force. [AI 2008] [1 mark]
3. Write symbolically the β − − decay process of 35
15 P. [AI 2010] [2 marks]
4. Two radioactive samples x, y have the same number of atoms at t = 0. Their half life are 3h and 4h
respectively. Compare the rate of disintegration of the two nuclei after 12 hours. [AI 2017] [2 marks]
5. Can it be concluded from β − decay that e– exist inside the nucleus.
6. Neutrons produced in the fission can be slowed down even by using ordinary water, then why is heavy
water is heavy used for this water is used for this purpose.
7. 23
10 Ne → Decays in the following way.

β +γ
23
23
10 Ne 
→11 Na + 0
−1

Find the minimum and maximum kinetic energy that the β − particle can have. The atomic masses of
23
10 Ne
and 1123 Na are 22.9454 u and 22.98984 u respectively.
8. A nucleus X initially at rest, undergoes α − decay according to the equation
92 X → Z Y + α (Alpha particle)
A 228

(a) find the values of ‘A’ and ‘Z’ (b) the α − particle in the above process is found to move in a circular
track of radius 1.1 × 102 m in a uniform field 3 × 103 T, find the energy during process and binding energy
of parent nucleus?
9. Explain the principle and working of a nuclear reactor with the help of a labeled diagram.
10. What is the role of moderator in a nuclear reactor.
11. Cadmium rods are used in nuclear reactor for what purpose.
12. Complete the equation for the following fission process 235
92 U + 10 n 
→ 90
38Sr + .........

13. A system of binary starts of masses mA and mB are moving in circular orbits of radii rA and rB,
respectively. If TA and TB are the time periods of masses mA and mB respectively then obtain TA = TB.
14. A 1000 MW fission reactor consumes half of its fuel in 5.00 y. How much 92
235
U did it contain initially?
Assume that the reactor operates 80% of the time, that all the energy generated arises from the fission
of 92
235
U and that this nuclide is consumed only by the fission process.

15. The fission properties of 239


94 Pu are very similar to those of 235
92 U. The average energy released per fission
is 180 MeV. How much energy, in MeV, is released if all the atoms in 1 kg of pure 239
94 Pu undergo fission?
16. The radionuclide 11C decays according to 11 +
T = 20.3 min
6 C → 5 B + e + v : 1/2
11

The maximum energy of the emitted positron is 0.960 MeV.


Give the mass values:

6 C) = 11.011434 u
m(11 and m(116 B) = 11.009305 u,
calculate Q and compare it with the maximum energy of the positron emitted.
17. The nucleus 1023 Ne decays by β − emission. Write down the β − decay equation and determine the
maximum kinetic energy of the electrons emitted. Given that:
23
m(10 Ne) = 22.994466 u
23
m(11 Na) = 22.089770 u.

18. The Q value of a nuclear reaction A + b → C + d is defined by Q = [mA + mb + mC – md]c2

149
where the masses refer to the respective nuclei. Determine from the given data the Q-value of the
following reactions and state whether the reactions are exothermic or endothermic.
(i) 11 H + 13H → 12 H + 12 H

(ii) 126 C + 126 C → 1020 Ne + 24 He

Atomic masses are given to be


m(12 H) = 2.014102 u

m(13 H) = 3.016049 u

6 C) = 12.000000 u
m(12
20
m(10 Ne) = 19.992439 u

19. The number of nuclei of a given radioactive sample at a time t=0 and t=T are N0 and N0/n respectively.
Obtain an expression for the half-life (T1/2) of the nucleus in terms of n and T.
[Compartment 2013] [2 marks]
20. In a given sampler, two radioisotopes, A and B, are initially present in the ratio of 1:4. The half lives of A
and B are respectively 100 years and 50 years. Find the time after which the amounts of A and b become
equal. [Outside Delhi 2012] Ans. 200yr [2 marks]
21. What is 𝛼 decay, 𝛽 -decay and 𝛾 –decay. Give example for each. [2 marks]
22. In the reaction given below :- [AI – 2016] [2 marks]
(a) 6 C →Y B + x + v
11 Z
(b) 6 C + 6 C →a Ne + b He
12 12 20 c

Find the value of x, y and z also a, b and c.


23. Tritium has a half-life of 12.5 years against α-decay. What fraction of a sample of pure tritium will remain
undecayed after 37.5 years?[Foreign 2013] Ans. 12.5% [2 marks]
24. Obtain the maximum kinetic energy of β − particle, and the
radiation frequencies of γ decays in the decay scheme shown in
Figure. You are given that
m(198Au) = 197.968233 u
m(198Hg) = 197.966760 u

150
Physical Quantity SI Unit Scalar/Vector Dimension

Nuclear Density Kgm −3 Scalar [M1L−3T 0 ]

Decay Constant s −1 Scalar [M 0 L0 T −1 ]

Half Life s Scalar [M 0 L0 T1 ]

List of Formulas
Quantity Formula Quantity Formula

Relation between 1
Binding Energy ∆E b =
∆mc 2
R = R 0A 3
nuclear size and
mass number = [zmp + (A − Z)m n − m]c 2

Mass defect ∆m= zm p + (A − Z) m n − m Half Life T1 =


0.693
2 λ

Radioactive Decay N = N 0 e−λ t Activity R = λN

Derivation List:
1. Derivation of Nuclear Density.
2. Derive the law of radioactive decay.

151
CH-14 (SEMICONDUTOR ELECTRONICS MATERIALS, DEVICES AND SIMPLE
CIRCUITS)
Session 14.1 (Semiconductors and Energy Bands)
1. Electronics:
• It is a branch of physics which deals with the controlled flow of electrons (current) through
inert gases,vacuum or semiconductors.
In the beginning, vacuum tubes were the building blocks of electronic circuits.
• Vacuum tubes:
Generally, it has two electrodes, viz., anode (often called plate) and cathode. In a vacuum
tube, the electrons are supplied by a heated cathode and the controlled flow of these
electrons in vacuum is obtained by varying the voltage between its different electrodes.
Vacuum is required in the inter- electrode space; otherwise the moving electrons may lose
their energy on collision with the air molecules in their path. In these devices the electrons
can flow only from the cathode to the anode (i.e.,only in one direction). Therefore, such
devices are generally referred to as valves.
• Drawbacks of Vacuum tubes:
Vacuum tubes are bulky, consume high power, operate generally at high voltages (~100
V) and have limited life and low reliability.
With the discovery of transistor in 1948, the vacuum tubes were replaced by the
semiconductor devicesbecause of its drawbacks.
2. Advantages of Semiconductors:
(i) Semiconductor devices are very small in size as compared to the vacuum tube. Hence
circuit using semiconductor devices are more compact.
(ii) Semiconductor device requires low voltage for their operation as compared to the vacuum
tube. So lot of electric power is saved.
(iii) In vacuum tubes, current flows when the cathode is heated and starts emitting electron. So,
we have to wait for some time for the operation of the circuit to having vacuum tube. On
other hand semiconductor device, no heating is required and circuit begins to operate as soon
as it is switched on.
(iv) Semiconductor device are shock proof, low cost, easily transported, efficient and reliable
compared to vacuum tube.
(v) Semiconductor devices do not produce any humming noise and have longer life than vacuum tube.
3. Classification of substances on the basis of resistivity:
(i) Metallic conductors:
The substances having low value of resistivity that is of the order of 10-2 to 10-8 Ωm are termed as
P P P P

metals.
Examples: Silver, copper, aluminium etc
(ii) Insulators:
The substances having high value of resistivity that is of the order of 1011 to 1019 Ωm are
P P P P

termed as insulators.
Examples: Wood, glass, diamond etc.
(iii) Semiconductors:
The substances having resistivity in between metals and insulators that is of the order of 10-5 P P

to 106 Ωm are termed as semiconductors.


P P

Examples: Silicon, germanium, gallium arsenide (GaAs) etc.


4. Types of semiconductors:
(i) Elemental semiconductors: Si and Ge
(ii) Compound semiconductors: Examples are:
a. Inorganic: CdS, GaAs, CdSe, InP, etc.
b. Organic: anthracene, doped pthalocyanines, etc.
c. Organic polymers: polypyrrole, polyaniline, polythiophene, etc.

152
5. Energy Bands in solids:
In isolated atom the valence electrons can exist only in one of the allowed orbitals each of a
sharply defined energy called energy levels. But when two atoms are brought nearer to each
other, there are alterations in energy levels and they spread in the form of bands. So, a collection
of many closely spaced energy levels is known as energy band.
Energy bands are of following types:
(i) Valence band
The energy band formed by a series of energy levels containing valence electrons is known
as valenceband.
(i) This band is always fulfill by electron.
(ii) Electrons are not capable of gaining energy from external electric field.
(iii) No flow of current due to such electrons.
(iv) The highest energy level which can be occupied by an electron in valence band at 0 K is
called fermi level.
(ii) Conduction band:
The energy band of higher energy level which is
either empty orpartial filled with electron above the
valance band is called the conduction band.
(i) This band is partially filled by the electrons.
(ii) In this band the electrons can gain energy from
external electricfield.
(iii) The electrons in the conduction band are called
the free electrons. They are able to move anywhere
within the volume of the solid.
(v) Current flows due to such electrons. Fig 1
(iii) Forbidden energy gap (∆Eg):
The difference in energy of lowest energy conduction band and maximum energy valance
band is knownas forbidden energy gap.
∆Eg= (C. B.) min − (V. B.) max = Energy gap between conduction band and valence band
R R R R

(i) No free electron present in forbidden energy gap.


(ii) Width of forbidden energy gap depend upon the nature of substance.
(iii) As temperature increases (↑), forbidden energy gap decreases (↓) very slightly.

153
6. Classification of solids on the basis of energy bands:
Property Metals Insulator Semiconductors
s

Energy Band

Energy gap Forbidden energy gap (E g )


R R The forbidden energy gap is large The forbidden energy gap is
iszero. (E g >3eV) and thus they do not
R R small (E g =1.1eV for Si and 0.7eV
R R

conduct. Even by applying strong for Ge).


electric fields, electrons cannot
beexcited.

Condition of Conduction band is either Conduction band is empty and Initially conduction band is
V.B. and partially filled or valence and valence band is completely empty and valence band is
C.B. at conduction bands overlap filled. filled, but with thermal
ordinary partially. excitation electrons are
temperat transferred to conduction
ure bands.

7. Holes in semiconductor:
At absolute zero temperature (0 K) conduction band of semiconductor is completely
empty and the semiconductor behaves as an insulator.
When temperature increases the
valence electrons acquires
thermal energy to jump tothe
conduction band (Due to the
breaking of covalent bond). If they
jumps to C.B. they leaves behind
the deficiency of electrons in the
valence band. This deficiency of
electron is known as hole or
cotter. A hole is considered
as a seat of positive charge, having
magnitude of charge equal to that
of an electron.
(1) Holes acts as virtual charge, although there is no physical charge on it.
(2) Effective mass of hole is more than electron.
(3) Mobility of hole is less than electron.
8. Intrinsic Semiconductors:
The semiconductor in which the current carriers are created due to thermal excitation only
across the for bidden energy gap is called an intrinsic semiconductor.
A pure semiconductor is called intrinsic semiconductor. It has thermally generated current carriers.
Structure and generation of hole:
(i) They have four electrons in the outermost orbit of atom and atoms are held together by
covalent bond.
(ii) With increase in temperature, the thermal energy of electrons increases.

154
(iii) Thus, some electron become free from covalent bond and jump to conduction band leaving
behind a vacancy (hole).
(iv) Number density of free electrons is n e and that of holes is n h . n e (in C.B.) = n h (in V.B.).Thus,
R R R R R R R R

n i =n e =n h where niis called intrinsic carrier concentration.


R R R R R R

(v) Semiconductors posses the unique property in which, apart from electrons, the holes also move.
Flow of charge carrier can be understood as follows.

Suppose there is a hole at site 1 as shown in Fig. 4(a). The movement of holes can be
visualised as shown in Fig. 4(b). An electron from the covalent bond at site 2 may jump to
the vacant site 1 (hole).Thus, after such a jump, the hole is at site 2 and the site 1 has now
an electron. Therefore, apparently, the hole has moved from site 1 to site 2.
Note that the electron originally set free [Fig. 4(a)] is not involved in this process of hole
motion. The free electron moves completely independently as conduction electron and
gives rise to an electron current, I e under an applied electric field.
R R

Remember that the motion of hole is only a convenient way of describing the actual
motion of bound electrons, whenever there is an empty bond anywhere in the crystal.
Under the action of an electric field, these holes move towards negative potential giving
rise to the hole current, I h .
R R

The total current, I is thus the sum of the electron current I e and the hole current I h
R R R

I = Ie+ Ih R R R

Recombination:
Apart from the process of generation of conduction electrons and holes, a simultaneous
process of recombination occurs in which the electrons recombine with the holes. Thermally
excited electrons collide with holes and recombine. At equilibrium, the rate of generation is
equal to the rate of recombination of charge carriers.
An intrinsic semiconductor will behave like an insulator at T = 0 K (fig 3(a)). It is the thermal
energy at higher temperatures (T > 0K), which excites some electrons from the valence band to
the conduction band. These thermally excited electrons at T > 0 K, partially occupy the
conduction band (fig 3(b)).
9. Limitations of intrinsic semiconductors:
Because of less number of charge carriers at room temperature, intrinsic
semiconductors have low conductivity so they have no practical use.
By adding some impurity conductivity can be increased.

155
10. Extrinsic semiconductors:
• A semiconductor obtained by doping a pure semiconductor with acceptor or donor impurity
atoms so as to increase its conductivity is called extrinsic semiconductor.
• Doping:
The process of deliberate addition of a desirable impurity to a pure semiconductor so as to
increase its conductivity is called doping.
The impurity atoms added are called dopants.
Requirements for doping:
(i) The semiconductor should be of high purity.
(ii) The size of dopant should be almost the same as that
of semiconductor atom. For this the atoms of
thirteenth and fifteenth group are most suitable.
(iii) The concentration of dopant atoms should be small,
about 1 part per million. 1ppm = 16 
 10 
(iv) The dopant atoms should not distort crystal lattices.
• Types of dopants (impurity) are of two types:
Pentavalent impurity Trivalent impurity

The elements whose atom has five valance The elements whose each atom has three
impurities e.g. As, P, Sb etc. These are also called valance electrons are called trivalent
donor impurities. These impurities are also called impurities e.g. In, Ga,Al, B, etc. These
donor impurities because they donates extra free impurities are also called acceptor
electron. impurities as they accept electron.

11. Types of extrinsic semiconductors:


(i) N-type semiconductor:
(a) Suppose we dope Si or Ge with a
pentavalent element. When an
atom of +5 valency element
occupies the position of an atom in
the crystal lattice of Si, four of its
electrons bond with the four silicon
neighbours while the fifth remains
very weakly bound to its parent
atom.
(b) As a result the ionisation energy
required to set this electron free is
very small and even at room
temperature it will be free to move
in the lattice of the semiconductor.
(c) Thus, the pentavalent dopant is donating one extra electron for conduction and
hence is known as donor impurity.
(d) The number of electrons made available for conduction by dopant atoms depends
strongly upon the doping level and is independent of any increase in ambient
temperature.
(e) On the other hand, the number of free electrons (with an equal number of holes)
generated by Si atoms, increases weakly with temperature.
(f) In a doped semiconductor, number of electrons is due to dopant as well intrinsically
generated while number of holes are due to intrinsic generation. Further hole
concentration decrease due to recombination.
(g) Thus, in an n-type semiconductor electrons are the majority charge carriers where
as holes are the minority charge carriers. Thus, n e >>n h .
R R R R

156
(h) Energy Band:
In n-type semiconductors, the extra electron is very weakly attracted
by donor impurity. A very small energy (0.01eV) is required to free it
from donor impurity. When freed, it will occupy the lowest possible
energy in the conduction band. Thus the donor energy level lies just
below the conduction band. At room temperature this small energy
gap is easily covered by thermally excited electrons. The conduction
band contains more electrons.
(ii) P-type semiconductor:
(a) This is obtained when Si or Ge is doped with a trivalent
impurity like A𝑙, B, In, etc. The dopant has one valence
electron less than Si or Ge and, therefore, this atom can
form covalent bonds with neighbouring three Si atoms but does
not have any electron to offer to the fourth Si atom.
(b) So the bond between the fourth neighbour and the trivalent
atom has a vacancy or hole. Since the neighbouring Si atom in
the lattice wants an electron in place of a hole, an electron in
the outer orbit of an atom in the neighbourhood may jump tofill
this vacancy, leaving a vacancy or hole at its own site.
(c) Thus the hole is available for conduction. Note that the trivalent
foreign atom becomes effectively negatively charged when it
shares fourth electron with neighbouring Si atom.
(d) Therefore, the dopant atom of p-type material can be treated as
core of one negative charge along with its associated hole.
It is obvious that one acceptor atom gives one hole.
(e) These holes are in addition to the intrinsically generated holes while the source of conduction
electrons is only intrinsic generation. Thus, for such a material,the holes are the majority
carriers and electrons are minority carriers.
(f) For p-type semiconductors, the recombination process will reduce the number (n i ) of R R

intrinsically generated electrons n e R

(g) Energy Band:


In p-type semiconductors, each acceptor impurity
creates a hole which can be easily filled by an
electron of semiconductor atom, i.e., a very small
energy (0.01-0.05eV) is required by electron of
valence band to move into this hole. Hence, the
acceptor energy level lies slightly above the
valence band. At room temperature, many
electrons get excited to acceptor energy level
leaving behind holes. These holes conduct
current. Thus, valence band has more holes than
conduction band.
 Note:
Due to doping, there is no change in overall charge neutrality of the semiconductor. This is
because when a donor atom is ionized, it creates a free electron or hole, but also it creates a
positively or negatively ionized donor atom. The charge on the free electron or hole and
ionized donor are equal and opposite. Hence, the net charge is zero.

 Equilibrium Condition:
The electron and hole concentration in a semiconductor in thermal equilibrium is given by
n i 2=n e n h . In an-type semiconductor, n e is greater than n i , thus n h becomes less than n i ,
R RP P R R R R R R R R R R R R

which implies number of holesare suppressed whereas in a p-type semiconductor, number of


electrons are suppressed.

157
Classroom Questions:-
Level – 1
1. Suppose a pure Si crystal has 5 × 1028 atoms m–3. It is doped by 1 ppm concentration of
P P P P

pentavalent As.Calculate the number of electrons and holes. Given that n i =1.5 × 1016m–3. R R P P P P

Ans. 5× 1022m–3, 4.5×109m–3


P P P P P P P P [2 marks]
8 -3
2. A semiconductor has equal electron and hole concentration of 2×10 m . On doping with a certain P P P P

impurity, the hole concentration increases to 4×1010m–3.(i) What type of semiconductor is P P P P

obtained on doping? (ii) Calculate the new concentration of the electrons. [Delhi 2013]
Ans. 106m–3
P P P [2 marks] P

16 –3
3. Pure silicon at 300K has equal hole and electron concentrations of 1.5×10 m . Doping by indium P P P P

increases the hole concentration to 4.5×1022m–3. Calculate the new electrons concentration in the
P P P P

doped silicon. Ans. 5×109m–3 P P P P [2 marks]

158
Homework Sheet 14.1
Theory
1. What is forbidden energy gap? [DPS] [1 mark]
2. C, Si and Ge have same lattice structure. Why is C insulater while Si and Ge intrinsic semiconductors?
[NCERT] [1 mark]
3. What is the difference between an n type and a p type extrinsic semiconductor? [Delhi-2012]
[1 makrk]
4. What are valence band and conduction band? [2 marks]
5. Draw energy band diagram for a n-type and p-type semiconductor at T > 0 K. [Delhi-2023] [2 marks]
6. Explain the term doping. What are extrinsic semiconductors? Name its two types. [2 marks]
7. What are energy bands? How are these formed? [Delhi 2006] [2 marks]
8. Distinguish between metals, insulators and semiconductors on the basis of their energy bands.
[Delhi 2013, Outside Delhi 2013] [2 marks]
9. What are the limitations of the intrinsic semiconductors when we use them for developing
semiconductor devices? [2 marks]
10. Distinguish between extrinsic and intrinsic semiconductors with the help of energy band
diagrams. [foreign 2017] [2 marks]
11. Distinguish between p-type and an intrinsic semiconductor. Give reason, why a p-type semiconductor
is electrically neutral, although n h >>n e ? [Outside Delhi 2012, Delhi 2008]
R R R R [3 marks]
12. Explain the formation of two types of extrinsic semiconductors. Also show their energy level diagrams.
[3 marks]
Application Based
13. The conductivity of an intrinsic semiconductor is very low. Why?
Ans. Because the number of free charge carriers are very low, so conductivity is low. [1 mark]
14. Why does the conductivity of a semiconductor increase with rise of temperature?
Ans. With rise in temperature, the valence electrons bounded to semiconductor atoms get enough
energy and jump conduction band and hence, the number of charge carriers increase. Thus,
conductivity increases. [1 mark]
15. Carbon and silicon both have four valence electrons each. Then, how are they distinguished?
Ans. In Carbon, the four valence electrons are each bounded to nucleus very strong due to small size
and high ionisation energy of carbon whereas in silicon the four valence electrons are loosely bond.
Thus, carbon forms covalent bonds with other elements whereas in silicon ionic bonds are formed.
[Delhi 2011] [2 marks]
Numerical
16. Suppose a pure Si crystal has 5 × 1028 atoms m–3. It is doped by 1 ppm concentration of pentavalent
P P P P

As.Calculate the number of electrons and holes. Given that n i =1.5 × 1016m–3. R R P P P P

Ans. 5× 1022m–3, 4.5×109m–3 [NCERT]


P P P P P P P P [2 marks]
8 -3
17. A semiconductor has equal electron and hole concentration of 2×10 m . On doping with a certain P P P P

impurity, the hole concentration increases to 4×1010m–3.(i) What type of semiconductor is obtained P P P P

on doping? (ii) Calculate the new concentration of the electrons. [Delhi 2013] Ans. 106m–3 [2 marks] P P P P

18. Pure silicon at 300K has equal hole and electron concentrations of 1.5×1016m–3. Doping by indium P P P P

increases the hole concentration to 4.5×1022m–3. Calculate the new electrons concentration in the
P P P P

doped silicon. Ans. 5×109m–3


P P P P [2 marks]

159
(Space for classroom notes)
U

160
Session 14.2 (P-n junction diode)
1. P-n junction:
• It is a single crystal of Ge or Si doped in such a manner that one half
portion acts as a p-type semiconductor and other half as n-type
semiconductor. In fact, the boundary dividing the two halves or
portions of such a semiconductor is called a junction and the
arrangement is known as p-n junction.
• Formation of p-n junction:
When a P-type semiconductor is suitably joined to an N-type semiconductor, then resulting
arrangement is called P-N junction or P-N junction diode.
a. Depletion region:
On account of difference in concentration of charge carrier in the two sections of P-N
junction, the electrons from N-region diffuse through the junction
into P-region and the hole from P region diffuse into
N-region. The movement of majority charge carriers
from N to P and P to N is known as diffusion.
Due to diffusion, neutrality of both N and P-type
semiconductor is disturbed, a layer of negative
charged ions appear near the junction in the P-
crystal and a layer of positive ions appears near the
junction in N-crystal. This layer is called depletion
layer.
1
(i) Width of depletion layer ∝
𝐷𝑜𝑝𝑙𝑖𝑛𝑔
(ii) Depletion is directly proportional to temperature.
b. Potential barrier (V B ):
R R

The potential difference created across the P-N junction due to the diffusion of electron
and holes is called potential barrier.
The barrier potential depends on (i) the nature of semiconductor, (ii) temperature, (iii)
amount of doping.
For Ge V B = 0.3V and for silicon V B =0.7V
R R R R

• Diffusion and drift current:


Because of concentration difference holes/electron try to diffuse from their side to the other side.
Only those holes/electrons cross the junction having enough kinetic energy. This diffusion results in an
electric current from the P-side to the N-side and is known as diffusion current (i df ).
R R

The barrier potential sets up a barrier electric field in the direction from n-region to p-region.
Due to this field, an electron on p-side(minority charge carrier) of the junction moves to n-side and a
hole on n-side of the junction moves to p-side. The motion of minority charge carriers due to the
electric field is called drift which results in a drift current (i dr ).
R R

Thus a drift current, which is in the opposite direction to the diffusion current starts. Initially, diffusion
current is large and drift current is small. As the diffusion process continues, the space-charge regions
on either side of the junction extend, thus increasing the electric field strength and hence drift
current. This process continues until the diffusion current equals the drift current. Thus a p-n junction
is formed.In steady state i df = i dr so i net = 0
R R R R R R

When no external source is connected, diode is called unbiased.


2. P-n junction diode:
• A semiconductor diode is basically a p-n junction with metallic
contacts provided at the ends for the application of an external
voltage. Fig 10
• It is a two terminal device.
• The circuit symbol for a p-n junction diode is as shown in fig 10.
• The p-side is known as anode and n-side as cathode.

161
3. Working of p-n junction:
An external potential to p-n junction can be applied in two ways
i. Forward Biasing:
If the positive terminal of a battery is connected to p-side
and negative terminal to n-side, then the p-n junction is
said to beforward biased.
Let applied voltage be denoted by V and barrier potential
by V B .As a result of forward biasing, the effective barrier
R R

potential decreases to V B −V and hence energy barrier


R R

across junction decreases.


The majority carriers begin to flow towards the junction.
The diffusion of holes and electrons into depletion layer
U

decreases its width. The effective resistance across p-n


U U

junction decreases. U

When V exceeds V B , the majority charge carriers flow


R R

easily and set up a large forward current which is


proportional to applied voltage. I-V characteristics of
forward biasing: The graph plotted between V and I for a
p-n junction in forward bias is known as forward
characteristic as shown in fig. 11.
Features of graph are:
(a) It is not a straight line, i.e., it does not follow Ohm’s
law (Ch-3,limitations of ohm’s law).
(b) Initially the current increases almost negligibly, till the voltage
crosses a certain value, called threshold or cut-in voltage. Before this voltage
is attained,depletion layer plays a dominant role in controlling the motion of
charge carriers.
(c) After cut-in voltage, the diode current increases rapidly, even for a very small
increase in diode voltage.
ii. Reverse Biasing:
If the positive terminal of a battery is connected to n-
side and negative terminal to p-side, then the junction
is said to be reversebiased.
Applied voltage is denoted by V and barrier potential by
V B . As a result of reverse biasing, the effective barrier
R R

potential increases toV B +V and hence energy barrier


R R

across junction increases.


The majority carriers begin to move away from junction, thus increasing depletion layer
width. The effective resistance across p-n junction increases.
No current flows across the junction due to majority charge carriers. However, due to
reverse biasing minority charge carriers are pushed towards the junction, setting up
reverse or leakage current is very small (  µA) and almost remains constant with change
in bias. (because concentration of minority charge is very low and does not change with
temperature). It is called reverse saturation current.
However, for special cases, at very high reverse bias (break down voltage), the
current suddenly increases. This is known as reverse bias break down.
At high reverse voltage, due to high electric field, the
minority charge carriers, while crossing the junction acquires
very high velocities and by collision breaks down the
covalent bonds, generating more carriers. A chain reaction is
established, giving rise to high current. This mechanism is
called avalanche breakdown.
I-V characteristics: The graph plotted between V and I for a
p-n junctionin reverse bias is known as reverse characteristic.

162
Difference in forward and reverse biasing:
Forward biasing Reverse biasing

(i) Positive terminal of the battery is Positive terminal of the battery is connected
connected to the P-crystal and to the N-crystal and negative terminal of the
negative terminal of the battery is battery is connected to P-crystal
connected to N-crystal

(ii) Width of depletion layer decreases Width of depletion layer increases

(iii) R Forward ≈ 10Ω - 25Ω


R R
RReverse≈ 105Ω

(iv) Forward bias opposes the potential Reverse bias supports the potential barrier
barrier and for V >V B a forward
R R and no current flows across the junction due
current is set up across the junction. to the diffusion of the majority carriers.
(v) Cut-in (Knee) voltage: The voltage at Break down voltage: Reverse voltage at which
which the current starts to increase. break down of semiconductor occurs. For Ge it
For Ge it is 0.3 V and for Si is 25 V and for Si it is 35 V
it is 0.7 V.

4. Dynamic Resistance of a diode:


It is defined as the ratio of small change in applied voltage to corresponding change in current. It is given
by
∆𝑽
r d = ∆𝑰.
R R

5. Rectifier:
• The process of converting an alternating current into a direct current is called
rectification and the device used for this purpose is called rectifier.
• Principle of rectifier: Property of PN junction diode to allow current to pass only when it is
in forwardbias. So when A.C. is applied across a junction diode current flow in positive half
cycle only.

163
• Working of diode as half-wave rectifier:
(a) The primary coil of a transformer is connected to the
a.c. mains and secondary coil with diode and load
resistance R L .
R R

(b) When a.c. is supplied to the primary coil, the


secondary coil supplies desired alternating current
at A and B.
During positive half cycle of a.c., end A is positive and
B is negative, thus the diode is in forward bias.
(c) Thus, it allows full flow of current and across load also.
The change in output current is synchronous to the input
current.
(d) During negative half cycle, end A is negative and B is
positive, thus diode is reverse biased.
(e) Thus, diode opposes the flow of current and no current
appears across load.
(f) Thus, signal for only positive cycle appears, i.e., half wave
appears. Thus, it is known as half-wave rectifier.
Frequency of rectified voltage is equal to input A.C. because in fig 14 time period for input
and outputvoltages are equal.
• Working of diode as full-wave rectifier:
(a) The circuit using two diodes, shown in Fig., gives output rectified voltage corresponding
to both the positive as well as negative half of the ac cycle.
Hence, it is known as full-wave rectifier.
(b) Here the p-side of the two diodes are connected to
the ends of the secondary of the transformer. The
n-side of the diodes are connected together and
the output is taken between this common point of
diodes and the midpoint of the secondary of the
transformer. So for a full-wave rectifier the
secondary of the transformer is provided with a
centre tapping and so it is called centre-tap
transformer.
(c) As can be seen from Fig. the rectified voltage by each diode
is only half the total secondary voltage.Each diode rectifies
only for half the cycle, but the two do so for alternate cycles.
Thus, the output between their common terminals and the
centre tap of the transformer becomes a full-wave rectifier
output. Suppose the input voltage to A with respect to
the centre tap at any instant is positive. So, at that instant,
voltage at B will be negative as shown in Fig. So, diode D 1 R R

gets forward biased and conducts (while D 2 being reverse


R R

biased is not conducting). Hence, during this positive half


cycle we get an output current (and a output voltage across
the load resistor R L )as shown in Fig. In the next half cycle
R R

when the voltage at A becomes negative with respect to


centre tap, the voltage at B would be positive. In this part of
the cycle diode D 1 would not conduct but diode D 2 would,
R R R R

giving an output current and output voltage (across R L ) R R

during the negative half cycle of the input ac. Thus, we get output voltage during both the positive
as well as the negative half of the cycle.
(d) Frequency of output voltage of a full wave is double than that of AC input because time period
become half.

164
Use of capacitor in a rectifier:
The rectified voltage from full-wave rectifier is in the form of pulses of the shape of half
sinusoids. Though it is unidirectional it does not have a steady value.
To get steady dc output from the pulsating voltage
normally a capacitor is connected across the output
terminals (parallel to theload R L ).
R R

When the voltage across the capacitor is rising, it gets


charged. If there is no external load, it remains charged
to the peak voltage of the rectified output.
When there is a load, it gets discharged through the load and the
voltage across it begins to fall. In the next half-cycle of rectified
output it again gets charged to the peak value. The rate of fall of
the voltage across the capacitor depends upon the inverse product
of capacitor C and the effective resistance R L used in the circuit
R R

and is called the time constant. It is equal to RC.


1
Here, the time rate of fall =
𝑅𝐶
To make the time constant large value of C should be large. So
capacitor input filters use large capacitors. The output
voltage obtained by using capacitor input filter is nearer
to the peak voltageof the rectified voltage. This type of
filter is most widely used in power supplies.

165
Classroom Questions:-
Level – 1
1. In the circuit in fig. 19 which one of the two diodes is forward
biased and which one is reverse biased? [Delhi 2013/2017] [1 mark]
2. What happens to the width of depletion layer of a p-n junction
when it is (i) forward biased, (ii) reverse biased? Ans. (i) Width
decreases, (ii) Width increases. [Compartment 2011, Outside Delhi
2011] [1 mark]
3. In half-wave rectification, what is the output frequency if the input
frequency is50 Hz. What is the output frequency of a full-wave
rectifier for the same input frequency? [NCERT]
Ans. 50Hz for half wave and 100Hz is the frequency of full-wave rectifier.
[1 mark]
Level – 2
4. Assuming that the two diodes D 1 and D 2 used in the electric circuit
R R R R

shown in the figure are ideal, find out the value of the current

flowing through 1Ω resistor. [Compartment] Ans. 2A [2 marks]


5. The V-I characteristic of a silicon diode is shown in the Fig 21.
Calculate the resistance of the diode at (a) I D = 15 mA and
R R

7
(b) V D = –10 V. Ans. 10 Ω, 1×10 Ω. [NCERT]
R R P P [2 marks]

166
Homework Sheet 14.2
Theory
1. Draw the circuit symbol for a p-n junction diode. [1 mark]
2. State the principle of working of p-n diode as a rectifier. [Foreign 2007, Delhi 2011] [1 mark]
3. How does an increase in doping concentration affect the width of depletion layer of a p-n junction
diode? [Delhi 2020] [1 mark]
4. Explain the terms depletion region and potential barrier. State the factor on which they depends.
[Delhi 2016, 2018] [2 marks]
5. Answer the following giving reasons :
(i) A p-n junction diode is damaged by a strong current.
(ii) Impurities are added in intrinsic semiconductors. [Delhi 2023] [2 marks]
6. Explain with the help of a circuit diagram, how the thickness of depletion layer, and value of
potential barrier affected when the p-n junction is forward biased? [Delhi 2013, 2020] [2 marks]
7. Explain the working of a PN-junction diode as a half wave rectifier. [2 marks]
8. How is forward biasing different from reverse biasing in a p-n junction diode?[Delhi 2011] [2 marks]
9. How is a depletion region formed in p-n junction?
[Compartment 2012,2013, Foreign 2013, Outside Delhi 2012, 2011, Delhi 2010] [3 marks]
10. With the help of a labeled circuit diagram, explain how a junction diode is used as a full
wave rectifier. Draw its input, output wave-forms. How do you obtain steady d.c. output
from the pulsating voltage? [Compartment 2013, Foreign 2013, Compartment 2012,
Outside Delhi 2012, 2019] [3 marks]
11. Explain briefly, with the help of circuit diagram, how V—I characteristics of a p-n
junction diode are obtained in (i) forward bias, and (ii) reverse bias. Draw the shape of
the curves obtained.
[Delhi 2013, Outside Delhi2013, 2010] [3 marks]
12. Explain briefly with the help of necessary diagrams, the forward and the reverse biasing of
a p-n junction diode. Also draw their characteristic curves in the two cases.
[Delhi 2017, 2018] [3 marks]
13. Explain briefly with the help of necessary diagrams, the forward and the reverse biasing of a
p-n junctiondiode. Also draw their characteristic curves in the two cases". [Board Delhi 2017]
[3 marks]
Application Based
14. In the circuit in fig. 19 which one of the two diodes is forward
biased and which one is reverse biased? [Delhi 2013] [1 mark]
15. What happens to the width of depletion layer of a p-n junction
when it is (i) forward biased, (ii) reverse biased? Ans. (i) Width
decreases, (ii) Width increases. [Compartment 2011, Outside Delhi
2011] [1 mark]
16. In the following diagram, is the junction diode forward biased or
reverse biased?

17. Can we take one slab of p-type semiconductor and physically join it to
another n-type semiconductor to get p-n junction? Give reason.
Ans. No, because continuous contact is not possible at atomic level through
physical contact and junction will behave as discontinuity for flow of charge
carrier. [NCERT] [1 mark]
18. In half-wave rectification, what is the output frequency if the input
frequency is 50 Hz. What is the output frequency of a full-wave
rectifier for the same input frequency?
Ans. 50Hz for half wave and 100Hz is the frequency of full-wave
rectifier. [1 mark]

167
19. Assuming that the two diodes D 1 and D 2 used in the electric circuit
R R R R

shown in thefigure are ideal, find out the value of the current flowing
through 1Ω resistor. [Compartment] Ans. 2A [2 marks]
Numerical
20. The V-I characteristic of a silicon diode is shown in the Fig 21.
Calculate the resistance of the diode at (a) I D = 15 mA and
R R

(b) V D = –10 V. Ans. 10 Ω, 1×107Ω.


R R P P [2 marks]

168
(Space for classroom notes)
U

169
Session 14.3(Special Purpose Diodes)
1. Zener Diode:
A junction diode specially designed to operate only in the reverse breakdown
region continuously (without getting damaged) is called Zener diode. Zener
diode is fabricated by heavily doping both p-, and n- sides of the junction. Due
to this, depletion region formed is very thin (<10-6 m) and the electric field of
P P

the junction is extremely high (~5×106 V/m) even for a small reverse bias
P P

voltage of about 5V.


2. Zener breakdown:
When reverse bias is increased the electric field at the junction
also increases. At some stage the electric field becomes so high
that it breaks the covalent bonds creating electron, hole pairs.
Thus a large number of carriers are generated. This causes a large
current to flow. This mechanism is known as Zener breakdown.
The I-V characteristics of a Zener diode are shown in fig. 23.
It is seen that when the applied reverse bias voltage(V) reaches the
breakdown voltage (Vz) of the Zener diode, there is a large change in the
current. Note that after the breakdown voltage Vz, a large change in the
current can be produced by almost in significant change in the reverse bias Fig. 23
voltage.
In other words, Zener voltage remains constant, even though current through the Zener diode
U

varies over a wide range. This property of the Zener diode is used for regulating supply voltages so
U U U

that they are constant.


3. Zener diode as voltage regulator:
To get a constant d.c. voltage supply after rectifying an input signal, zener diode is used. It
gives a constant power supply, i.e., constant voltage across
load.
Its working is as follows:
(a) The unregulated dc voltage (filtered output of a rectifier)
is connected to the Zener diode through aseries
resistance R s such that the Zener diode is reverse biased.
R R

(b) If the input voltage increases, the current through R


and Zener diode also increases.
(c) This increases the voltage drop across R s without any R R

change in the voltage across the Zener diode.


(d) This is because in the breakdown region, Zener voltage
remains constant even though the current through the
Zener diode changes.
(e) Similarly, if the input voltage decreases, the current through R s and Zener diode also decreases.
R R

(f) The voltage drop across R s decreases without any change in the voltage across the Zener diode.
R R

(g) Thus any increase/ decrease in the input voltage results in, increase/ decrease of the voltage drop
across R s without any change in voltage across the Zener diode.
R R

(h) Thus the Zener diode acts as a voltage regulator.

4. Optoelectronic devices:
The p-n junctions can be designed so that current through them changes either by causing electron
excitation (from valence band to the conduction band) by light photons, or electron excitation by
suitablebias voltage resulting in emitting of light photons are called optoelectronic devices. They are
classified as follows: (i) Photodetectors for detecting light signals, (ii) Photovoltaic devices to convert
light energy into electricity, (iii) Devices to convert electrical energy into light.

170
5. Photodiode:
A Photodiode is a special purpose p-n junction diode fabricated
with a transparent window to allow light to fall on the diode. It
is operated underreverse bias.
(a) When the photodiode is illuminated with light (photons) with energy (hν)
greater than the energy gap (E g ) of the semiconductor, then electron-hole pairs are generated
R R

due to the absorption of photons.


(b) The diode is fabricated such that the generation of e-h pairs takes place in or near the
depletion region of the diode.
(c) Due to electric field of the junction, electrons and holes are separated such that electrons
reach n-side and holes reach p-side.
(d) Electrons are collected on n-side and holes are collected on p-side giving rise to an emf.

(e) When an external load is connected, current flows. The magnitude of the photocurrent depends on
the intensity of incident light (photocurrent is proportional to incident light intensity).
(f) It is easier to observe the change in the current with change in the light intensity, if a reverse bias is
applied. Thus photodiode can be used as a photo detector to detect optical signals.
(g) The I-V characteristics of photodiode are given in fig 26.

6. Light Emitting Diode (LED):


It is a heavily doped forward biased p-n junction which spontaneously converts the biasing electrical
energy into optical energy, like infrared and visible light. In it, a p-n junction made from translucent
semiconductor such as GaAs is provided with metallic contacts. When it is forward biased through a series
resistance R, light photons are emitted from the non-metallised surface of p-region. The resistance R limits
the current through LED and hence controls the intensity of light emitted by it.
Working:
(a) When the diode is forward biased, electrons are sent from n → p (where they
are minority carriers) and holes are sent from p → n (where they are minority
carriers).
(b) At the junction boundary the concentration of minority carriers
increases compared to the equilibrium concentration (i.e., when there is no bias).
(c) Thus at the junction boundary on either side of the junction, excess minority carriers are
there which recombine with majority carriers near the junction. Electron are in
conduction band and hole are in valance band so energy of electron is more than that of
hole. So on recombination, this difference in energy is released in the form of photons in
visible region of EMW.
(d) When the forward current of the diode is small, the intensity of light emitted is small. As the forward
current increases, intensity of light increases and reaches a maximum. Further increase in the
forward current results in decrease of light intensity.
(e) The V-I characteristics of a LED is similar to that of a Si junction diode.
(f) But the threshold voltages are much higher and slightly different for each colour.

171
LEDs have the following advantages over conventional incandescent low power lamps:
(a) Low operational voltage and less power.
(b) Fast action and no warm-up time required.
(c) The bandwidth of emitted light is 100 Å to 500 Å or in other words it is nearly (but
not exactly)monochromatic.
(d) Long life and ruggedness.
(e) Fast on-off switching capability.

7. Solar cell:
It is a junction diode which converts solar energy into electricity and is based on
U

photovoltaic effect(generation of voltage due to bombardment of light photons).


UU U

Construction: It consists of a p-n junction made from Si or


GaAs. Herea thin layer of p-type is grown on n-type
semiconductor. The top of the p-layer is provided with
some finger electrodes. This leaves enough space for the
light to reach the thin p-layer and hence the underlying p-
n junction. The bottom of n-layer is provided with a
current collecting electrode.
Working:
(i) The generation of emf by a solar cell, when light falls
on, it is due to the following three basic processes:
generation, separation and collection—
(a) Generation of e-h pairs due to light (with hν >
E g ) close to the junction;
R R

(b) Separation of electrons and holes due to


electric field of the depletion region. Electrons
are swept to n-side and holes to p-side;
(c) The electrons reaching the n-side are collected by the
front contact and holes reaching p-side are collected
by the back contact. Thus p-side becomes positive and
n-side becomes negative giving rise to photovoltage.
Semiconductors with band gap close to 1.5 eV are
ideal materials for solar cell fabrication.
Uses: Solar cells are used to power electronic devices in satellites and space vehicles and
also as power supply to some calculators. The V-I characteristics of solar cell are given as
follows.

Solved Examples
Eg 1: Why Si or GaAs is used in formation of PN-junction
diode for solarcell? [NCERT]
Ans: The solar radiation spectrum received by us is
shown in Fig.The maximum is near 1.5 eV.
For photo-excitation, hν >Eg. Hence, semiconductor with band gap
~1.5 eV or lower is likely to give better solar conversion
efficiency.Silicon has Eg
~ 1.1 eV while for GaAs it is ~1.53 eV. So Si and GaAs are
used in solarcell.
Note: Although E g of GaAs is more than that of Si but
R R

GaAs is preferred over Si because GaAs has higher


absorption coefficient of radiation.

172
Classroom Questions:-
Level – 1
1. A p-n photodiode is fabricated from a semiconductor with band gap of 2.8 eV. Can it detected a
wavelength of 6000 nm? Justify. [Ans. No, because energy of 6000 nm wavelength is E = 0.2 eV and it is
less than that of band gap = 2.8 eV] [Delhi 2013, Outside Delhi 2013, NCERT] [2 marks]
Level – 2
2. A p-n junction diode when forward biased has a drop of 0.5V which is assumed to be independent of
temperature. The current in excess of 10mA through the diode produces a large joule heating which
damages the diode. If we want to use 1.5 V battery to forward bias the diode, what should be the value
of resistor used in series with the diode so that maximum current does not exceed 5mA? Ans. 200Ω
[2 marks]
3. In a Zener regulated power supply a Zener diode with V Z = 6.0 V is used for regulation. The load current
R R

is to be 4.0 mA and the unregulated input is 10.0 V. What should be the value of series resistor R S ?
R R

Ans. 167Ω [NCERT] [2 marks]


4. Explain briefly, with the help of circuit diagram, how V—I characteristics of a p-n junction diode are
obtained in (i) forward bias, and (ii) reverse bias. Draw the shape of the curves obtained. (b) A
semiconductor has equal electron and hole concentration of 6×108/m3 . On doping with certain
P P P P

impurity,electron concentration increases to 9×1012/m3. (i) Identify the new semiconductor obtained
P P P P

after doping. (ii) Calculate the new hole concentration. [Delhi, Outside Delhi ] [5 marks]

173
Homework Sheet 14.3
Theory
1. What are optoelectronic devices? [Ans: These are the devices which convert electric
energy to light energy or vice-versa.] [1 mark]
2. State the function of zener diode in a circuit. [Hint: Refer to pt. 1] [1 mark]
3. Name the junction diode whose I-V characteristics are drawn below
[Delhi Board 2017] [1 mark]
4. Draw the circuit diagram of an illuminated photodiode in reverse bias.
How is photodiode used to measure light intensity?
[Hint: Refer to pt. 4 and fig. 25] [Delhi 2010] [2 marks]

5. Explain the working and use of solar diode. [Hint: Refer to pt. 6] [2 marks]
6. How does a light emitting diode (LED) work? Give two advantages of LED’s over the conventional
incandescent lamps.[Hint: Refer to pt. 5] [Outside Delhi 2012, 2010] [3 marks]
7. State the working principle of an LED. Write any two important advantages and to disadvantages of
LED. [Delhi 2022] [3 marks]
8. (a) In the following diagram, which bulb out of B 1 and B 2 will glow and
R R R R

why?
(b) Draw a diagram of an illuminated p-n junction solar cell.
(c) Explain briefly the three processes due to which generation of emf
takes place in a solar cell.
9. (a) In the following diagram ‘S’ is a semiconductor. Would you increase or
decrease the value of R to keep the reading of the ammeter A constant when
S is heated? Give reason for your answer.
(b) Draw the circuit diagram of a photodiode and explain its working. Draw its
I-V characteristics.

10. Zener diode is fabricated by heavily doping both p- and n- sides of the junction. Explain, why? Briefly
explain the use of zener diode as a dc voltage regulator with the help of a circuit diagram.
[delhi 2017, 2020] [3 marks]
11. Why is a zener diode considered as a special purpose semiconductor diode? Draw the I-V
characteristics of a zener diode and explain briefly how the reverse current suddenly increases at the
breakdown voltage. Describe briefly with the help of a circuit diagram how a zener diode works to
obtain a constant dc voltage from the unregulated dc output of a rectifier.[Hint: Refer to pt. 1 and
2][Outside Delhi 2012, Compartment 2012, Delhi 2011, Foreign 2010, 2020] [5 marks]
Application Based
12. The current in the forward bias is known to be more (mA) than the current in the reverse bias
(µA). Whatis the reason, then, to operate the photodiode in reverse bias?
Ans: When photo diode is illuminated with light then due to breaking of covalent bonds, equal
number of additional electron and holes come into existence whereas fractional change in minority
charge carrier is much higher than fractional change in majority charge carrier. Since the fractional
change of minority carrier current is measurable significantly in reverse bias than that of forward
bias. Therefore,photo diodes are connected in reverse bias.
[Compartment 2012, Delhi 2012, Outside Delhi 2012] [1 mark]
10. Why Si and GaAs are are preferred materials for solar cells? [NCERT] [1 mark]
11. Why cannot we use Si ad Ge in fabrication of visible LEDs? [Delhi 2020] [1 mark]
12. A p-n photodiode is fabricated from a semiconductor with band gap of 2.8 eV. Can it detect a
ℎ𝑐
wavelength of 6000 nm? Justify. [Ans: No, because energy of 6000 nm wavelength is E =
λ

174
(6.63 ×10−34 )(3 × 108 )
= = 0.2 ev and it is less than that of band gap = 2.8 eV]
(6000 × 10−9 )(1.6 × 10−19 )
[Delhi 2013, Outside Delhi 2013, NCERT] [2 marks]
Numerical
13. A p-n junction diode when forward biased has a drop of 0.5V which is assumed to be independent of
temperature. The current in excess of 10mA through the diode produces a large joule heating which
damages the diode. If we want to use 1.5 V battery to forward bias the diode, what should be the
value of resistor used in series with the diode so that maximum current does not exceed 5mA? Ans.
200Ω [2 marks]
14. In a Zener regulated power supply a Zener diode with V Z = 6.0 V is used for regulation. The load
R R

current is to be 4.0 mA and the unregulated input is 10.0 V. What should be the value of series
resistor R S ? Ans. 167Ω [NCERT]
R R [2 marks]

15. Explain briefly, with the help of circuit diagram, how V—I characteristics of a p-n junction diode are
obtained in (i) forward bias, and (ii) reverse bias. Draw the shape of the curves obtained. (b) A
semiconductor has equal electron and hole concentration of 6×108/m3 . On doping with certain
P P P P

impurity,electron concentration increases to 9×1012/m3. (i) Identify the new semiconductor obtained
P P P P

after doping. (ii) Calculate the new hole concentration.


[Delhi, Outside Delhi ] [5 marks]

175
(Space for classroom notes)
U

176
(Space for classroom notes)
U

177

You might also like